Sunteți pe pagina 1din 164

Bus Orgs: Fall 2009 (Albert)

AGENCY

i. Who is an agent? a. Giving someone the power to bind you in K i. Legal std where we look at statute; if your conduct matches the statute, then we have agency ii. Restatement is NOT the law until enacted by legislature b. Voluntary relationship i. Does not necessarily need to be in writing c. Example: i. I want you to go to the meeting for me and sign these 5 documents ii. The signature line is set up for Miriam Albert 1. He signs John Campo as agent for Miriam Albert d. When agent exceeds scope of his/her authority, the agent is bound, NOT the principal e. Master-Servant relationship is missing from 3rd restatement i. In Master-Servant, the master controls how the agent does what he does f. Example: i. If I hire you to rent my house, that is principal-agent ii. Relevant ONLY because in order to have tort liability for your agent, your agent must be a servant g. Gorton v. Doty (1937) i. Doty = teacher at HS and volunteers car to take the team to a game at another school ii. Coach involved in accident while driving car and Gorton sues Doty (teacher, owner of car) for injuries iii. Agency: relation that exists when one person acts for another 1. Principal and agent 2. Master and servant 3. Employer/proprietor and independent contractor iv. Restatement of Agency 1: relationship that results from manifestation of consent by one person to another that the other shall act on his behalf and subject to his control, and consent by the other so to act v. Court decided that coach was acting as an agent of Doty (principal) and she was liable 1. The only manifestation of consent is the conversation between Doty and coach in which Doty agreed to allow coach to use car if he drove it a. Not necessary that there is a K or that the agent makes a promise; compensation also not necessary. There only needs to be an understanding or an agreement b. Only you may drive my car = she is dictating how her car will be driven (condition to her lending the car) c. She does NOT say I want you to do this FOR me 2. When an agent acts in the scope of agency, the principal can be liable 3. Dissent says that there is no agency, Doty was simply loaning the car to coach (when she said that coach had to drive it, she meant that none of the boys could drive the car) a. No manifestation that you do this for me b. Everyone agrees that Restatement 1 is the right test c. Prudent auto owners have insurance i. Shes either a prudent owner, or shes NOT a prudent owner and does not have insurance vi. Long caselaw history that driver was presumed agent of owner of car ii. Equal Dignities Rule 1

a. An agency agreement only has to be in writing to be enforceable if the contract the agent is authorized to sign must be in writing under the Statute Of Frauds b. A. Gay Jenson Farms Co. v. Cargill, Inc. i. Ps are farmers who sold grain to D Warren ii. D Cargill financed Warren and Ps allege that D also acted as a principal on Ks made by Warren through the course of its dealings with Warren 1. Warren is doing basic financing 2. Cargill needs the money because they need the grain market iii. Ct finds that an agency exists here this is a non-traditional context not a voluntary agency the only way that D pays the farmers is if there is an agency here if there is one, it is judicially imposed, not a voluntary agency this is not a Restatement 1 agency finding of agency is via Restatement 14 O Cargills status is changed by the court 1. Creditor who assumes control of his debtors business may become liable as principal for the acts of the debtor in connection with the business iv. Court finds that an agency relationship exists based on the extent of Cs control and influence over W it become a principal with liability for transaction entered into by Warren, which acted as its agent 9 factors that led court to its decision: 1. (i) Cs constant recommendations to W by telephone 2. (ii) Cs right of first refusal on grain 3. (iii) Ws inability to enter into mortgages, to purchase stock or to pay dividends without Cargills approval 4. (iv) Cs right of entry onto Ws premises to carry on periodic checks and audits 5. (v) Cs correspondence and criticism regarding Ws finances, officers salaries and inventory 6. (vi) Cs determination that W needed strong paternal guidance 7. (vii) Provision of drafts and forms to W upon which Cs name was imprinted 8. (viii) Financing of all Ws purchases of grain and operating expenses 9. (ix) Cs power to discontinue the financing of Ws operations v. Based on these 9 factors C assumed de facto control over W agency created by 14 O, creditor becomes a principal here: on their own, none of these factors are too glaring, but when they are considered in the totality of circumstances, C starts to sound less like a creditor and much more like being involved in the business they are starting to sound like they are running the business (had their name on letterhead, etc there was such a high degree of control over the day to day operations of the business that the status was changed from that of a creditor to a principal) 1. Comment to RS 14O: A security holder who merely exercises a veto power over the business acts of his debtor by preventing a purchase or sales above specified amounts does not thereby become a principal HOWEVER, if he takes over the management of the debtors business either in person or through an agency, and directs what contracts may or may not be made, he becomes a principal, liable as any principal for the obligations incurred thereafter in the normal course of business by the debtor who has now become his general agent the point at which the status changes is when the creditor assumes de facto control over the conduct of his debtor, whatever the terms o the formal contract with his debtor may be. 2. If you are in your debtors business too much, you lose role as creditor and become agent vi. Ways to Create Agency: 1. By agreement 2. By ratification 2

a. This would be where you didnt have a successful agency, but you wind up with one 3. By estoppel 4. Agents acting with authority may bind principals a. Authority is the starting point for analysis of contract actions b. Authority also is an element in vicarious liability-based tort actions against the principal (esp. the scope of employment requirement) iii. Liability of Principal to Third Parties in Contract a. Mill Street Church of Christ v. Hogan i. Elders hired Hogan to paint church and maybe another church member (Petty) to assist 1. Hogan had been hired in the past and was allowed to hire his bro for help 2. Bro took the job, ladder broke, and he broke his arm & filed for workers comp ii. Person alleging agency and resulting authority has burden of proving that it exists iii. Hogan had actual implied authority to hire his brother because: 1. Based on need and past experience in hiring helpers for work for the Church iv. Hogan also had apparent authority, since P church had previously allowed painter to hire bro b. Dweck v. Nasser i. The Parties 1. Dweck formerly was president, CEO, member of board of Kids Intl a. 30% stockholder 2. Nasser is Spanish citizen in Switzerland; chairman of board of directors and controls 52.5% of its equity ii. The Settlement Agreement 1. Purports to settle all claims/disputes among parties 2. Dweck must pay 52.5% of profits from 1/1/01 until 5/18/05 generated by entities that allegedly competed with Kids 3. 1.05 million payment from Dweck to Nasser as reimbursement for litigation expenses he incurred in connection with Kids dispute 4. Nasser had to compensate Dweck for value of her 30% equity interest iii. History of Case 1. Dispute happened in 1/2005, when Nasser discovered Dweck operating competing businesses out of Kids offices 2. Nasser removed Dweck as president and replaced her with his nephew 3. Parties discussed settlement in early 2005 but did not reach agreement 4. Dweck filed complaint in court challenging Nassers termination of her employment a. Nasser filed motion to dismiss i. Motion to dismiss claim that Nasser breached his fiduciary duties by replacing Dweck with his allegedly unqualified nephew 5. This court granted Nassers motion: upheld Nassers termination of Dweck 6. Passage of time to 2007: Dweck hired attorney (Wachtel) to facilitate settlement a. Nassers attorney of record is Heyman b. Wachtel reached out to Shiboleth, Nassers close friend and primary attorney for many years! i. Wachtel knew Shiboleth many years because of Shiboleths representation of Nasser ii. Shiboleth worked with Wachtel iv. Nasser is Bound to the Settlement Agreement 1. Three Sources of Agency Agreement a. Actual Authority: expressly granted authority orally or in writing 3

b. Implied authority: derivation of actual authority; often means actual authority either (1) to do what is necessary, usual, and proper to accomplish or perform an agents express responsibilities or (2) to act in a manner in which an agent believes the principal wishes the agent to act based on agents reasonable interpretation of principals manifestation in light of the principals objectives and other facts known to the agent c. Apparent authority: such power as a principal holds his agent out as possessing or permits him to exercise under such circumstances as to preclude a denial of its existence 2. Plaintiffs claim there is sufficient evidence to show that Shiboleth was vested with all three sources a. Actual: Nasser granted Shiboleth authority to settle litigation b. Implied: Nassers course of dealings with Shiboleth over the past 20 years make it clear that Shiboleth had at least implied authority to settle litigation i. Was permitted to speak in Nassers name c. Apparent: I will tell you to sign the agreement you shall do so c. RESTATEMENT 7 authority is the power of the agent to affect the legal relations of the principal by acts done in accordance with the principals manifestations of consent to him d. Categories of authority are ways of classifying the proof P must offer to bind the principal to contract e. TYPES OF AUTHORITY (FROM CASE): i. Actual 1. Actual express authority a. Requires manifestation of consent from the principal to agent b. You intentionally give someone authority c. EXAMPLE: i. it is authority that P intentionally confers on A; ii. P tells A to do X, and A does X. iii. P is bound as per 144. d. RESTATEMENT 2d 26 Creation of Authority; General Rule i. Authority to do an act can be created by written or spoken words or other conduct of the principal which causes the agent to believe that the principal desires him so to act on the principals account e. HYPO:Wal-Mart wants Federline to be its new spokesperson. Wal-Mart authorizes president Lee Scott to sign Kevin to a contract and he does so. When Lee Scott signs up Kevin, as per boards instructions, what authority is created? i. AIA bc P cant always think of everything that it would authorize A to do. If, in order to carry out Ps instructions to A, A takes other steps necessary to carry out the instructions, P is bound. 2. Actual implied authority a. Requires manifestation of consent from the principal to agent b. You intentionally give someone authority c. Implied authority to do whats necessary to discharge As express authority d. RESTATEMENT 2d 35 when incidental authority is inferred i. Unless otherwise agreed, authority to conduct a transaction includes authority to do acts which are incidental to it, usually accompany it or are necessary to accomplish it e. POLICY i. Onus is on the principal to draft instructions to agent carefully ii. Both AEA and AIA depend on communication between P and A 4

iii. Includes such powers as are practically necessary to carry out the duties actually delegated (Mill Street) 3. Actual authority by virtue of Custom:
a. b. c. d. Principal doesnt give the agent instructions Similar to implied/incidental authority (except there are less instructions) the industry or trade custom grants the authority Even if P has denied authority to A, it still may be authorized by virtue of custom

ii. Apparent 1. This focuses on what the principal says and their conduct toward third parties 2. AA exists only where there is some connection between third party and the principal. a. Its insufficient for A to make an unauthorized assertion of his authority. b. We need words or conduct by P or some business custom which leads TP to believe that A has the requisite authority. 3. The agent can be the vehicle of the manifestation from the principal to TP 4. This is Authority that the principal lets the TP think that the agent has a. TP justifiably relies on Ps communication of consent for A to act on his behalf. 5. not actual authority but is the authority the agent has held out the Principal as possessing (Mill Street v. Hogan) 6. QUICK TEST TO DETERMINE APPARENT OR ACTUAL a. Did P communicate his consent for A to act for him to A or to TP? i. If to A, AEA ii. If to TP, AA. 7. RESTATEMENT 2d 8 apparent authority a. Power to affect legal relations of another person by transactions with TPs, as an agent for the other, arising from and in accordance with others manifestations i. Apparent authority exists only to the extent that it is reasonable for TP dealing with agent to believe that agent is authorized. Further, the third person must believe the agent to be authorized. ii. Comment B: manifestation of P may be made directly to a TP, or may be made to community by signs, by advertising, by authorizing the agent to state that he is authorized, or by continuously employing the agent. 8. RESTATEMENT 2d 159 apparent authority a. A disclosed or partially disclosed principal is subject to liability upon contracts made by an agent acting within his apparent authority if made in proper form and with the understanding that the apparent principal is a party b. The rules as to the liability of a principal for authorized acts, are applicable to unauthorized acts which are apparently authorized 9. RESTATEMENT 2d 27 creation of apparent authority a. Its created as to a third party by written/spoken words or any other conduct of principal which causes the third person to believe that the principal consents to have the act done on his behalf by the person purporting to act for him 10. POLICY: a. Apparent Authority incentivizes careful drafting of authorization on right people b. If Principal doesnt want to be bound by Agent, then Principal shouldnt tell other people he will be bound f. RESTATEMENT 2d 34 How To Interpret Authority i. An authorization is interpreted in light of all circumstances including: 5

1. Situation of parties, relations to each other, and the business in which theyre engaged 2. General usages of business, trades or employments to which authorization relates 3. Facts that agent has notice of respecting the objects the principal wants to accomplish 4. Subject matter, circumstances under which the act is to be performed and its legality 5. Formality and care with which an instrument showing authority is drawn g. Three-Seventy Leasing Corporation v. Ampex (apparent authority) i. 370 seeks damages from Ampex for breach of K to sell 6 computer memories 1. 370 buys computer hardware to lease to others 2. 370 had verbal commitment to lease these units to EDS 3. Written document submitted by Ampex to 370 for purchase of 6 computer units at specific pricing delivery was to be made to EDS signature block for rep of each company 4. 370 received this document and signed it, but never signed by Ampex 5. 370 argues that this document was offer to sell by Ampex, accepted by 370 upon signature a. Ampex says document was only a solicitation which became offer to purchase upon signing by 370, and this offer was never accepted by Ampex ii. Letter on co. stationery sent by sales rep after unsigned contract was sent to TP created AA iii. To ensure there is no longer apparent authority you must give actual or constructive notice to all Third Parties who dealt with the agent iv. RULE an agent has apparent authority to bind P when P acts in a manner as would lead a prudent person to suppose that A had authority he purports to exercise 1. Absent knowledge on the part of TPs to the contrary, A has the apparent authority to do those things which are usual and proper to the business v. Apparent authority by virtue of custom:TP must KNOW Principal has placed Agent in a certain position and it must be customary for Agents in that position to enter this type of agreement iv. Consequences of creating an agency relationship a. Ratification (opting into an agency / choosing to be bound) i. How does P ratify? 1. Must be a K or act that would have been valid if authorized initially [cant be illegal] a. P must have been in existence when the unauthorized act was done 2. Must be effected with same formalities that would be necessary for unauthorized act 3. Affirmation P must know or have reason to know all material facts / can be through: a. express affirmation by the principal; b. implied affirmance thru acceptance of benefits when its possible to decline c. implied affirmance through silence or inactiona principal cant wait forever d. Implied affirmation through bringing a lawsuit to enforce the contract. 4. Ratification will be denied legal effect where necessary to protect rights of innocent TP a. No intervening events: TP withdraws, P loses capacity, change in circumstances, lapse of time. TP is treated as an offeror, who can withdraw any time prior to acceptance ii. ELEMENTS OF RATIFICATION:
1. 2. 3. 4. 5. Intent to ratify; Full knowledge of all material circumstances Contract would have been valid if the act had been authorized Principal had to be in existence at the time of the unauthorized act Third Person is still in the contract a. It is NOT ENOUGH that the principal receives proceeds from a transaction not purportedly done on his account to be ratification

iii. RESTATEMENT 2d 82 Ratification is the affirmance by a person of a prior act which did not bind him but which was done or professedly done on his account, whereby the act, as to some or all persons, is given as if orignially authorized by him. iv. RESTATEMENT 2d 83 affirmance is either:
1. EXPRESS Manifestation of an election by one on whose account an unauthorized act has been done to treat the act as authorized or 2. IMPLIED Conduct by him justifiable only if there were such an election

v. Botticello v. Stefanovicz 1. Walter leases the farm to Botticello for $85,000 with an option to purchase a. Mary, Walters wife, keeps saying that she will not sell farm to him for less than $85,000 b. Walter and attorney draw up an agreement, signs it, but he doesnt fully own prop 2. No evidence that Walter was the agent of his wife Mary a. Marys part ownership found later 3. Marriage cannot create an agency relationship 4. No indication that Mary had intent to ratify agreement or knew the circumstances surrounding the deal a. Mary would have had to ratify knowing fully all of the circumstances surrounding the contract and it is not clear that she actually did b. Ratification is defined as the affirmance by a person of a prior act which did not bind him but which was done or professionally done on his account b. HYPOS: a. Suppose Jon wants his assistant the lovely young Haley to negotiate and sign a contract for him with Walmart. What if Jon faxes a letter to the assistant and a copy to Walmart stating that the assistant is authorized to contract on his behalf. What authority is created? i. P=Jon ii. A=Haley iii. TP=Walmart iv. Actual Express Authority with Haley because it is coming from the principal to the agent. He has allowed her to contract on his behalf, in writing. (P A) v. By faxing it to the TP (Walmart), he is also creating Apparent Authority. Her AA is broader than her actual authority. (P TP) b. What if, at the bottom of the copy of the letter faxed to the assistant only, he scrawls, But call me first babe to get my permission. What if Jons assistant goes ahead and signs the contract without calling him. Is Jon bound? i. The scope of her actual authority was that she was reqd to call him. He is not bound under actual authority (P A) ii. He is bound under Apparent Authority (P TP). Authority was given by Jon to Wal-Mart. (She is acting under her apparent authority). c. What if, at the bottom of his letter he faxes to Haley and Walmart, he scrawls, But call me first babe to get my permission. i. He is still not bound under actual authority, but now he is also not bound under Apparent Authority. As long as the limitation goes to both parties, he is not going to be bound. ii. POLICY: AA places the incentive to draft careful authorizations on the right people. If P doesnt want to be bound by A, then P tells other people he will be bound. 1. A limitation on actual authority is only binding on TPs that have notice. a. Notice of the limitation on actual authority operates as a limitation on AA. iii. If there is AA, can TP enforce the contract against P? 1. Then yes it can be enforced 7

d. Now lets change the facts to remove Walmarts copy of the letter. Now can Jon [P] enforce it against TP? i. If Haley doesnt abide by her limitation ii. Yes, he can be bound by ratification. iii. She did not call him first, even though he told her to. Can he enforce the K? 1. Yes: ratification 2. When agent goes too far, the principal is not bound. a. Principal CAN become bound if he chooses to be; we call this Ratification b. P agrees to be bound; agency is created just for that K c. Agency by ratification gives the P the option to choose to be bound e. Suppose Lindsay Lohan (whom Jon has never met but feels like he has through his close relationship with Lindsays dad) negotiates a contract on his behalf with Walmart. Lindsay sends the contract to Jon. Jon then contacts either Lindsay or Walmart, and says that he likes the contract. i. Lindsay did NOT have actual or apparent authority, but he can choose to be bound ii. This is called ratification. 1. Is ratification fair to the TP? a. TP thought they were doing business with P; TP ends up with what they thought they were going to get i. Yes, as long as the TP doesnt find out about the lack of authority and want to get out. ii. If Walmart found out that Lindsay didnt have actual authority, they might want to get out. f. One more example i. Pam owns Whiteacre. Al is her real estate broker, her agent. Ted is an outsider who claims that Al entered into a contract on Pams behalf to sell Whiteacre to Ted. 1. This means they have a voluntary relationship. He has given her a scope of authority. That agreement has to be in writing b/c contracts involving real estate have to be in writing according to the SoF. 2. If the manifestation has come from P to A we are talking about actual authority. 3. The manifestation by Pam to Ted would be apparent authority. 4. The difference here stems from what evidence Ted is seeking to here. a. If he is relying on a manifestation from P to A, he is relying on actual authority. b. If he is relying on a manifestation from P to TP, he is relying on apparent authority. (Pam sends Ted a letter saying she contracted with Al to have him sell her property) 5. If we have a situation where the P went to the trouble of limiting the actual authority to the A but failed to convey that limitation to the TP, we are more worried about protecting that TP. 6. The P could have avoided this by conveying that limitation so we are less interested in protecting them as opposed to that innocent TP. 7. Need to find out how that TP learned of that authority and see where it came from. 8. What we dont want to see is the agent claiming authority where they dont. 9. The only way the agent becomes the vehicle for creating apparent authority is where the agent is authorized to do that. ii. Suppose Ted seeks to prove the existence of authority by evidence relating to communications between Pam and Al; maybe a letter from Pam to Al in which Pam directed Al to sell Whiteacre. 1. What type of authority is Ted trying to establish? 8

a. Actual Express Authority iii. But suppose Ted seeks to establish authority by evidence relating to communications from Pam to Ted. 1. Suppose Pam sent Ted a letter in which she said that she had ordered Al to sell Whiteacre. In this case, what type of authority would Ted be trying to establish by introducing the letter into evidence? a. Apparent Authority g. The difference between actual and apparent authority arises out of the way in which Ted seeks to prove that Al empowered to enter into the contract. h. In other words, the different categories of authority really are ways of classifying the proof the plaintiff must offer to bind the principal to the contract. i. CRITICAL POINT: i. Apparent authority exists only where there is some connection between the third party and the principal. ii. You must always look at how TP learned of the As alleged authority and ask whether P reasonably can be said to have been the source of that knowledge. iii. Its insufficient for A to make an unauthorized assertion of his authority. iv. We need words or conduct by P or some business custom that leads TP to believe that A has the requisite authority. j. So, for example, suppose Al tells Ted that he is Pams agent. Does Al have apparent authority? Could Al have actual authority? i. If Pam told Al go find Ted and tell him I said you can do X, Al would have actual authority. ii. In order to determine whether Al has apparent authority, we need to take it back and see whether he has actual authority. k. Suppose Pam had been present when Al made his statement to Ted. She was silent. Now does Al have apparent authority? i. Yes, b/c she would say something if this wasnt true (estoppelprohibition based on conduct) b. Estoppel (remedy in contract NOT tort to protect the TP)
i. If there was no agency relationship, then ask whether there was agency by estoppel?

ii. ELEMENTS OF ESTOPPEL: 1. Principal creates thru intentional or negligent words, acts, inaction or omissions an appearance of authority in purported agent 2. TP reasonably & in good faith acts in reliance on appearance of authority to detriment a. Third party is unaware that there is no agency b. The principal argues to get out of an agency by estoppel i. Principal is ESTOPPED from denying that an agency exists ii. agent without authority is seeking to bind the principal 3. TP changes her position in reliance upon the appearance of authority a. Authority created by estoppel = APPARENT AUTHORITY iii. RESTATEMENT 2d 8B Estoppel, Change of Position 1. A person not party to a transaction purportedly done on his account can be liable if he: a. intentionally or carelessly caused the belief [AA] b. or failed to correct the belief, and the TP reasonably relies on the belief iv. FUN FACT ABOUT ESTOPPEL 1. Where agent had authority (of any kind) contract is binding on both P and T. a. Estoppel only binds Principal (it is a remedy) b. In all other types of authority, the contract is binding on P and TP. Estoppel is a one way street. P has no cause of action at all in an estoppel situation. 9

v. Hoddeson v. Koos Bros. 1. Hoddeson bought furniture but it wasnt in stock and she didnt get a receipt a. The man, presumably the salesman, took out a pad and wrote down her order and she handed him cash b. She did not ask for a receipt 2. Hoddeson believed that someone not a salesman had duped her and taken her $ a. She argued that agency was formed by estoppel in that the stores lack of supervision allowed an impostor to impersonate one of their salesman b. She has burden of proving this 3. Holding court finds that Hoddeson used ordinary prudence and that the store allowed there to be an agent by estoppel v. ANALYSIS AND PROBLEMS a. Pam the Writer: i. Will use a principal-agency analysis 1. Principal: Pam 2. Agent: Alex (her husband) 3. TP: ABC ii. Question is, if Alex in fact had no authority to enter into a K on her behalf, can Pam be bound? 1. Under those facts, it makes sense that Alex is liable and not Pam. An agent that exceeds his authority binds only himself. iii. But, we have an additional fact, the cashing of the check. 1. Does this amount to ratification? a. If she was a writer who had written many books and the check doesnt specify which book the check is for, that might change things. b. If we could show that she wasnt affirmatively agreeing to be bound, there will be no ratification. c. On the facts that we have, ABC is going to win. iv. When we add the fact that she thought the check was for royalties on one of her previous books that was published by ABC, then her argument that this was not ratification is strengthened. b. Alan the Deranged Fan: i. Will use a principal-agency analysis again: 1. Principal: Pam 2. Agent: Alan 3. TP: The Landscaping Company ii. Will the company win on its claim that she ratified the K by accepting and retaining its benefits? 1. She never authorized Alan to do this 2. She also did not ratify b/c she did not engage in any sort of conduct that makes it look like she is affirming this K; by the time she got home, it was completed 3. It would be different if she came home when they were just setting up and getting ready to perform the landscaping a. What is the difference between her saying thanks and cashing the check? 4. In order for it to be an affirmance we either need language affirming or conduct that explicitly demonstrates her desire to be bound 5. She is not bound unless she chooses to be bound and we cant see by her conduct that she is choosing to be bound c. Paula the Investor: i. She has given her broker a limited grant of authority 10

ii. Has there been a ratification here? 1. If she didnt want to ratify, she would have called him right then. 2. Even waiting a day might be ok. 3. However, as time goes on, the argument against ratification weakens. 4. We are heading towards implied affirmance by silence in an action. 5. Essentially by not disapproving, she will end up ratifying the K 6. The length of time the would-be principal waits is relevant here and it will be a question for the jury. d. Whiteacre Manor: i. Principal-Agency Analysis: 1. P = Paula 2. A = Alan 3. TP = Ted ii. We will not hold a TP to a K at his peril. iii. She cannot ratify after the fact; she would have been able to ratify before the mansion burned to the ground. iv. Key is fairness; a material fact to the K has changed and the TP needs to be given a chance to pull out of the deal e. Paula the Producer Hypos 1. Paula is a movie producer who has hired an assistant, Amy. Paula says to Amy, Go out and hire a camera person. How would you characterize Amys authority to hire? a. Actual express authority is hire a camera person b. Actual implied authority is to do whatever is necessary, e.g., interview, take out ads, etc. 2. Tom, a camera person who is desperately looking for work, calls Paula. He says, I hear youre going to be making another one of your fantastic movies, darling. I would just love to work for you if I can fit it into my schedule. 3. Paula, who detests Tom but doesnt like to be the one to say no to anyone, replies, Tom darling, I would just love to be able to work with you some time, but I have turned over to Amy the job of hiring our camera person. Lets do lunch some day. Bye now. a. This gives Amy apparent authority (a TP believes from P) 4. Amy happens to be in Paulas office during this conversation. Upon hanging up the phone, Paula turns to Amy and says, Do not, under any circumstances, hire Tom. I cant stand him. a. This limits Amys actual authority. It does nothing to the apparent authority to TPs. 5. A week passes. Amy is having great difficulty finding a camera person. She has lunch with Tom, who convinces her that he would be perfect for the job and that he and Paula will get along just fine. He also says that he needs a decision in a hurry because of other opportunities. Amy cannot reach Paula. She offers the job to Tom, who accepts. a. When she offered the job to Tom she was outside the scope of her actual authority. b. However, in terms of apparent authority, it was not limited. 6. When Paula hears of this, she fires Amy and refuses to honor the commitment to Tom. Tom sues. Who will win? a. There was no notice to Tom, the TP. 11

b. As such, it was reasonable for Tom to believe that Amy had the authority to hire b/c of what Paula told him. She manifested her consent and the scope of authority and as a result, she is going to be bound. 7. Lets change the facts: Tom calls Paula and leaves a message saying that he would like to talk to Paula about working on her movie. 8. Paula tells her secretary to call Tom and to tell Tom that Amy is responsible for hiring and that Tom should speak to Amy. The secretary does so. . Paula tells Amy not to hire Tom but Amy does so anyway. a. Any actual authority here? i. Her actual authority has a limitation on it. Hire a cameraman, but dont hire Tom. ii. In both cases, Paula limited Amys actual authority. b. How about apparent authority? i. There is no difference; Paula has still not limited Amys apparent authority despite the factual difference. 10. Lets change the facts. Paula tells Amy to hire a camera person. Paula also tells Amy to be nice to Tom, if she should meet him, and not to mention any conversation between the two of them about him, but not to hire Tom. 11. Tom speaks only with Amy. Amy tells Tom that she has authority to hire a camera person; in fact Tom had already heard this. Amy hires Tom. a. Apparent authority? i. We know Amy does not have the actual authority to hire Tom. Thus, the issue is whether she has apparent authority. ii. There is a difference in terms of the manifestation of authority here since he heard it through the grapevine. Paula is no longer the direct source of Toms knowledge. The key is whether Paula can be the ultimate source of Toms knowledge. iii. If Tom only knew from what Amy said, we would ask whether it was reasonable. 1. If there was evidence of a previous relationship or she had hired on other movies, that would be different than if she was a brand new hirer. iv. If he has heard from a source that can reasonably be said to have originated from the P, then apparent authority is more likely to exist. v. If Tom had not heard before what Amy said to him, she would not have apparent authority b/c she cant create her own apparent authority where none in fact exists. b. Restatement 8, Comment b may help i. Restatement 8, Comment b: The manifestation of the principal may be made directly to a third person, or may be made to the community by signs, by advertising, by authorizing the agent to state that he is authorized, or by continuously employing the agent. ii. This helps us see all the ways in which AA can be created. 12. Okay, now lets say Paula is an experienced movie producer, with a good reputation for producing family films. She decides that she wants to back the production of an R-rated movie, from which she thinks she can make a great deal of money. 13. She approaches a sleazy producer-director, Alice, who is best known for her art films, and says, Make this movie for me. Ill pay you a salary and 10 percent of the gross. 12

14. Do whatever you would do if you were the sole producer. Dont let anyone know that I am involved in any way. And, by the way, do not hire Tom to do the camera work. He worked for me on a movie last year and did a terrible job. 15. Alice accepts and then hires Tom. Then Paula decides not to make the movie. She pays off Alice, who leaves town. Tom then finds out about the contract between Paula and Alice and sues Paula. a. Who will win? i. It is clear that Amy did not have actual authority to hire Tom. ii. He cant reasonably believe there was authority here. iii. Paula is an undisclosed P. No one knew Paula was in it. iv. When Tom learned about the K, it is too late for him. v. He would need at least AA, which would come to him through a manifestation from a source which Paula is the ultimate source of. 16. Now suppose Alice hires Tom and then tells Paula. Paula reminds Alice of her instruction not to hire Tom. Alice admits error and begs forgiveness. Paula says, O.k., Ill go along with your choice. a. If she then wont pay Tom, and Tom sues, is Paula liable? i. This is ratification. ii. The P is choosing to be bound here even though the A had no authority. iii. She is bound b/c of the ratification. f. i. RESTATEMENT 2d 161 unauthorized acts of General Agent 1. Agent subjects principal to liability for acts done on his account which usually go with transactions which agent can conduct if, altho theyre forbidden by principal, other party believes that agent is allowed to do them and has no notice that hes not allowed vi. Additional hypos: a. Alice is Peters traveling salesperson, and authorized to collect accounts (that is the scope of Alices authority); before the agreed termination of the agency, Peter wrongfully discharged Alice. b. Then Alice goes to see Tom, an old customer, and collects on an account Tom owes to Peter. c. Then she calls on Laura, a new customer, and takes a big order, collects the order price, sends the order to Peter and takes off with Laura and Toms money. d. Peter delivers the goods to Laura. i. (a) Peter sues Tom on the account. Decision? 1. But what about her apparent authority? a. P can always terminate actual authority; actual notice was necessary b. Needed actual notice b/c Tom was an existing client. c. Anyone who has dealt w/ the agent as agent must be notified individually. ii. (b) Peter sues Laura for the price of the goods delivered. Decision? 1. Laura is a new client so she needed at least constructive notice. 2. When Alice goes to see Laura, she does NOT have actual authority to bind Peter 3. The only way Peter is going to win is if Alice lacks apparent authority as well a. She is not entitled to actual notice since she is a new customer b. She retains Apparent Authority because he failed to alert the rest of the world i. Bound on Toms giving money to his fired agent, because she was entitled to constructive notice 4. Would it make a difference if Peter had put an ad in the paper about firing Alice? 13

i. It does make a difference Lauras lawsuit ii. When you are terminating agency authority, you must let everyone you dealt with that hes not an agent anymore iii. Must pick a population for constructive notice the newspaper! iv. He should know who they are b. No b/c of the ratification. When Peter delivers the goods, it is the same thing as saying although my agent bound me w/o authority, I agree to be bound by it. Disclosed/Undisclosed P vii. Disclosed Principal (Where everyone knows they are the Principal): a. P had liability on K made by agent with authority; b. A has no liability UNLESS: i. It is the clear intent of all parties that A have liability; or ii. A exceeds his authority and thus binds only himself and not P viii. Undisclosed Principal (the existence and identity of the Principal are NOT known to the 3rd party) a. Agent treated as though a party to the contract b. Third party must elect who to sue i. If the 3rd party ever figures out who the P is, they are given a one-time option to switch 1. Why would you want to switch? a. If there is an upgrade to be made, e.g., in terms of assets ii. If you never find out, you are stuck ix. Partially Disclosed Principal (Where you know there is a P, but you dont know who they are, e.g., celebrities, criminals, where it might affect the price, etc.) a. These are treated like Undisclosed Principals: i. Agent is still treated as though a party to the contract ii. Third party must still elect who to sue 1. If the TP learns the identity of the P, he gets to pick 2. However, he is rolling the dice that he is stuck w/ that Agent Servant vs. Independent Contractors (you need master servant relationship in TORT) iii. RESTATMENT 2d 2 master, servant, independent contractors 1. All masters are principals, but not all principals are masters a. All servants are agents, but not all agents are servants b. Master / servant is a subset or principal / agent 2. A servant is an agent performing services in the master's affairs whose physical conduct is controlled or is subject to the right of control by the master a. If there is no principal-agent relationship then there is NO master-servant relationship and there is no way to impute vicarious liability b. Master does not have to actually exercise control over what agent does; he merely needs to have the right to control agent's physical performance 3. 2(3) An independent contractor [non servant agent] is a person who agrees to carry out some task but is NOT subject to the principal's control in doing so a. In other words, where a P sets forth the desired result but doesnt have right to tell the agent HOW to achieve that result, A is an independent contractor iv. RESTATEMENT 2d 219 when master is liable for torts of his servants 1. Master is subject to liability for torts committed by servants in the scope of employment 2. Master is not subject to liability for torts committed by servant outside scope unless: a. Master intended conduct b. Master was negligent or reckless 14

c. Conduct violated non-delegable duty of the master d. Servant purported to act/speak on behalf of principal & there was reliance upon apparent authority, or was aided in accomplishing tort by existence of agency v. RESTATEMENT 2d 220 Definition of a Servant v. Independent Contractor
1. FACTORS in determining whether someone is a servant or an independent contractor: a. Extent of control master may exercise over the details of the work b. Whether one employed is engaged in a distinct occupation i. distinct occupation shows Independent Contractor c. The kind of occupation d. Skill required in the particular occupation e. Whether employer supplies instrumentalities, tools, & place of work for worker i. If yes then this shows Master Servant relation f. Length of time person is employed Longer time shows Master Servant g. Method of payment, whether by time or by job time shows Master Servant h. Whether work is a part of regular business of employer If yes shows M-S i. Whether parties believe they are creating master servant relation

vi. RESTATEMENT 2d 228 General Statement of Scope of Employment Doctrine 1. conduct of a servant is within the scope of employment if but only if a. It is of the kind he is employed to perform b. It occurs substantially within the authorized time and space limits c. It is actuated, at least in part, by a purpose to serve the master d. Force, if used, is not unexpectable by the master vii. RESTATEMENT 2d 229 kind of conduct within scope of employment 1. List of factors to consider, including whether act is commonly done by such servants; time place and purpose of act; prior relationship, if any between master and servant etc.
viii. Liability of Principal to Third Parties in Tort

1. 3 STEPS: a. Is there an agency relationship? b. Is Agent, Principals servant or independent contractor c. If servant, was conduct within scope of employment? ix. Murphy v. Holiday Inns, Inc. 1. Murphy seeks $ for personal injuries sustained while she was a guest at the hotel a. Murphy: Holiday Inn, its agents and employees allowed her to fall b. Holiday Inn: we have no relationship with regard to operator of premises other than license agreement permitting operator of motel to use the name Holiday Inns subject to terms/conditions of licensing agreement 2. Holding no agency relationship existed a. Agency defined: fiduciary relation which results from manifestation of consent by one person to another that the other shall act on his behalf and subject to his control, and consent by the other so to act b. had no power to control Betsy-Lens business or demand a share of profits, to supervise employees, determine wages, etc. c. D did not own the premises d. Nature and extent of control agreed upon will determine whether there is agency 3. The more Holiday Inn controlled Betsy-Len = more likely master servant relationship a. franchisee and franchisor Relation may or may not be a M-S Relation b. There was NO control or right to control methods or detail of doing day-today work by Holiday Inn over this specific motel (no power to control maintenance of presmises) 15

x. Fiduciary Obligations of Agents

a. RESTATEMENT 13 an agent as fiduciary with respect to matters within scope of his agency b. RESTATEMENT 404 an agent who, in violation of duty to his principal, uses for his own purposes or those of a third person assets of the principals business is subject to liability to the principal for the value of the use. If the use predominates in producing a profit he is subject to liability, at the principals election, for such profit; he is not; however, liable for profits made by him merely by the use of time which he has contracted to devote to the principal unless he violates his duty not to act adversely or in competition with the principal. c. RESTATEMENT 2d 379 A has a duty to use reasonable care, diligence and skill in performing work; degree required is a reasonable person under similar circumstances-- duty of ordinary care. d. RESTATEMENT 2d 381 notice to A is considered notice to P, as a matter of law, so A MUST give P information A learns as A e. RESTATEMENT 2d 382 A has duty to keep and make available to P an account of all the property and money received and paid out on behalf of the principal. The agent must keep separate accounts for the principals funds and the agents personal funds, with no intermingling of these accounts. f. RESTATEMENT 2d 385 A must follow all lawful and clearly stated instructions of the principal, and must perform the services specified for as long as and in the way specified in the contract. The agent incurs liability for any losses caused by disobeying these instructions. g. RESTATEMENT 2d 386- obligations to principal go away after termination of authority h. RESTATEMENT 2d 387 A cant put his [or some TP] interest ahead of Ps [so no dual agencies] Principal always comes first, A must be selfless i. A is entitled to reasonable compensation from P; and absent agreement with P, the duty of loyalty forbids A from receiving any other compensation in connection with the agency relationship unless P knowingly and voluntarily agrees to the contrary. Clover v. Snowbird Ski Resort (Utah 1991) (Note Casep. 62) xi. An employee of the ski resort was skiing at a high speed down tan intermediate slope. He ignored a sign instructing skiers to ski slowly. He executed a jump and upon landing struck and severely injured the plaintiff. He was a chef at one of the restaurants at the resort and a supervisor of others. He was expected to ski between restaurant locations. He took a couple of runs before heading down the mountain to begin work as a chef at a restaurant located at the bottom. While he was on the way down, the accident described occurred. xii. In, Clover v. Snowbird Ski Resort, why wasnt the conduct a frolic and detouri.e., substantially removed from the authorized time and space limits of the employmentsince he was skiing for pleasure? a. Key is: when are you w/in the scope of your employment? b. If he had punched in, it begins to look more like he was within the scope of his employment. c. We have two possibilities: either the employee is within the scope or is outside of the scope d. Not a total abandonment of his employment i. He didnt have a substantial deviation e. If he was at another ski resort, then he would clearly be outside the scope f. If you are w/in the scope of your employment, vicarious liability will impute to your master i. Evaluate the scope ii. Then look to see if the agent was w/in the scope at the time in question 1. These are questions for the jury xiii. a. Town & Country House and Home Service, Inc. v. Newberry i. Plaintiff is allowed to stop D from further solicitation of customers
1. There is no fiduciary relationship between agents and homeowners 2. Dsviolated fiduciary duty where Ds used customer lists to obtain customers a. The patronage of these customers was secured by years of business effort and advertising, and expenditure of time and money

16

i. The customers of P could not be obtained by simply looking up in phonebook ii. Although they did not solicit Ps customers until they were not employed by P anymore, the Ps customers were the only ones they solicited b. P screened these clients carefully and took many steps to find them ii. Ds would not be liable for using a similar cleaning technique to compete with the business iii. RULE on Using Customer Lists / Non-compete agreements: 1. A business proprietor may not solicit former employers customers IF a. Customers are not openly engaged in business in advertised locations b. whose availability as patrons cannot readily be ascertained, c. whose patronage has been secured by years of business effort, advertising, and expenditure of time and money

CHAPTER 2: PARTNERSHIPS SECTION 1. WHAT IS A PARTNERSHIP? AND WHO ARE THE PARTNERS? CHAPTER 2: PARTNERSHIP LAW SECTION 1. WHAT IS A PARTNERSHIP? AND WHO ARE THE PARTNERS? A. PARTNERS COMPARED WITH EMPLOYEES y Builds on agency law foundation as partnership is an example/application of agency law y Statutory body of law UPA (1914) UPA (1997) y UPA Section 6(1) A partnership is an association of two or more persons to carry on as co-owners a business for profit  If we do not specify anything, the UPA provisions apply automatically y Partnerships: A partnership is a voluntary association The hallmark of a partnership is that all partners have unlimited personal liability for all partnership debts Must be able to pick your partners for that reason Must be carrying on a business-must be doing something Must be co-owners Profit-sharingbest evidence of co-ownership for purposes of a partnership definition is the sharing of profits (share profits = partners) Has to be for profita for-profit enterprise as opposed to a non-for-profit enterprise y Just b/c you say something is a partnership doesnt make it so y Also, just b/c you say something isnt a partnership doesnt mean it wont be construed as such; thus, it is possible to have a partnership and not know it y The UPA is a set of rules on how partnership should operate. y It serves as a set of default rules. If a statute speaks of a topic that you didnt cover, then you are going to be bound by what the UPA says. y Can always opt out of the UPAso think of each UPA provision as starting w/ the phrase unless the parties state otherwise If you do not opt out, you are bound by the UPA. y On the exam we will be told which UPA has been adopted for that question y Example: Creditor is owed money; partnership has no money; creditor can come after the individual partners  If creditor comes to one partner, they likely could not find the other partner 17

You must pay the full debt; then you have a cause of action against your partner Sue for breach of partnership agreement Agreement of partnership is between us

Fenwickv. Unemployment Comp. Commn (N.J. 1945) Question Presented: Whether Chesire was from Jan. 1, 1939 to Jan. 1, 1942, a partner or an employee of the prosecutor-respondent, Fenwick, trading as United Beauty Shoppe. y Facts: Chesire was a cashier and reception clerk at the shop. There was an agreement entered into by Chesire and Fenwick after she requested a wage increase and Fenwick expressed a willingness to pay higher wages if the income of the shop warranted it. The agreement said that the parties were associating themselves into a partnership.  (1) Intention of parties: agreement is not conclusive; Intent was only to enter into agreement that would provide possibility of increased compensation to Chesire; protect Fenwick from being obliged to pay increase unless business warranted it UPA 202(a) says intention is irrelevant  (2) Right to share in profits: clearly was here, but is not required in partnership. Ties into definition of co-ownership  (3) Obligation to share losses: Chesire would NOT share losses Sharing losses is not required  (4) ownership/control of partnership property/business: Fenwick contributed all capital and he had all control; she had no right to share capital upon dissolution UPA default rule: one partner = one vote All have to manage the enterprise because all have unlimited liability  (5) Community of power in administration: exclusive control of management was Fenwick only  (6) Language of agreement: language excludes her from most ordinary rights of partner  (7) conduct towards 3rd parties: filed partnership income taxes, but only held themselves out to Unemployment Commission and state income tax as partners This is NOT a requirement of partnership  (8) Rights of parties on dissolution: she had same rights as if she quit  Key phrase was that Chesire was to be given a certain percentage of the net profits if the business warrants it. This is not profit sharing. It is more like an illusory promise in contracts. y Who was disputing the existence of a partnership, and why did they care? The Unemployment Compensation Commn disputed the existence of the partnership, because if Chesire was an employee, she was the 8th& deciding employee for the purpose of determining the status of Fenwick for the year 1939 as an employer subject to the terms of the statute which requires employer payments for unemployment compensation fund y What is the definition of a partnership under NJ law? [A]n association of two or more persons to carry on as co-owners a business for profit. y What is the best evidence of co-ownership? **Sharing of the profits (if business warranted it)** y Is an agreement necessary to create a partnership? Yes, you do need an agreement Is a written agreement necessary to create a partnership?  No it does not need to be writing unless there are statute of fraud issues y Was the existence of a written agreement determinative? No, it was not. We had parties in this case that believed they had a partnership when they did not in fact have one. y Court correctly finds that there is no partnership in this case
y y

18

B. PARTNERS COMPARED WITH LENDERS Martin v. Peyton (N.Y. 1927)

y y

Facts: Hall, partner in the firm of KNK, obtains loan of $500K of Liberty bonds from Peyton, his friend  The firm was in trouble; invested in unwise speculations Hall also entered into agreement with his other friends, Perkins and Freeman, who all wanted to help him Representing KNK, and got into negotiations, and pending proposition made that Petyon, Perkins, and Freeman, should become partners Agreement in 3 documents, executed on same day, all part of one transaction  Peyton, Perkins, Freeman would loan KNK $2.5 million of liquid securities, which would be returned to them by April 1923  Firm might hypothecate to secure $2 million, using proceeds as its business necessities required To insure PPF against losses, KNK were to turn over large number of their own securities, which were so speculative that they could not be used as collateral for bank loans In compensation for loan, PPF would receive 40% of profits of firm until return made (not to exceed 500K, but more than 100K) It is still a loan, even though was xfer of securities PPF also given option to join firm if any of them had desire to do so before June 4, 1923 Until securities returned, directing of firm to be in Halls hands Whenever you see a complicated transaction like this, it generally is b/c they had to do it that way. With the cap of 40%, does that make it more or less like profit-sharing? Less b/c if you are profit-sharers, would get an unlimited amt. Seems more like this is interest; they are getting up to the amt. needed to make them whole on the loan. They were given the right to buy in and given inspection and veto rights

Issue

Did they form a partnership?


$2.5 million loan in marketable securities

PPF

Dividends 40% of profits (capped) Option to buy equity Inspection and veto rights

KNK

y y y

Only way to get to the giant bag of money is to show that KNK and PPF were in a partnership. Know they didnt want to be in a partnership. Does their conduct satisfy the definition of partnership? They are not sharing profits in a classic sense. 19

y y

Not unusual for a creditor to have certain kinds of entry rights, e.g., inspection and veto rights. The ct. correctly finds both that they did not intend a partnership and their conduct did not satisfy the definition of a partnership. Lets start with The Agreement what are the rights of the trustees under the Agreement? Rt. to be kept informed of all transactions affecting them Rt. to dividends & income accruing therefrom Can substitute for any of the securities loaned securities of equal value Their consent is reqd to sell any of the securities held by respondents, proceeds to go to them Securities loaned shall always be sufficient in value to permit of their hypothecation for $2 million if they rise in price, excess can be withdrawn by Ds; if they fall, they shall make good the deficiency Must be kept advised as to the conduct of the business & consulted as to important matters May inspect firm books & are entitled to any information they think important May veto any business they think highly speculative or injurious Each member of KNK is to assign trustees their interest in the firm What cant the trustees do under the Agreement? The trustees cant bind the firm and initiate any transaction as partner may do What did the indenture do? An indenture is a K under which creditors lend more. Mortgage of the collateral delivered by KNK to trustees to secure performance of agreement What did the option do? The option did nothing here b/c they chose not to buy in. Permits PPF to entire firm at later date by burying 50% or less of interests or any of members at a stated price Would the case have come out differently if KNK was set up as a corporation? In a corporation the investors have limited liability assuming they behaved properly so the case wouldnt come out differently b/c they still wouldnt be liable. Reconcile this with the Cargill case in Chapter 1- why are the results different? Their involvement in the entity is much less. The concept is the same but the involvement is different. Continuum of a lender whos hands off and a lender whos very hands off  A lender who is hands-off will be just a creditor and not a partner The recharacterization as a partner is devastating for a person b/c their assets are vulnerable. Analysis Question #3 on Page 96:  They really wanted passive control  Did NOT want to be partners  Why didnt PPF enforce?

C. PARTNERSHIP BY ESTOPPEL Young v. Jones (D.S.C. 1992) y Basic facts: o Ps from TX were induced to invest to their detriment with some bank in SC o Ps assert that PW-Bahamas and PW-US operate as a partnership or partners by estoppel  Ps want to get to where the money is by suing PWWorldwide  Court finds that there is no partnership in FACT o Evidence of Partnership in Fact or by Estoppel:  There is a brochure that makes it sound that there is a partnership y Bad fact is that you cant rely on something they didnt know because they didnt find the brochure until after they filed the action  Audit letter was signed just as Price Waterhouse o The court does not find that the 2 PW firms are partners by estoppel 20

y y y y y

y y y y y y y y y y y y

y y

We saw the concept of agency by estoppel Partnership by estoppel is identical to the concept of agency by estoppel. The law imposes a partnership where there is no partnership to protect the 3rd party. The audit letter was based on a falsified financial statement. Need to show that all the PWs are part of the PW Worldwide. Nice thing is that it an agency and if we can show that the PWs are all part of PW Worldwide, then PW Worldwide has to pick up the tab as the principal. All partners have unlimited liability on all partnership debts. Key here: if you are going for PW Worldwide, must assume there was a reason why PW-Bahamas wasnt the one being suede.g., didnt have any money FN on bottom of 106what is the relationship btwn. PW Worldwide and PW-Bahamas. All kept PW in the name. There is value to name recognition as seen in the Holiday Inn case. What helps the argument that there is a partnership here? The brochuretalked about how they were international and had locations everywhere As P, first arg. Is partnership in fact. Voluntary assoc. of two or more associations or entitiesand so on. Then plead in the alternative, if no partnership in fact, there should be a partnership in estoppel. The brochure makes it sound like it is one big happy family They never saw the brochurecant claim you filed a lawsuit based on something you just found The fact that the letterhead had PW on it is probably why got this as far as it did. If it said PWBahamas. It never would have gotten this far. Any evidence of a partnership in fact? Court says no What about estoppel? What is the relevant SC law? S.C. Code (UPA 7.1): Persons who are not partners as to each other are not partners as to third persons Rule: A person who represents himself, or permits another to represent him, to one as a partner in an existing p-ship or with others not actual partners, is liable to any such person to whom such representation is made who has, on the faith of the representation, given credit to the actual or apparent partnership. (UPA 16.1) Under S.C. law, is this partnership by estoppel?  Nofacts do not support a finding of liability for partners by estoppels under the statutory law of S.C. What is the diff. btwn. the S.C. concept of partnership by estoppel and agency by estoppel? You have to not only rely on the partnership, the S.C. requires that it be evidenced by the extension of credit to the would-be partnership. This is specific to S.C., other states allow reliance to be demonstrated in other ways, e.g., giving up another job Law is imposing a partnership b/c of fairness to the TP The piece that is diff. by partnership by estoppel is that many states by statute define how that reliance must be demonstrated. What would be the result of a judicial finding that PW-Bahamas and PW-US are partners by estoppel? PW-Worldwide would be liable to the P Estoppel: Holding out of partnership On audit letter, a member of PW-US signs the letterhead Does not help under SC because there is no extension of credit

SECTION 2. THE FIDUCIARY OBLIGATIONS OF PARTNERS 21

A. Introduction Meinhard v. Salmon (N.Y. 1928) y Referred to as co-venturers instead of as partners; but given the same fiduciary duties as would be assigned to partners y Gerry leased Hotel Bristol to Salmon for term of 20 years Salmon undertook to change hotel building for use as shops/offices for $200K Salmon, in course of treaty with Gerry as to execution of lease, was in course of treaty with Meinhard for necessary funds  Joint venture in writing  M would pay S half of money to reconstruct, alter, manager, operate property  S would pay M 40% of net profits for first five years of lease and 50% for remainder of lease  If there were losses, each party would bear them equally  S would have sole power to manage, lease, operate the building  Co-venturers with same fiduciary duties as partners Heavier duty was on S since he was manager too y When lease was near end, Gerry became owner of reversion He had long term plan to lease property to someone else who would demolish building and replace with something else This fell thru, and Gerry came back to offer new lease to Midpoint, owned by S.  20 years, existing buildings unchanged for 7 years, new building to be placed  S did not tell M about this at all; did not know about anything Later, M found out; made demand on S that lease be held in trust as asset of the venture between the two; judgment to S, at 25% Court:  He should have told M; who would have had a chance to compete for the lease  Breach of fiduciary duty 1) Fiduciary relationship 2) Fiduciary duty follows y When did the original relationship end? At the end of the 20 year lease? When a 20-yr. K ends, does it automatically re-new?  Noif you want to stay, you have to renew. Things dont automatically re-new unless that is how you set it up. Towards the end of 20-yr. term, landlord wants to make a new deal.  However, Meinhard isnt told anything about this new deal. Ms arg is that he should have been consulted.  He assumed that anything that would happen would involve him. He is not legally entitled as a matter of k law: K was silent?  So anything we do for him is going to be an equitable remedy. He says take the joint venture agreement and call it an asset.  Arg is that there was some additional agreement that is not memorialized in this p-a, that this J-V is part of a bigger relationship.  Clearly he thinks he is entitled to it based on the past 20 years. Orif this agreement was a partnership asset, you have to tell your partners everything about your partnership. y What does Cardozo have to say about the termination of the original 20 year lease? Cardozo finds a breach of the DOL: but there was no partnership here! Two or more people, yes. Spitting profits, yes. Carrying on a business for profit no. Meinhard was a passive investor.  In a technical sense, he isnt a partner 22

Cardozo essentially says Meinhard was entitled to have someone come to him and give him notice of what he was doing  Sounds like Cardozo wanted to give Meinhard an opportunity to compete  If this was a partnership, then this would be appropriate This is a very protective opinion. Albert: four cornersif you mean it, put it in the K Cardozo says there is a duty of good faith and disclosure  Duty of loyaltymust put the entitys interest ahead of your own; however, it is unrealistic that people are always going to renounce thoughts of self. Here we have no p-ship, but we have a holding that finds a fiduciary relationship. How could we have contractually protected Meinhard? Put a provision that says we renew automatically unless we say otherwise, or if you choose to enter into any K that affects this property, let me know What if Salmon read about Gerrys interest in the property and approached him with a proposal would he have had any obligation to notify Meinhard or let him in on the deal? Seems worse. Albert things it is failure to disclosure when there is an obligation to disclose. If he went to Gerry he winds up w/ an increased obligation What if instead Salmon read it in the NY Times? He would be less evil now b/c EVERYONE including Meinhard could have read it. We are trying to level the playing field. Meinhard choose to be a background guy. Also would be more problematic if this happened earlier.

The Partnership Interest y Alice is a partner in a three partner law firm who wishes to cash out and retire. Can Alice sell her membership in the partnership to Bob?  Nonot in the way we think of the word sell  As per UPA (1914) 18g, no person can become a member of a partnership without the consent of all the partners  All we know is that Alice wants Bob to be a partner, we dont know about the other partners  An addition of a partner into the partnership terminates the partnership agreement; why should they have unlimited personal liability for Bob? Can Alice sell her 1/3 of partnership assets to Bob?  Partners all own undivided interest in partnership assets Does Alice have any transferable rights in the partnership?  All Alice can do is assign her right to receive partnership profits  UPA (1914) Sections 24, 26, 27(1), UPA (1997) Article 5 B. OPTING OUT OF FIDUCIARY DUTIES Perretta v. Prometheus Dev. Co. Factual & Procedural Background y Limited Partnership Defined: o Liability for firm debts is limited to amount of capital contribution y Defendants: o Prometheus Income Partners (Partnership) was CA limited partnership  Organized to manage 2 apt complexes  General Partner: Prometheus Development Corp (PDC), a CA corporation y 100% owned by DNS trust, controlled by Diller, PDCs sole director, President, and CFO y Plaintiffs: 23

o Louis & Frank Perretta, limited partners in Partnership, suing as representatives of class of limited partners Late 2000: o PDC notified limited partners that it was contemplating merger whereby Partnership would be merged into PIP Partners, an entity owned by DNS trust and Dillers daughter, which owned approx 18.2% of limited partnership units of the partnership o PDC originally offered $1200 per partnership unit, but in March 2002, that changed to $1736/unit  (Diller & daughter offering to buy out all limited partners for cash in transaction in form of merger. Advantage of merger = if merger were approved by sufficient number of limited partners, all limited partners would be required to accept cash for units and Dillers would have no minority owners to contend with) o PDC issued proxy statement to limited partners describing terms of proposed merger  PDC stated that PIP partners would vote neutrally, meaning that PIP will vote its units for or against proposal in same proportion as total number of units voted by unaffiliated partners o Vote: just over 50% of limited partnership units approved Merger  PIP took it over 50%, otherwise it would not have happened

Discussion y A General Partners Duty of Loyalty under CA Law o Under CA law, general partner of limited partnership has same fiduciary duties as partner in any other partnership o Ps claim that Merger was self-dealing transaction which violated PDCs duty of loyalty by setting unfairly low price  Not all self-interested transactions violate this duty; question is NOT whether interested partner is benefited, but whether partnership or other partners are harmed  No breach of fiduciary duty if there is a full/complete disclosure, partner discloses all facts surrounding transaction to other partners, and secures their approval and consent y What constitutes a majority vote? o CA law: majority vote is sufficient to ratify transaction, but in determining whether majority was achieved, all potential votes MUST be included in denominator  Majority not achieved, because unaffiliated votes in favor were only 46% y Manifestly Unreasonable o CA law permits partnership agreement to vary/permit ratifications of violations of duty of loyalty ONLY if provision is not manifestly unreasonable o Ratification is an exception to thorough and relentless scrutiny of self-dealing o CA statutes in supported areas of law support idea that interested partners should NOT be able to count their votes in ratification vote o Allowing interested partner to participate in ratification subverts very purpose of ratification itself: PIP vote was NOT neutral because it failed to account for those who did not return proxy y DOL: fail to put your own interests behind that of the entity y Page 117 Questions: o CA Statute in case (FN 9):  Cannot opt out of 103(b)  Parties can contract for decreases in DOL, but CANNOT obliterate it  103(b) tells us some things pship can do y CA: everything that follows is measured against manifest unreasonableness y UPA provision: there are things you can do, and one subpart measured against manifestly unreasonable 24

y Literally, under terms of statute, we would end up with caselaw o Is there an argument that default rule for general partnerships  What is the difference? y Limited: not responsible for debts beyond their contribution y General Partnership: interested vote should count more or less? C. Grabbing and Leaving Meehan v. Shaughnessy (Mass. 1989) y Why did Meehan and Boyle want to leave? They want to leave b/c they feel they can get a better deal by opening their own firm. They feel that they werent getting enough money; not getting their fair share. The two are certainly allowed to leave, but the firm can limit what they take w/ them. y They speak to some associates at PC about leaving to join the new firm they want to create. y Associates are not members of the partnership so they have no liability for partnership debts. y They approached the clients by going to some of them in advance and asking them if they leave, whether they would follow them. y They rent office space, they have new letterhead and so on. y You can rent office space while in a partnership w/ someone else. y Clearly setting up an enterprise that is in direct competition w/ their current firm. y The fiduciary duty that is triggered is the duty of loyalty b/c setting up shop is potentially taking up opportunities from the current firm. y People start to catch on to this. On 3 separate occasions the partners are asked whether they are leaving and they say no. y They decide to move the plan up and they move forward w/ the leaving. y As they were planning to leave, how did the partners tell their cohorts to carry on? y They say to stretch it out until they are in the new shop. That way the new firm would get the fee. y If you properly take clients w/ you, that wont violate the duty of loyalty. y The stretching starts to sound like a violation. However, they dont end up actually doing this. There was no evidence of inappropriate conduct. y They were told they could take the cases they brought into the firm and also could take cases they were working on w/ them if they paid a fee for them. y The partnership agreement reqd 3 months notice. However, they give less. But one of the partners signs off on it when they say they dont want to give that much. The act of one partner binds the entire partnership. One partner sufficient to bind partnership y UPA Section 20 (1914) vs. 403(c)under section 20, if someone asks you, you have to answer truthfully about anything affecting the partnership; if not asked, then shut your yap y In 1997, in 403(c), partners must cough up info even if they are not asked y Doesnt matter here b/c they asked y Only time this distinction would matter is if you are in a state that doesnt require it and they werent asked y How did PC allocate partnership draw: seniority, eat what you kill, or otherwise? Voluntarily retiring partner, upon payment of a fair charge, could remove any matter in which the partnership had been representing a client who came to the firm through the personal effort or connection of the retiring partner, subject to the rt. of the client to stay w/ the firm.
y

HYPO: y Mark and Norma are partners. Peter comes to Mark at his office with Norma, with a strong case, so Mark decides that is the moment to end his partnership with poor Norma. What are his obligations with respect to Norma? Probably violated his duty of loyalty 25

Change the facts [keeping an eye on whether the changes make it more or less of a partnership opportunity]: (a) Suppose that Peter approaches Mark in the local sports club rather than in the office, but knows that Mark is a partner with Norma;  It is not about the place (b) suppose that Peter approaches Mark in the club, and does not know that Mark is a partner of Norma;  Need to look at what is a partnership opportunity; you knew you were in a partnership; so how do you protect yourself? The fact that the TP doesnt know makes it seem less like a partnership opportunity, but is still one. (c) suppose that Peter approaches Mark and tells him that he hates Norma and does not want her involved;  Still comes to you in your partnership capacity (d) suppose that Peter has not yet approached Mark, but Mark has heard through a third party that Peter probably will approach him soon, and Mark therefore anticipatorily ends his partnership with Norma;  This looks more like a partnership but any steps he takes to get out while he knows of the opportunity are going to be suspect (e) suppose that Mark has just won a high profile case for M&N, that this victory has dramatically boosted his city wide reputation, that Mark can therefore reasonably expect a big increase in the number of cases that will come his way, and that Mark therefore anticipatorily ends his partnership with Norma.  Key is timing; he doesnt know of a specific opportunity here

Basic Partnership Concepts y Remember, UPA is our set of default rules, to govern partnerships where there is no partnership agreement OR the partnership agreement is silent on a particular UPA topic y Statutory review 103(a): UPA sets out the default rules to the extent the partnership agreement does not otherwise provide. The parties can, for the most part, provide whatever terms they want.  Keep that phrase in your mind when you look at any provision of the UPA Exceptions are listed in 103(b), including:(These are things you cannot opt out of)  the partnership agreement cannot restrict the right to access for books and records under 403(b); Why 403(b)? j B/c of the unlimited personal liability on all partnership debts. Would be wildly unfair if you were prohibited from seeing anything. We are protecting people from their own worst instincts.  cant eliminate the duty of loyalty under 404(b), but can curtail it; Think of it like marital property; can agree to waive duty of loyalty claims in certain specific areas.  unreasonably reduce the duty of care under 404(c)eliminate the obligation of good faith and fair dealing under 404(d), etc. 301: sets out agency power of partners to bind the partnership to third parties: each partner is an agent for the partnership for the purpose of its business, and each partners acts for apparently carrying on in the ordinary course of the partnership business binds the partnership, unless the partner had no authority and the third party had notice of this fact.  Overlap here btwn. partnership and agency law 26

A partner has apparent authority and his acts bind the partnership, unless he lacked actual authority, and the TP knows the partner lacks authority. 401: sets out partners rights and duties between and among themselves only- not third parties: Some of the default rules [imposed unless the parties agree otherwise] that we have talked about include: (Note: this is intra-partnership as opposed to 301 which is inter-partnership (btwn. the partnership and the world)  (b) profits are split equally, losses like profits; May have a profits provision and not a losses provision or visa versa or may lack both If only have a profits provision but not losses provision then will split losses like profits If the agreement is silent on profits, will split them equally and will follow what the losses provision says If both provisions are lacking, then will split profits equally and will split losses like profits  (c) partnership must reimburse/indemnify partners for liabilities in the ordinary course of the partnership; Reimbursement is when you are given back the money you spent for something Indemnification is a similar concept; will still make the person whole but you are making the person whole on exposure to liability, e.g., when get sued for good faith performance for agency duties Both reimbursement and indemnification deal w/ money  (f) each partner has equal rights in the management and conduct of the business. Unless provided otherwise, will be one partner, one vote  (j) In the absence of an agreement to the contrary, matters arising in the ordinary course of the business may be decided by a majority of the partners. If there are 6 partners, a simple majority is 4 If there are 10 partners, a simple majority is 6 A super majority is a way to empower the minority; it also makes it more cumbersome In the absence of an agreement to the contrary, a simple majority is going to work Amendments to the partnership agreement and matters outside the ordinary course of the partnership business require unanimous consent of the partners. Why is it fair to require the unanimous consent of all partners to amend the partnership agreement? j B/c you need unanimous consent to create the agreement in the first place Keep in Mind: When you have a partnership w/ 2 partners, the majority rule doesnt help you

Partnership Profits y Profits divided equally (pro rata). UPA (1914) 18(a); UPA (1997) 401(b) y What if one partner contributed 60% of the initial capital? Change in result? Unless you specify otherwise, profits are divided equally The contribution is quantified and the account is crated It is an investment in the partnership and when it dissolves then you get it back However, just b/c you put in more initial capital does not mean you get more of the profits y What if one partner does 60% of the work? Change in result? Unless specified otherwise, no change y What if partnership agreement provided for profits to be divided 90% to Alice; 10% to Bob and for losses to be divided equally. Enforceable? Yes, freedom of K event though it is a kind of a bad deal for Bob Partnership Profits/Losses 27

Partnership agreement provides for profits to be divided: 90% to Alice; 10% to Bob. Agreement silent on losses. How are losses allocated? Will split the losses like the profits: 90:10 Partnership agreement provides for losses to be divided: 90% to Alice; 10% to Bob. Agreement silent on profits. How are profits allocated? Will split the profits equally Partnership agreement is silent on how to split both profits and losses- how do we split? Profits will determine itwill split the profits equally and then split the losses like the profits

Creditors Access to Firm and Personal Assets Creditor of firm seeks to attach personal assets of a partner to collect debt. Allowed? This is allowed A partnership creditor can go after personal assets of a partner for a partnership debt y Personal creditor seeks to attach firm assets. Can the creditor seize partnership property to satisfy the debt? Is the creditor out of luck? A personal creditor can only get the partnership income A partner has an undivided interest in the partnership (doesnt own anything outright) Partnership Management y Abel, Baker & Charlie form a partnership Capital contributions:  Abel 55%  Baker 25%  Charlie 20% Profits track capital y Abel votes to hire secretary; Baker and Charlie vote no Who wins?  As to the secretary, need to determine whether this is in the ordinary course of partnership business; this is definitely ordinary course; since 2 people said no and it is ordinary course of business, secretary is out. (Need a majority to change in ordinary course of partnership) y Baker and Charlie vote to admit new partner; Abel votes no Who wins?  As for the new partner, unanimous consent is reqd. This is outside the ordinary course of business. A new partner would terminate the partnership agreement so the new partner is out. (Amendment requires unanimous consent) y Baker and Charlie vote to increase their draw, contrary to partnership agreement; Abel votes no Who wins?  Ordinary course or amendment to partnership? Harder to decide This is a big enough deal that is probably an amendment and as such, unanimous consent would be reqd.

D. EXPULSION Lawlis v. Kightlinger & Gray (Ind.App. 1990) y Facts: Lawlis became partner in 1972; partnership compensation based on unit system  1982: became alcohol abuser, and did not practice in early 1983 and mid-1984  Sought treatment for alcoholism  Disclosed this to Partnership Finance Committee in July 1983  Partnership meets with doctor and comes up with Program Outline 28

y y y y y y

y y y y

Conditions for Lawliss continuing relationship with partnership It must be set out and clearly understood that there is no second chance, and he signed this document  He then fell back to alcohol, and they gave him a 2nd chance (this is a waiver in K-law) Finance Committee decided he would have to meet specific conditions to continue j Meetings with specialists, obtaining favorable reports from specialists about likelihood of a favorable treatment outcome Told he would return to partner if he met the conditions Lawlis is under the impression that after he is given the 2nd chance and he complies w/ the steps, he will be reinstated as a full partner Eventually he asks for a substantial restoration which would allow him to go back to where he was (more money) before the alcohol problem began They say no (led by Wampler, Finance Committee member) and recommend that he be fired, gave him notice, and they gave him 6 months compensation and insurance to allow him to find a new job in the interim Lawlis argues that the expulsion occurred when Wampler came to his office months before the severance meeting However, the fact that there was a vote at the severance meeting and he got to vote contradicts this. They only proposed to fire him. Then he tries to argue they got rid of him for a predatory as opposed to a bona fide purpose Tried to argue that they wanted to take him out of the mix so there are more profits for everyone else he argues this was predatory Ct. says the partnership gave him a lot of chances ; ct. found the partnership was as kind as they could be This arguable predatory purpose is undercut by the Guillotine Clause they had the necessary 2/3 to kick Lawlis out He also tried to argue firing him violates the fiduciary duty owed to him When you cut someone loose as a partner or agent, the fiduciary duties owed to that person end Rule: Where the remaining partners in a firm deem it necessary to expel a partner under a no cause expulsion clause in a partnership agreement freely negotiated and entered into, the expelling partners act in good faith regardless of motivation if that act does not cause a wrongful withholding of money or property legally due the expelled partner at the time he is expelled Can agree as partners about a list of actions that are not ok; but if you want to use the list against someone, need to make sure their bad conduct fits into the list before you can expel the person However, the clause must be legal, e.g., cant be discriminatory With a no-cause expulsion clause can kick someone out w/o a reason at all; this is what Lawlis had agreed to on day 1 Page 131: What if he had been expelled the first time?  Would have worked out the same way

SECTION 3. PARTNERSHIP PROPERTY Putnam v. Shoaf Facts: y Frog Jump Gin operated for many years (profits and losses both occurred) y Prior to Feb 1976, gin operated at a loss o Originally, equal partnership between Charltons and Putnams  1974: Mr. Putnam died, and Mrs. Putnam, by agreement, succeeded to her husbands interest (she became 50% owner) 29

 When he died, pship died and then it reformed  She did NOT inherit the her husbands interest y 1976: Mrs. P. wanted to sever relationship with other partners o Gin was heavily indebted to Bank of Trenton and Trust Co., and Mrs. P. wanted to be relieved of this liability o Shoafs wanted to obtain her 50% interest in the partnership  Examination of financial records: negative $90K  Shoafs agreed to take over if Mrs. P. and Charltons would each pay $21K into partnership accounts  Shoafs agreed to assume personal liability for all partnership debts o All of this was completed y Known assets of Gin: o Gin, equipment, land (all held in name of partnership) y Mrs. P. conveyed interest to Shoafs by quit claim deed o Details: she gave by quitclaim deed all rights, title, and interest she had in the property  Personal property described o Also agreement between Mrs. P. and Charltons  mutually agreed that the partnership is dissolved. Parties release and discharge each other from any claims and demands on account of, or gowing out of the partnership  Mrs. P is completely released from any and all liability, debts, causes of action of the Gin o Mrs. P. also obtained from Bank of Trenton a complete release from all personal liability to the Bank in consideration of Shoafs assumption of all obligations of the Gin y Old bookkeeper fired and new one hired y 1977: old bookkeepr was found to have engaged in scheme of systematic embezzlement from Gin o Suits filed by Gin against bookkeeper & banks that had honored her forged checks o Mrs. P. was allowed to intervene, claiming interest in any fund paid by banks  Judgment of $68K was paid by banks; of that paid to Charltons as owners of halfinterest; other half is in dispute between Shoafs & Mrs. Ps estate Analysis: y Mrs. Ps ownership interest that she conveyed to Shoafs o Rights in specific partnership property  Possessory right of equal use or possession by partners for partnership purposes o Interest in partnership o Right to participate in management y She intended to transfer her partnership interest to the Shoafs o Being a partner was the last thing she wanted; she got out y Interest in the real property was and always was in the partnership o She did not have any specific interest in specific assets of Gin, to retain or convey. All she had was partners interest in Share of the profits (and losses) y How should this transaction have been effected? o Should have dissolved pship, distributing assets to P and C o P then should have assigned share of assets and liability to Shoafs via quitclaim deed o Shoafs and Cs then should have formed new pship, contributing assets of old one

What is partnership property? y Asset acquired in name of partnership is partnership property o Transfer directly to partnership in its own name o Transfer to 1+ partners acting in their capacity and in name of partnership Partnership Capital 30

Initial capital contribution o UPA silent o Service partnership: one in which one partner contributes only labor y Capital Account o Running balance reflecting each partners ownership o Allocation of profits increases capital account o Allocation of losses decreases capital account Rules for Distribution 40. Rules for Distribution In settling accounts between the partners after dissolution, the following rules shall be observed, subject to any agreement to the contrary: (b) The liabilities of the partnership shall rank in order of payment, as follows: I. Those owing to creditors other than partners, II. Those owing to partners other than for capital and profits, III. Those owing to partners in respect of capital, IV. Those owing to partners in respect of profits. y If pship agreement calls for 90/10 split on profits and says nothing about losses, then default is whatever profits are (90/10) If pship agreement calls for 90/10 split on profits and 50/50 split on losses, then losses is 50/50 because an agreement trumps the default rule. If there is no specification for profits, then it defaults at 50/50. If there is 90/10 for losses its 50/50 for profits. Creditor of firm seeks to attach personal assets of a partner to collect debt. Allowed? y Yes. Hint: UPA (1997) 306: all the partners are liable jointly and severally for all partnership obligations

SECTION 5. THE RIGHTS OF PARTNERS IN MANAGEMENT Deadlock: National Biscuit v. Stroud (N.C. 1959)p. 140-42 y Facts: Stroud and Freeman formed a partnership to run a grocery store. Stroud told National Biscuit that he would not be personally liable for any more bread it sold to the store. Freeman ordered more bread, National Biscuit delivered it, and National Biscuit sued Stroud for payment. y Remember: partners have full personal liability for debts of the partnership. If the partnership is liable on the contract, so is Stroud. y Need to really define the partnership to figure out whether something is w/in that partnership such that it is in the ordinary course of business y Issue: Whether either of them can unilaterally decide that the partnership will not be liable for Ks made w/ a particular supplier. y What was going on here was an attempt to limit actual authority which was an attempt to limit apparent authority when it was told to the TP y What actual authority do these partners have? These partners have the actual authority to buy an sell grocery and to deal with: electricity, food, shelves, cash registers, employees, bags, water, phones, etc. There are things that would not be bread-related that would fall w/in the scope of this business. y Can either of them unilaterally decide that the partnership will not be liable for contracts made with a particular supplier? 31

No-one party cannot attempt to unilaterally change the game w/ regard to deals made in the ordinary course of business The Courts Logic: Under UPA 301(1): every partner is an agent for the partnership for the purpose of its business.  So Freeman and Stroud are agents for the partnership Strouds Food Center, for the purpose of its business, which is to sell groceries. So the supplier has been informed of an invalid limitation on authority It would be different if Freeman actually lacked authority and Stroud told Nabisco Under UPA 301(1), acts of the partners for apparently carrying on in the ordinary course of the partnership business bind the partnership, unless the partner had no authority to act for the partnership on this, the third party involved had notice.  So the partnership is bound on the contract Freeman made with Nabisco, unless he lacked authority and Nabisco had notice. Here, there is no evidence of any limitation on his authority, so we dont need to examine any notice issues- there was no lack of authority to notify Nabisco about. A partner that lacks authority only binds himself Under UPA 306(a), partners are jointly and severally liable for all partnership obligations. Under UPA 401(f), all partners have equal rights of management Under UPA 401(j), in the absence of an agreement to the contrary, matters arising in the ordinary course of the business may be decided by a majority of the partners. Key is that these provisions will apply unless it is provided otherwise What about Strouds comment to Nabisco? Does that affect Freemans ability to bind the partnership? The comment to Nabisco while serving as notice is notice of an invalid limitation on authority. For the statute to work and for the partner to be off the hook, need no actual authority and the TP to know about it What could Stroud have done to protect himself from liability? He could have dissolved the partnership

Deadlock: Summers v. Dooley (Idaho 1971)p. 142-44 y Facts: Summers and Dooley had equal stakes in a trash-collection partnership. Dooley couldnt work, so he hired his own replacement. Summers decided that they needed a third man, but Dooley disagreed. Summers went ahead and hired a third man anyway, and tried to bill the partnership for the cost. Summers contends that in spite of the fact that one of the two partners refused to consent to the hiring of addl help, nonetheless, the non-consenting partner retained profits earned by the labors of the third man and therefore the non-consenting partner should be estopped from denying the need and value of the employee and has by his behavior ratified the act of the other partner who hired the addtl man Held?  Summers loses. Partnership matters are (absent an agreement to the contrary) decided by majority vote, this requires a majority vote, and Summers doesnt control a majority. y Under UPA 301(1): every partner is an agent for the partnership for the purpose of its business. So Summers and Dooley are agents for the partnership, for the purpose of its business, which is a trash collection business. y Under UPA 301(1), acts of the partners for apparently carrying on in the ordinary course of the partnership business bind the partnership, unless the partner had no authority to act for the partnership on this, and the third party involved had notice. Summers could argue that hiring a new worker is within the ordinary course of the partnership business, and so he had authority to act for the partnership, and so the partnership is bound on his contract with the new worker. y Under UPA 401(f), all partners have equal rights of management 32

Under UPA 401(j), in the absence of an agreement to the contrary, matters arising in the ordinary course of the business may be decided by a majority of the partners. Dooley would counter that he refused to consent to the hiring of additional help, and that here, a majority of the partners did not authorize Summers to hire the new worker. Here, unlike Stroud, what is being sought is to broaden what the partnership can do Since a majority of the partners did not consent to the hiring of the new worker, the partnership was not bound by Summers contract with the new worker purportedly on behalf of the partnership.  Summers attempt to bind the partnership is unlawful  The same law is applied to an attempted limitation and an attempted expansion  With unlawful expansion, the partnership will not be bound What about the ratification argument? 1 partner continually voiced objected to the hiring of the 3rd manhe did not sit idly by & acquiesce to the actions of his partner. He allowed the 3rd guy to pick up trash through conduct.  He DID voice objections, and this worked.  Factors in favor of ratification: Time (15 months)

Reconciliation
National Biscuit Summers v. Dooley

Any difference arising as to ordinary matters connected with the partnership business may be decided by a majority of the partners

business differences must be decided by a majority of the partners provided no other agreement between the partners speaks to the issues

Contract binding because: Stroud was not, and could not be, a majority of the partners

Contract not binding because a majority of the partners did not consent to the hiring of the third man

Comparing the Two Cases: y What is the nature of the claim at issue? Who is suing who? In National Biscuit, suit was brought by a third party seeking to hold the partnership liable for the acts of one of the partners. TP v. Pship In Summers, suit was brought by one partner against the other for contribution towards an alleged partnership expense. Pner v. Pner  When you pay more than your fair share of a partnership debt under a partnership agreement, you sue the other partners to get your fair share (lawsuit for contribution) y What about the kind of authority involved? Did the partner in question have the authority to bind the partnership? In National Biscuit, Freeman could bind the partnership to third parties because ordering bread was an act in the usual way of business for his firm under UPA (1914) 9(1), a point that follows straightforwardly from apparent authority principles. The partnership could have restricted that apparent authority, but not without a majority vote and Stroud did not control a majority. So no restriction on authority, and the partnership is bound. In Summers, arguably, Summers never purported to hire the man on behalf of thefirm, so the apparent authority principles of UPA 9(1) never kicked in.  And even if those principles had applied, they would only have governed the partnership's liability to the third party (here, the employee). They would not have governed the allocation of expenses among partners, as per the partnership agreement or the UPA. 33

Reconciliation y One way of looking at this problem is to see it as a conflict between two basic principles of partnership law; i.e., the rule that all partners are agents of the partnership with power to bind the partnership and the rule that all partners have equal rights to participate in the management of the partnership. y As between the partners and some third party, the former principle controls (National Biscuit). As between the partners, the latter controls (Summers). Day v. Sidley & Austin SECTION 6. PARTNERSHIP DISSOLUTION
y

y y y

y y

Expiration of the partnership term 31(1)(a) When a partnership terminates, the partners can agree to terminate the partnership among themselves but they also have to get the message to the rest of the world Act of one or more of the partners in a partnership at will 31(1)(b) At will means it goes on forever; partnerships can also have a term (will terminate at a certain date which is specified and when that date comes and goes, the partnership will be terminated) You always have the ability to get out of a partnership If you terminate early in a term partnership early, there may be damages At will goes on until someone wants out so you can never really terminate it early Agreement of all partners 31(1)(c) Expulsion of a partner 31(1)(d) Subsequent illegality of the partnership purpose 31(3) Why a subsequent illegality?  B/c if it was illegal to begin w/, then you never had a partnership agreement to begin w/ Death of a partner terminates by operation of law 31(4) Bankruptcy of any partner or the partnership 31(5) Bankruptcy acts as dissolution:  Think of the fact that all the partners have personal liability for this partners debts Court decree 31(6) [incompetence; incapacity; misconduct; impracticality; other equitable circumstances].

Court Decree:  When you are seeking a dissolution of partnership by a ct. decree, it means you are out of options. Will need to go to ct. and plead equity. Most typical group pleaded is incompetence (unfair for the others to have personal liability for decisions made by the person who is now incompetent).  Your best case scenario for getting rid of a partnership is termination by agreement Impracticality:  Means the partnership can only be operated at a loss. Partnership is losing money and will never turn a profit. Will only need to do this where one of the partners refuses to accept this fact. Need to demonstrate the partnership cant recover. Dissolution v. Going out of Business Dissolution is not the same as going out of business: A dissolution is simply the change in relationship of the partners caused by any partner ceasing to be associated in the carrying on of the firms business. UPA (1914) 29. Its a legal status nothing more 34

y y

y y y y y y

Dissolution is a legal conclusion. When it is dissolved, the partnership ceases to exist. Will still need to wind up and turn the remaining stuff into money. Will sell the inventory and all such items. Need to liquidate The first people to get paid are the outside creditors (the non-partner creditors). The next group are the inside creditors (partners who have lent money to the partnership). They would do this b/c this is a way to invest in a business. Then will return the capital that was contributed. Hope the pile of money is bigger than the line. If the line is bigger, then the unlimited personal liability comes in. Weirdly enough they might be paying themselves back (e.g., capital contributed). Person who does the winding up gets a little more money for doing so

Dissolution: Effect on Partnership y After dissolution, the partnership must be wound up, absent agreement among the partners to carry on the business. Assuming that the business will not be continued, the winding up process generally contemplates that the firms assets will be distributed to the partners. y Winding up: The process of shutting down post-dissolution y Upon dissolution, the authority of partners to act on behalf of partnership is terminated except in connection with winding up of partnership business. UPA (1914) 33; UPA (1997) 804. Continuation per Agreement: Effect on Partnership When the parties decide to continue the partnership under 802: Technically creates a new partnership Creditors of former partnership automatically become creditors of new partnership. UPA (1914) 41. Called reconstituting Continuation per Agreement: Effect on Departing Partner Departing partner entitled to accounting Fair value of partnership Interest from date of dissolution in event of unreasonable failure to pay y Departing partner remains liable on all firm obligations incurred while he was a partnerUPA (1914) 36; UPA (1997) 703. y For future partnership debts, departing partner must provide appropriate notice to TPs to cut off his personal liability actual for existing creditors, constructive for all other TPs y Either kind of debt can be released with a novation. Regarding release, creditor might say that my name is not on that release; these were debts incurred while you were a partnership Only way this partner will get out of this is if the agreement is btwn. the remaining partners minus this partner and the creditor and the outgoing creditor. Everyone needs to agree that the outgoing partner has no responsibility. Absent a novation (agreement btwn. all 3 parties), that withdrawing partner has liability on anything that happens before they leave Regarding the release, while not binding on the creditor, the leaving partner can sue his/her former partners Will use constructive notice, for new creditors that werent creditors of the partnership when the partner leaves When you have a novation w/ the creditor and that creditor lends money again, that novation acts as actual notice to that creditor Prewithdrawl debts: 35
y

  

Unlimited liability Agreement Novation for each creditor

Continuation per Agreement: Effect on New Partners If a new partner joins the firm when it continues after a dissolution, the new partner is also liable for the firms old debts, but such liability can only be satisfied out of partnership property. UPA (1914) 41(1); UPA (1997) 306(B). Limit the liability of incoming partners to their capital contributionthat is what the UPA is saying y The new partner cannot be held personally liable for the old debts, unless he or she expressly agrees to be so held. y Withdrawing partner MUST give notice in order to escape liability Actual notice to each creditor Term Partnerships y Explicit term Duration specified in partnership agreement Specific purpose/object specified in partnership agreement y Implicit term Example: When a partner loans money to the firm, to be repaid from profits, the partnership is for a term until the loan is repaid Implicit term can be read into the partnership; e.g., when a partner lends money to a partnership and loan will be paid out of profits, ct. will read in that the term has to be at least as long as it takes to pay back that loan

Owen v. Cohen (Cal. 1941)p. 150-53 y Facts: Owen and Cohen became partners in a bowling alley through oral agreement (the fact that it is an oral agreement is irrelevant) Owen put up $6,986, which was to be repaid out of the profits of the business; treated as a loan (not a contribution). Cohen is a pain to work withor would be, if he did any work y Owen sues for dissolution Why?  They dont think they have a partnership for a term, they think it is one at will y Trial ct. puts in a receiver (a neutral party that comes in to figure out what the dollars and cents are)means the judge doesnt trust the parties y Why does it matter when the loan gets repaid? Only matters who is 1st or 2nd when there isnt enough money to pay both Key is that if will pay back out of the profits of sale and then split the partners; the evil partner will get a lot less than if the loan gets set aside Wants the partnership to continue to pay loan out of profits y Ct. felt Cohens misconduct was sufficient to order dissolution of the partnership. y Why was a ct. order sought? It is in Owens best interests to allow a ct. to step in to dissolve the partnership Keep in mind that it is possible to have a partnership and not know it and also to have a partnership for a term and not know it. Needed a ct. to declare it so he would not be in breach. Judicial determination of status of loan required Receiver probably needed for orderly liquidation Potential for wrongful dissolution  UPA (1914) 31(1)(b) v. 31(2) 36

Dissolution of a term partnership (a.k.a. partnership for a term) prior to expiration of the term is wrongful  Adverse consequences; see UPA (1914) 38(c) Hence, there always exists the power, as opposed to the right, of dissolution Holding: When a partner advances a sum of money to a partnership with the understanding that the amount contributed was to be a loan to the partnership and was to be repaid as soon as feasible from the prospective profits of the business, the partnership is for the term reasonable required to repay the loan.

HYPO: y Albert, Bette and Carol each own 1/3 of the Roy Lumber Company, a general partnership; as per their partnership agreement, they share profits and losses equally, and all purchases over $500 have to be authorized in advance by two partners, and only Albert can draw checks. Bette, without permission and on the firms account, buys a $2500 bracelet, $5000 forklift and $2000 of logs from Doug who runs a store with an eclectic inventory. Before Bette made these purchases, Albert told Doug that Bette was not authorized to buy logs. USE IRAC Notice that we are being told it is a general partnership What is missing is what UPA is used in the state we are in Default on profits and losses There is al imitation on actual authority (purchases over $500 must be authorized); only binding on TPs that have notice Critical that Bette has no permission (she is not seeking personal liability) y Doug comes to Roy Lumber to talk about the outstanding balances owed to him. And Albert wont pay for Bettes purchases. Doug calls Albert a bad name, and Albert punches him in the nose. While Doug is lying on the ground, an employee of Roy Lumber negligently drops a log on Dougs leg, breaking it. What are Dougs rights with respect to (1) the logs; (2) the forklift; (3) the bracelet; (4) the broken nose; and (5) the broken leg. Possible people to pay are Bette, the partnership, or no one (1) Was Bette allowed to buy logs? She did not have actual authority to buy logs b/c purchases over $500 must be authorized. This comes from the partnership agreement where actual authority is contained. Conclusion: partnership has no liability on the logs b/c Bette exceed her authority. Rule: need to discuss actual authority and apparent authority. In this case, not in partnership agreement. If no actual authority then no apparent authority. Also there was notice to Doug that Bette was not authorized to buy logs. Thus Bette is lacking both actual authority and apparent authority b/c have a limitation on actual authority and notice to TP. Only person who can be liable to Bette herself. If she had actual authority, then there would have been apparent authority to buy the logs b/c it would be ordinary course of business to buy the logs. (2) Why is the forklift different? Difference is that he never called about the forklift so he wouldnt know about the lack of actual authority so the apparent authority was still there. A forklift is in the ordinary course of business. Since there was no notice, the limitation on authority was not binding on the TP who had no notice. Thus, the partnership has liability on the forklift purchase b/c of AA. (3) The difference here is that this is clearly outside the scope of a lumber company. She has no actual authority to bind the bracelet b/c as per the partnership agreement she needs authorization for purchases over $500. In terms of apparent authority, he did not have to call on this one b/c a diamond bracelet is outside the scope of a lumber business. Thus, Bette has liability on the bracelet. (4) This was broken by Alberts first. This is an intentional tort. No master-servant relationship here. Also different b/c this is outside the scope of the business b/c punching someone in the nose is outside the scope of running a lumber co. No vicarious liability b/c that third element is not present. The second you do an intentional act, you pull yourself outside the scope. 37

(5) This was broken by a negligent act of an employee. The employee is presumably a servant of Roy Lumber Co. This is an accident. He is clocked in and it sounds like carrying logs from place to place in the lumber co. is w/in the scope of this business. Vicarious liability. Page v. Page page 158-161 y P and D are partners in linen supply business y P appeals from judgment declaring partnership to be for term rather than at will y Facts: o Oral partnership agreement; within 1st two years each partner contributed about 43K for purchase of land, machinery, linen to start business o First 8 years = unprofitable (lost 62K) o Major creditor = corporation wholly owned by P that supplies linen and machinery necessary for day-to-day operation of business o Partnership improved and became profitable, but P still wants to terminate y Uniform Partnership Act: o Partnership may be dissolved by express will of any partner when no definite term or particular undertaking is specified o Trial Court:  Partnership is for term (reasonable time to enable partnership to repay from partnership the indebtedness) o D testified that terms of partnership were to be similar to former partnerships of P and D, and that understanding was that they would start business, and let business pay for itself  Former partnership which earnings were to be retained until obligations were repaid was different from partnership  All obligations were to be repaid from profits, according to previous relationships  Concedes that there was no understanding as to the term of present partnership in event of losses  Claims that partners expected to meet current expenses from current income and to recoup investment if business were successful, and that this was sufficient to create partnership under rule from Owen y In that case, partner advances a sum of money to a partnership with understanding that amt contributed is loan to partnership, the partnership is for the term reasonably required to repay the loan y Ruling: o D failed to prove any facts from which agreement to continue the partnership for term may be implied  Hope that partnership would pay for expenses does not establish a definite or particular term required under Corporations Code o Partner at will is not bound to remain in partnership, regardless of whether business is profitable or unprofitable o P has power to dissolve partnership by express notice to D  If it is proved that P acted in bad faith and violated fiduciary duties by attempting to appropriate to his own use the new prosperity of the partnership without adequate compensation, dissolution would be wrongful Consequences of Dissolution: Prentiss v. Sheffel Overview: y Whether 2 majority partners in 3-man partnership at will who have excluded third partner from pship mgmt and affairs should be allowed to purchase partnership assets at judicially supervised dissolution sale 38

y Purchase is proper Facts: y Partnership created to acquire and operate a shopping center in Phoenix y Ps grounds for dissolution: o D had been derelict in pship duties, and he failed to contribute balance of his proportionate share of operating losses incurred by Center o Sought trial courts permission to continue pship during pendency of suit and thereafter y D filed counterclaim seeking winding up of pship and appointment of receiver. o He had bee wrongfully excluded from pship and rights as partner were violated o Trial Court Findings  Each P owned 42% interest, while D was owner of 15%  No detailed pship agreement  Many unresolved disputes between the parties (title to property, and how mgmt decisions would be made)  As a result of disputes, relationship deteriorated and Ps notified D that any further dealings would go thru attorney  D never was denied physical access to Center  D had not made payments of all of his share of deficits  Centers losses had been materially reduced; no showing of waste as result of mgmt  Freeze-out of D from pship mgmt and affairs o Trial court concluded that pship-at-will existed between Ps and D which was dissolved as result of freeze-out of D from mgmt and affairs of pship o Receiver was appointed by court until pship property could be sold  Refused Ds request that order be entered forbidding Ps from bidding at contemplated judicial sale o Receiver and trial court proceeded with liquidation and sale of Center  Ps were high bidders  Court entered order confirming sale of Center to them o D appeals y Ruling/Rationale: o D: I was wrongfully excluded from mgmt of pship and therefore Ps should not be able to purchase pship assets at judicial sale because he would be disadvantaged  Record is contrary: he WAS excluded, but not for Ps to obtain pship assets in bad faith excluded because of inability of partners to function in relationship  D has failed to demonstrate how he was injured by participation of Ps in sale y Absent Ps bid, the highest bid would have been much lower y Ds interest was enhanced by Ps participation! y D was NOT forced to sell his interest to Ps o He had same right to buy assets as they did, by submitting highest bid The Sharing of Losses: Kovacik v. Reed Facts: o K told R that he had a chance to remodel kitchens, and he asked R to become his job super and estimator o K had 10K to invest, and if R would be super and estimator, he would share profits on a 50-50 basis o K did not ask R to share losses, and R did not offer (did not even discuss losses) o On all jobs K provided money and R did not provide money o K told R that venture lost money, and demanded money from R for losses  R denied, and said he never agreed to be liable for losses o K brought suit, seeking to recover of all losses from R 39

o Trial Court: o P and D were to share equally all joint venture P&L and D agreed to share equally in P&L of venture o Judgment awarded for K Ruling/Rationale: o General Rule: in absence of agreement, law presumes partners intend to participate equally in profits and losses of common enterprise, irrespective of any inequality in amt each contributed to capital employed in venture o However, in this case one partner contributes money and the other contributes skill and labor, then neither party is liable to the other for contrib. for any loss sustained o Upon loss of money, the party who contributed it is not entitled to recover any part of it from the party who contributed only services  In event of a loss, each party loses his own capital (money in the case of K, and his labor in the case of R). Buyout Agreements I. Trigger events a. Death b. Disability c. Will of any partner II. Obligation to buy versus option a. Firm b. Other investors c. Consequences of refusal to buy i. If there is an obligation ii. If there is no obligation III. Price a. Book value b. Appraisal c. Formula d. Set price each year e. Relation to duration IV. Method of payment a. Cash b. Installments (with interest?) V. Protection against debts of pship VI. Procedure for offering either to buy or sell a. First mover sets price to buy/sell b. First mover forces others to set price SECTION 7: LIMITED PARTNERSHIPS Holzman v. De Escamilla Facts: o Hacienda farms, Ltd. Was organized as limited partnership o Escamilla is general partner; Russel & Andrews = limited partners o Bankruptcy in 1943; Holzman was appointed as trustee of estate  Brought action to say that R and A were general partners and therefore liable to creditors of pship because they had taken part in control of pship business o Trial court: 3 Ds were liable as GPs o Escamilla raised beans on farmlands o R&A conferred and agreed to which crops would be planted 40

o A agreed to plant watermelons o All 3 determined that stringbeans would be planted o R&A came to farm 2x a week to consult about which crops to plant  Did not want peppers or eggplant o A&R requested that E resign as manager, and Miller was put in instead o Two bank accounts for Hacienda Farms  One provided that checks could be drawn on signatures of any 2 of 3 partners  Money withdrawn on 20 checks singed by R&A, all other checks except for 3 had Es signature with one of Ds o Civil Code Section 2483:  Limited partner becomes liable as GP if he takes part in control of business o They did control the business o Albert: DO NOT LET LIMITED PARTNERS HAVE ABILITY TO WRITE CHECKS! Hypo: y Jim, Kassie, Mim each own 1/3 of all Port Washington Woodwork, a general partnership y They share profits and losses equally y All purchases over $500 have to be authorized in advance by 2 partners o Limiting actual authority y IRAC for each thing y The planks: o Kassie bought planks o Did she have actual authority? They were $2000. o Can she write a check binding the partnership?  No  Jim is the only one who can draw checks  She needs 2 parties to agree any time there is a purchase over $500 o Limitation on apparent authority o Limitations on third parties are only binding on third parties who have notice o Kassie is liable y Cindy has to sue Kassie, she cannot sue the partnership. What makes that rule is the definition of actual authority, the definition of apparent authority, and that limitations on apparent authority are only binding on third parties that have notice. (Cite to the UPA provision). o In this case, Kassie lacks actual authority to buy a $2000 by check because agreement says Jim can only write checks. She lacks apparent authority because she told Cindy about the limitation. y If you were Cindy, would you think that someone who works for this company has apparent authority to buy the forklift. o Jim did not communicate this limitation to Cindy o Pship is BOUND on forklift  Pship can sue Cassie herself y She buys the bracelet o Exceeds $500 o Do not need to let Cindy know this is so far outside the scope of a contracting business that it is NOT reasonable for Cindy to believe that she has apparent authority. o Kassie is liable! y Black eye and broken leg? o Can she sue partnership for black eye or wooden plank o Possibility of vicarious liability o Respondeat superior 41

CHAPTER 3: THE NATURE OF THE CORPORATION SECTION 1. PROMOTERS AND THE CORPORATE ENTITY Categories of corporations 1) Public (aka publicly held) a. Characterized by a public secondary market in which shares of the company are listed for traded b. Examples: IBM, Microsoft, Etc. 2) Close (aka closely held, private) a. Characterized by absence of a secondary market for its stock i. No public market for the shares b. Often (but not always) a relatively small number of shareholders who actively participate in the firms management c. May display many characteristics of partnerships i. Some are, in a sense, incorporated partnerships MBCA o What is the MBCA? o Model Business Corporations Act o Put together by lawyers and practitioners o Statement of how law ought to be in a perfect world o Where is it binding law? o No, it has not been enacted anywhere o Can be adopted by states o On exam, must tell prof. that the MBCA is not the law unless or until it is enacted 5 critical attributes of a corporation: 1) Separate legal entity: a. Separate from whom? i. The owners 2) Limited liability: a. For whom? i. The shareholders on corporate debts ii. Liability for debts capped at their contribution iii. This is the difference from partnership: corporations have limited liability iv. MBCA 6.22(b): Unless otherwise provided in the articles of incorporation, a shareholder of a corporation is not personally liable for the acts or debts of the corporation except that he may become personally liable by reason of his own acts or conduct. 3) Separation of ownership and control: a. MBCA 8.01(b): All corporate powers shall be exercised by or under the authority of, and the business and affairs of the corporation managed by or under the direction of, its board of directors. b. Bifurcated: Board of Directors who run enterprise, and shareholders who own the enterprise i. Shareholders elect the BOD c. Rule of Thumb: Boards act, shareholders react d. Shareholders entitled to vote on: i. Election of directors (MBCA 8.03-.04) 1. Most important thing that shareholders do ii. Any amendments to the articles of incorporation and, generally speaking, by-laws 42

4)

5)

(MBCA 10.03, 10.20) iii. Fundamental transactions (e.g., mergers; MBCA 11.04) iv. Odds and ends, such as approval of independent auditors Liquidity a. Assets that can be turned into cash b. When assets are liquid, they have a nice cash value c. Publically traded stock has much more liquidity than private stock (pick up a newspaper to figure out the price) d. Much easier than partnership: stock is freely sellable Flexible capital structure: a. The permanent and long-term contingent claims on the corporations assets and future earnings issued pursuant to formal contractual instruments called securities i. Many ways to package such claims; e.g., stocks and bonds

Difference between debt and equity securities: y Debt securities (a.k.a. bonds, debentures, notes) typically consist of two distinct rights: o The bondholder is entitled to receive a stream of payments in the form of interest over a period of time o At the end of the bonds prescribed term (i.e., at maturity), the bondholder is entitled to the return of the principal o Bondholders are creditors, not owners y Equity securities (a.k.a. shares) represent the units into which the proprietary interests in the corporation are divided o Residual claimants: equal right to participate in distributions of the firms earnings and, in the event of liquidation, to share equally in the firms assets remaining after all prior claims have been satisfied o A limited right to participate in corporate decision-making by electing directors and voting on major corporate decisions Capital structure terminology: y Authorized shares: The articles must specify the maximum number of shares the corporation is authorized to issue. y Issued shares: sold to shareholders y Outstanding shares: The number of shares the corporation has sold/issued and not repurchased. o Authorized but unissued shares are shares that are authorized by the charter but which have not been sold by the firm. y Treasury shares were once issued and outstanding, but have been repurchased by the corporation o Issued shares outstanding shares = Treasury shares y EXAMPLE: o If the charter authorizes the firm to issue up to 10,000 common shares. Suppose the firm sells 2,000 shares to investors. How would you characterize the corporations 10,000 shares?  2000 are authorized, issued, outstanding  8000 are authorized, not issued, and not outstanding. They are available to be issued o Now suppose the corporation bought back 500 of its outstanding shares. How would you characterize the corporations 10,000 shares?  The 8000 are authorized, not issued, and not outstanding  1500 are authorized, issued, and outstanding  500 that were brought back are treasury shares o What if the corporation in our example is incorporated in a state that has adopted the MBCA?  MBCA doesnt recognize treasury shares 43

y y

They would put the 500 that you buy back with the 8000. So you would have 8500 that are authorized, not issued, and not outstanding 1500 would be authorized, issued, and outstanding

Issuance of stock: y Board of directors prerogative. o Shareholders involved only if:  Board wants to sell more shares than are presently authorized in its charter charter amendment requires shareholder approval  Board of directors wants to issue a new class of shares not authorized in the charter o If the charter authorizes the class of shares in question and there are sufficient authorized but unissued shares, the board is free to sell shares for any valid purpose as long as the corporation receives adequate consideration for the shares. y Choosing a state of incorporation o Delawares dominance  No minimum capital requirements  The need for only one incorporator (a corporation may be the incorporator)  Favorable franchise tax in comparison to other states.  For companies doing business outside of Delaware: y no corporation income tax y no sales tax, personal property tax or intangible property tax on corporations y no taxation upon shares of stock held by non-residents and no inheritance tax upon non-resident holders  A corporation may keep all of its books and records outside of Delaware and may have a principal place of business/address outside of the state of Delaware as well  Highly competent judiciary in company law and extensive and detailed case law on this subject The incorporation process Process of incorporation: o File articles with SOS under MBCA 2.03, that starts the corporations life o Post incorporation: o Draft bylaws (MBCA 2.06) o Organizational meeting (MBCA 2.05) o Issue stock Promoters and pre-incorporation liability: o Promoter: Someone who purports to act as an agent of the business prior to its incorporation o Promoter retains liability on pre-incorporation contracts unless and until promoter gets a novation o Pre-incorporation contracts o Possible outcomes for pre-incorporation contracts signed by a promoter: o Promoter retains liability, corporation has none o Promoter and corporation both have liability o Promoter has no liability, corporation has liability Review 1. Once the articles are filed, does the corporation become liable on/a party to the contract? y Corporation only becomes liable on pre-incorporation K if it chooses to. The articles being filed makes no difference. 2. If the articles are not filed, is the promoter liable on the contract? y The promoter is the only one bound 44

3. 4. 5.

If the promoter forms the corporation later, can the corporation become a party to the contract? If the promoter later forms the corporation, can he avoid liability? y Novation If the articles are not filed or are defectively filed, can the defectively formed entity (or individuals) enforce the contract?

De jure v. de facto corporations o De jure corporation: a corporation in good standing under the law o De facto corporation: acting like a corporation while not fulfilling legal requirements. o Focuses on the efforts of the defendant: o Treat firm as a corporation if the organizers:  in good Faith tried to incorporate  had a legal right to do so [the existence of a statute allowing incorporation of the venture]  some action as a corporation o example: you ask your lawyer to organize a corporation to buy and sell real estate. You then decide that you need a computer, and buy one from IBM on credit in the name of the corporation. IBM believes it is selling to the corporation. Your business then fails, IBM sues for the purchase price, and you discover for the first time that your lawyer never sent the corporate papers to the secretary of state. o Fine distinction to be drawn between de facto corporation  [often used in place of "actual" to show that the court will treat as a fact authority being exercised or an entity acting as if it had authority, even though the legal requirements have not been met] o andcorporation by estoppel  Treat firm as though it were a corporation if the [TP plaintiff] person dealing with the firm: (1) thought it was a corporation all along and (2) would earn a windfall if now allowed to argue that the firm was not a corporation o This doctrine has nothing to do with the defendant making a good try. It comes from the injustice created when you try to gain an advantage by changing your story as to the existence of a fact or legal relationship. Southern-Gulf Marine Co., No. 9, Inc. v. Camcraft, Inc Facts: Involves a boat building K. When the agreement is signed, Southern Gulf is not yet incorporated. Thus, it looks like a pre-corporation K. The Pres. of Camcraft acts w/in the scope of authority to bind his co. He is not binding himself in a personal sense. Barret, president of Southern Gulft signs as himself (personally) and is also signing in his corporate capacity as an agent (which is of course, imaginary since there is not yet a corporation that can be bound). Southern Gulf gets incorporated in the Cayman Islands. K said a TX Corp., but it turns out to be a Cayman Islands corp. Camcraft seems to waive thisfine w/ them that it is not TX. All goes wrong b/c they didnt build the boat they promised. Reason was b/c the boat building industry got more expensive and it turned out to not be a good deal for the boat co. They must have felt that it was better to default and go through litigation than to build this boat. Southern Gulf wants specific performance. Claim of breach of K.  Parties to the K were Camcraft and Barrett (b/c when Barrett signed the corporation did not yet exist)  So we dont have a K btwn. two corporations. 45

Attempt to argue that there is not really a K at all since the corp. did not exist yet; thus, cant be a breach of K.

Holding: Camcraft is estopped to deny SGMs corporate status Hence, SGM may sue to enforce the contract Rule of law: A third party who dealt with the firm as though it were a corporation and relied on the firm, not the individual defendant, for performance is estopped When does estoppel apply? 1. After a defective attempt to incorporate, the participants in the venture themselves cite their defective incorporation in an effort to escape liability on the contract with plaintiff; the court will estop them from asserting the corporation did not exist and therefore the contract is not enforceable when the contract was made supposedly on their behalf. a. This would apply in SGM if it were SGM who was trying to avoid liability on the contract. 2. The more difficult application of the concept is to our actual SGM facts. The participants in the venture [SGM] ask the court to estop a third party they have dealt with from denying the existence of the corporation. Seems pretty straight forwards at first glance. a. The TP agreed to deal with a corporation, and is trying to weasel out based on facts that really don't impact it- they are just looking for an excuse to avoid liability. If they were permitted to deny the existence of the corporation on these facts, they would get more than they bargained for - a windfall. Reasoning in Southern-Gulf Marine: On the estoppel claim the court finds that in order to weasel out of a contract due to the character of the organization to which it is obligated the defendant would need to show its substantial rights might thereby be affected, which it was unable to do. SGM is not a de jure Texas corporation as stated in the contract, but that fact hasnt caused Camcraft any substantive problems and Bowman was informed of the Cayman Islands incorporation and accepted it. So they are estopped from asserting the plaintiffs lack of corporate capacity when the Vessel Construction Agreement was signed. Enterprise Liability y Ex: if I set up 10 different corporations, but I am sole shareholder. All 10 have same garage, same bank account, same woman who does bookkeeping, etc. y PCV: going thru one corporation to the shareholder o Going to other corporations o Blowing walls off corporation o Blurring distinction between owner and corp. o Taking away limited liability of a shareholder y Enterprise Liability: o Blurring distinction between other corporations Walksovszky v. Carlton Facts: o Complaint: o P was injured in NYC when rundown by cab owned by D Seon Cab, negligently operated by Marchese o Carlton, stockholder of 10 corporations, including Seon, each of which has 2 cabs registered in its name o Only minimum auto liability insurance carried by each cab o Corporations, seemingly independent, are operated as single entity with regard to: 46

 Financing  Supplies  Repairs  Employees o P claims that he is also entitled to hold stockholders personally liable for damages because multiple corporate structure constitutes unlawful attempt to defraud members of general public who might be injured o Defendant: o Moved pursuant to CPRL 3211(a)7 to dismiss complaint on ground that to him it fails to state cause of action  Special Term granted motion but Appellate Division reversed  D Carlton appealed Issue: Can separately incorporated corporations. . . Ruling/Rationale/Analysis: o Law allows incorporation of business for purpose of enabling proprietors to avoid personal liability; however, there are limits o Courts will pierce corporate veil whenever necessary to prevent fraud or achieve equity o Whenever someone uses corporation to further his own interests, he will liable for corporations acts under respondeat superior o This case: o P explicitly alleged that none of the corporations had a separate existence of their own o No allegations that D was conducting business in his individual capacity Limited Liability MBCA 6.22(b): a shareholder of a corporation is not personally liable for the acts or debts of the corporation except that he may become personally liable by reason of his own acts or conduct Limited Liability MBCA 6.22(b): a shareholder of a corporation is not personally liable for the acts or debts of the corporationexcept that he may become personally liable by reason of his own acts or conduct Limited Liability MBCA 6.22(b): a shareholder of a corporation is not personally liable for the acts or debts of the corporation except that he may become personally liable by reason of his own acts or conduct Tips to avoid Piercing Corporate Veil: 1. Avoid unity of interest respect the separate existence of the corporation, and your clients probably wont need to be concerned about personal liability. 2. Hold your statutorily required meetings, keep minutes, elect officers, etc., but, most important, keep corporate funds and transactions separate from individual funds and transactions and not constantly shift money in and out of the corporate account. Substantial compliance is probably enough. So what do we need to PCV? (1) a total disregard for the separate existence of the corporation (2) Most of the cases and the commentators state that there is a second element that must be proved: injustice or some such concept. Do all courts require that second element? No; for some courts, however, a total disregard for the separate existence of the corporation is enough, on the theory that if the shareholder ignored the separate existence it would be unjust for the creditor to be required to respect it. What proof is required? 1. In terms of the injustice: 47

a. Case holdings inconsistent: i. Clear that creditor will not be paid UNLESS you pierce the corporate veil 1. Thus, the creditor will experience injustice b/c they cannot get anything from a company that has no assets a. This is too easy, so not enough b. Need something more as we will see Does the plaintiff have to be aware of the defendants disregard of the corporate formalities? 1. As a matter of common sense, seems unlikely that someone would, e.g., ask to see corporate books and records 2. In a K situation, it is much more volitional Is there a difference between tort and contract creditors and their need for corporate formalities? 1. When you are a tort creditor, it is due to something that is not by choice a. Much less voluntary structure b. Wont be as concerned in tort creditor situation about corporate formalities c. May not know about unity of interest elements because of this d. Want K creditors to be more mindful of that CERCLA Under CERCLA, a parent corporation may be liable under traditional piercing the corporate veil principles. It may also be liable for its direct participation in operating a hazardous waste site, where its agents directly operate, or participate in operating, the facility on its behalf. Enterprise liability All 10 corporations were alleged to be part of a single enterprise that actually conducts the business, so plaintiff ought to be able to recover from any or all of the 10 in the enterprise. What does enterprise liability depend on? It depends on proof that Carlton did not respect the separate identities of the corporationsfor example, here are some factors to consider in determining whether two or more corporations have been operated as a single business enterprise include whether the corporations had: (1) common employees; (2) common record keeping; (3) centralized accounting; (4) payment of wages by one corporation to another corporations employees; (5) a common business name; (6) services rendered by the employees of one corporation on behalf of another; (7) undocumented transfers between corporations; (8) unclear allocation of profits and losses between the corporations; (9) the same officers; (10) the same shareholders; and (11) the same telephone number. Generic Questions Is it improper to incorporate your business for the express purpose of avoiding personal liability? o No; that is why we incorporate entities for express purpose of avoiding personal liability Is it improper to split a single business enterprise into multiple corporations so as to limit the liability exposure of each part of the business? [recall Southern-Gulf Marine] o No, nothing illegal is done The Formalities Issue 48

Many of these cases place great emphasis on factors like failure to comply with corporate formalities. Why are such factors relevant? o Probably comes from idea that if you are not going to honor separate existence, it is unfair to the other people Is it enough that the creditor will be unable to collect the full amount owed unless the court pierces the veil? o That will not be enough o If it were true, every creditor who bumps up against corporate veil and cant get assets will claim injustice Sea-Land Services, Inc. v. Pepper Source (7th Cir. 1991)p. 194-200 y PCV in Illinois Sea Land Facts:  Sea-Land shipped peppers for Pepper Source. Pepper Source didnt pay for the freight bill. In 1987, there was a default judgment issued against Pepper Source. But Pepper Source has been dissolved and had no assets. Thus, they are judgment-proof. So P really hasnt won much, yet. In 1988, P brought suit against (1) Marchese, owner of Pepper Source, (2) 5 businesses owned by Marchese, and (3) 1 business owned by Marchese w/ another person.  Thus, the only way Sea-Land could get any money for the default was to pierce the corporate veil. Under Illinois law [the Van Dorn test], to pierce the corporate veil, a court must find (a) a unity of interest and ownership between corporation and shareholder, AND (b) a situation where failing to PCV would either (i) sanction fraud [dont need proof of intent to defraud creditors]; or (ii) promote injustice. Holding: The first part of the test is met, but the second part has not yet been met for purposes of summary judgment. An unsatisfied judgment alone is not enough to establish an injustice. There needs to be something more. Remanded for a determination. y Van Dorn Test- Illinois law To determine whether (a) exists, courts look at four factors: (w) the lack of corporate formalities, (x) the commingling of funds and assets, (y) under-capitalization, and (z) the use by one corporation of assets of another [which sounds to me a lot like commingling] Reasoning: Under IL law, unity of interest isnt enoughyou need fraud! Do we have unity of interest? Yes. What facts support that? The corporation never had any corporate formalities, no filing of articles of incorporation, there were no bylaws, they used corporate funds for personal expenses, they moved funds back and forth, and it is undercapitalized. Thus, they are able to satisfy the first prong by showing these factors. However, the court is looking for something beyond the creditors inability to collect. An unsatisfied judgment is not enough. They are looking for something like fraud. Something like an intent to hide assets, etc. The court remanded because it couldnt find the injustice. y How do you show that failing to PCV would promote injustice? Representation of ability to pay Unjust enrichment y What kinds of behavior would satisfy the court on this prong? Essentially a deliberate attempt to avoid the creditors y And in this case? On Remand: the court relied on the fact that Marchese engaged in blatant tax fraud by treating his personal expenses as deductible corporate business expenses, and had used corporate funds for his own benefit while avoiding corporate debts. The court found that Marchese had assured a Sea-Land rep in a phone conversation that the freight bill would be paid, even though he knew that he would manipulate the corporate funds to insure there would not be funds to pay the Sea-Land bills. This, said the court, constituted fraud. 49

This is a good example of where a ct. should find the corporate veil to be pierced as he was moving around funds in other to avoid creditors SECTION 3. SHAREHOLDER DERIVATIVE ACTIONS A. INTRODUCTION y HYPO: Suppose you invest in a firm that specializes in nano ipods. The treasurer absconds with 5,000 of the ipods. Your stock is now worth less, since the firm has a smaller collection of wares to sell.  The value of your stock is less b/c the firm has a smaller inventory  Who can you sue? Not injured personallyrather secondarily or derivatively injured; the value of your share decreased, but so did everyones shares Can you sue the treasurer individually? j You cant y Derivative suits Why cant shareholders who are hurt by bad choices made by the directors sue and get a recovery to compensate them for the damage they suffered from the resulting lowering of the stock price?  We will get to the business judgment rule which is a defense raised by officers and directors who are sued in that capacity If when decision was made, there was a rational business purpose, the officers and directors will be left alone Derivative suit: a suit in equity against a corporation to compel it to sue a third party. The quintessential derivative suit is a suit by a shareholder to force the firm to sue a manager for fraud.  The CoA is to force the corp. to do something  Normally, have to give the corp. the chance to do the right thing and then if you dont, you have the right to do it y Direct and Derivative Suits Direct  Alleges a direct loss to the shareholder Didnt pay me and only me by dividend  Brought by the shareholder in her own name  Cause of action belonging to the shareholder in her individual capacity  Arises from an injury directly to the shareholder Derivative  Alleges a loss to the shareholder that derives from a loss to the corporation. E.g., no one got their dividends  Brought by a shareholder on corporations behalf Corp. is the P  Cause of action belongs to the corporation as an entity  Arises out of an injury done to the corporation as an entity Why would a plaintiff rather bring a direct suit than a derivative suit?  Because who wouldnt want the check made out to them personally versus to the corporation  Better for you to gain money in your bank acct. versus gain money b/c the shares in the co. go up as a result of the check from the lawsuit  You can avoid the barriers: Avoid the possibility of a bond-posting statute j Some people may not be able to bring a lawsuit that they could have won b/c of this reqt 50

Avoid the demand j You always have to make a demand or explain why a demand was excused From Tooley v. DLJ Ask:  Who suffered the alleged harm, the corporation or the suing stockholders, individually? If it is the corporation, then it will be a derivative suit If it the individual stockholders, it will be a direct suit  Who would receive the benefit of any recovery or other remedy, the corporation or the stockholders, individually? If only you benefit, it is direct If directors reconfigure shareholder rights to harm preferred shareholders to the advantage of the common, then the preferred have a direct claim; If a manager absconds with corporate funds, the shareholders have a derivative claim. Example #1: ABC Corp entered into a contract with Jane Jones. Jones breached the contract, but ABC Corp has not sued her for that breach.  May a shareholder of ABC Corp sue Jones directly? May be supportable business reasons not to pursue a claim. As long as you have a rational business purpose for your action, here, not bringing a lawsuit, you will be protected When that comes to the shareholders attention, they cannot sue Jones directly j The shareholder is NOT in privitythe K is btwn. ABC and Jones; the shareholder is not a party j Shareholder CAN attempt to sue Jones derivatively Example #2 ABC Corps treasurer embezzles all its money and absconds. Shareholders stock is now worthless.  May a shareholder of ABC Corp sue treasurer directly? You CANNOT sue the treasurer directly b/c you are only injured b/c the corp. is injured; your injury is derivative Plaintiff Qualifications: Shareholder Status:  The shareholder is going to stand in the shoes of the corp.  Cant have more than one derivative suit at a time  MBCA 7.41(1): Must be a shareholder at the time of the alleged wrongdoing; limits standing to shareholders Albeit by negative implication Creditors may not bring derivative suit  Someone who buys after is not injured; they are essentially buying at a discount Contemporaneous Ownership  MBCA 7.42: Must be a shareholder when suit commenced how do we reach that conclusion?  Many states say also must remain a shareholder through final judgment  Why should this matter- whats wrong with buying a lawsuit? Collection agencies buy claims all the timeWe are looking for people who genuinely have the corporations best interest at heart   How about a plaintiff who buys the share AFTER the wrongdoing- any problems there? Yes, as discussed before, they essentially bought at a discount They wouldnt really be a fair and adequate representative How is this requirement satisfied when the corporation merges out of existence will the plaintiffs standing survive the transaction? 51

You sue on behalf of the corporation that gets merged out of existence Fair and Adequate Representative  MBCA 7.41(2): Named plaintiff must be a fair and adequate representative of the corporations interests  On what grounds might one challenge a plaintiffs fairness or adequacy? What if they are a competitor in addition to being a shareholder? j Conflict of interest issues Role of shareholder in derivative suit: The shareholder is in a fiduciary role he is a self chosen representative and so courts find it reasonable to impose standards of responsibility, liability, and accountability. You signed up for it so the ct. finds it reasonable to impose such standards on you You are it for your fellow shareholders so you better do a good job Policy Concerns To whom does a derivative suit belong?  It belongs to the corp. Why didnt corporation sue?  You need a nice rational business purpose for your decision not to sue The good news about derivative suits: Derivative suits allow shareholders to hold directors accountable Supreme Court called it a remedy born of stockholder helplessness The bad news about derivative suits: Potential abuses like strike suits meritless nuisance suits brought for settlement value That is why we need the demand mechanismto filter out frivolous lawsuits Who gets the recovery? The corp. How do lawyers figure into all this? Lawyers are driving this area of litigation b/c they get a piece of it They will be ones who will have someone buy a share to bring a lawsuit What do bond posting statues add to this mix? They act as a little bit of a hurdle Requires you to put up cash to back your claim to demonstrate that your claim has merit

y y

Cohen v. Beneficial Industrial Loan Co. (1949)p. 232-36 y Upholds bond posting statute y Why did he demand that the corporation take action? B/c he felt the directors and officers were enriching themselves to the tune of almost $100 million dollars He was reqd to make the demand as a predicate for the derivative action y How does the NJ statute work? It makes it such that some shareholders have to post a bond and others do not It applies to any case before final judgment y Would any shareholder bringing this suit face this statute? Only those w/ less than 5%of the aggregate par value or state capital value of all outstanding shares of such corporations stock of every class unless the shares or voting trust certificates held by such holder or holders have a market value in excess of $50,000 This discriminates against smaller shareholders y Why does the choice of law matter so much to plaintiff here- why is he so dead set against NJ law? Choice of law is critical  New Jersey has a security for expenses statute  Neither Delaware code nor FRCP 23.1 requires security for expenses 52

Was the corporation too late to make this request, when the litigation was first filed two years before? No, he is not too late y Why doesnt this disparate treatment of otherwise similar shareholders violate the due process clause? Since the statute requires the payment only of the corporations reasonable expenses, it does not offend the Due Process Clause. The application of the statute only to those whose shares are worth $50,000 or less is not a denial of equal protection. It is not unlawful discrimination y Is that fair to the < 5% / $50,000 shareholders? y Is it possible that this bond posting may be a bar to bringing a suit? Yes, it is definitely possible May be too big of a procedural hurdle But the cost of getting rid of it would be too great Eisenberg v. Flying Tiger Line, Inc. (2d Cir. 1971)p. 236-39
y

The Eisenberg transaction

Flying Tiger
Flying Tiger Corp. Flying Tiger Line

Flying Tiger Corp.


Flying Tiger Line

24

y y

Complicated transaction In the reorganization, FT organized a wholly-owned subsidiary called FTC. FTC in turn organized its wholly-owned subsidiary called FTL.  Neither of them have assets or anything In a three party transaction, FT then merged with and into its sub-sub/grandson FTL  All they really did was move the operating co. assets into the bottom co. FTL renamed itself FT and took over operations. The shareholders of old FT received an identical number of FTC shares. Thus, the shareholders of FT wound up with shares of a holding company, FTC that owned the operating company FTL now called FT. Why on earth would these companies go through this exercise? To get around the regulation Plaintiff argues: The series of corporate maneuvers were designed to dilute his voting rights and his ability to control the air freight company The reshuffling deprived him and other shareholders of their voting rights with respect to the operating company (FTL, then renamed FT).  He ends up with stock in the holding co. now instead of the operating co. HOW? Is Eisenberg really hurt by the reorganization?  Before the merger, he could vote on fundamental changes to FT and elect the FT board.  Now, he can vote on fundamental changes to FTC (the holding company his FT stock was converted into) and elect the directors of FTC. 53

y y

Because FTC wholly owns FT (the new operating company), and those FTC directors vote the FT stock, they vote on the fundamental changes to FT and elect its directors.  Eisenbergs interest is a bit more indirect than before, but not by much.  Eisenberg has not lost much by way of voting rights.  Moreover, Eisenberg could not muster the 1/3 necessary to challenge the reorganization anyway.  If he could not contest this reorganization, he could hardly hope to accomplish much with a direct vote on the operating company. That is not a reason to not give it to him, but it helps put his claim in context What about an argument by Eisenberg that because the post-reorganization shareholders had a more indirect vote, the managers of FT could now shirk their jobs more?  In other words, Eisenberg might have argued that the shareholders DIRECT vote helped to constrain the managers and ensure corporate efficiency. Now that shareholders had a less direct vote, FT (arguably) became less efficient.  Why didnt he make that argument? That is a derivative suit and he doesnt want to get into the demand/bond reqt So defendant argues: FT claimed that Eisenbergs suit was derivative, and moved for him to be required to post security for the corporations costs. Why did defendant do that instead of answering the merits?  B/c they can; it is cheaper  If they need to get into the merits then they will  But they rather get into the merits on something they consider to be frivolous after they have increased up the costs  So bond posting statutes tend to have a little bit of a chilling effect Holding: Eisenberg does not have to post security b/c his suit was not a derivative action so no security was needed. Different result if Eisenberg argued the directors violated their duty of loyalty and sought damages? Yes, he would have been suing derivatively and would have had to post a bond What about the note and question on individual recover in the Wyoming derivative action on page 241- Lynch v. Patterson(Wyo. 1985)? Unusual fact pattern: in stockholder derivative action, trial court did not err in awarding judgment in favor of minority stockholder as an individual rather than to corporate treasury, as corporate recovery would simply return funds to control of wrongdoers, and as ordering payment to corporate treasury would risk necessitating subsequent suit by minority stockholder to compel directors to declare dividend or apply the funds to legitimate corporate purposes. The court said: Direct recovery assures that Patterson will reap some benefit from his lawsuit. We refuse to order payment into the corporate treasury in this case and risk necessitating a subsequent suit by Patterson to compel the directors to declare a dividend or apply the funds to legitimate corporate purposes. How can they just do that?  They can do it because it is an equitable procedure

B. THE REQUIREMENT OF DEMAND ON THE DIRECTORS y Demand Most states require plaintiffs in derivative suits to approach the board and demand that it sue the alleged wrongdoers.  Essentially putting the board on notice that something is up; there is a beef that the shareholder has regarding something he/she believes is going wrong  Chances are that the corp. already knows about the problem, but it may respond to the demand now even though it did nothing up until now since now someone has noticed 54

 

 

             

May be other reasons, however, why the corp. may choose not to litigate If demand is required, the failure to make demand is a procedural barrier and the suit will be dismissed [unless and until the demand is met] If a shareholder has a beef against the corporation, why not automatically provide a day in court? Why do we need the demand concept? Allows the corp. to decide to take over the CoA or allow the shareholder to take over derivatively What is the purpose of the demand requirement? To give the corp. a chance to fix it Will see situations where the demand is a complete waste of time, e.g., b/c you know the board isnt going to do anything b/c either the entire board is involved or the key players on the board are involved So we are concerned about the impartiality of the board Naming all of them in the complaint doesnt excuse demandneed to show that all or enough of them are up to no good in order to excuse demand If the bad decision was the enrich themselves at the expense of the corp. there is no way that will meet the business judgment rule; that rule only protects people that made rational decisions that just turned out to be bad The decision of the board to accept or refuse the demand is evaluated based on the business judgment rule Where the board is too disabled by conflict such that we doubt their impartiality they lose the protection of the business judgment rule regarding the decision to accept or reject the demand According the Marxcourt [which sets the standard for demand excusal under NY law], the purpose of the demand requirement is to relieve courts from deciding matters of internal corporate governance by giving the directors the opportunity to correct the alleged abuse provide corporate boards with reasonable protections from harassment suits on matters within the boards discretion (strike or nuisance suits) discouraging strike suits commenced by shareholders for personal gain and not for the benefit of the corporation NOTE: ON EXAM IF APPLYING NY LAW, THIS IS THE STANDARD TO APPLY According to the Grimescourt [which sets the standard for demand refusal under Delaware law], the purpose of the demand requirement is: by requiring an exhaustion of intercorporate remedies, the demand requirement invokes a species of alternative dispute resolution procedure which might avoid litigation altogether. If litigation is beneficial, the corporation, not the shareholder, can control the proceedings; If demand is excused or wrongfully refused, the stockholder can control the litigation. NOTE: ON EXAM IF APPLYING DE LAW, THIS IS THE STANDARD TO APPLY Basic Policy Issue When, if ever, should the corporation, acting through the board of directors or a committee of directors, be permitted to prevent or terminate a derivative action? Put another way, who gets to control the litigation: the shareholder or the corporations board of directors? Arguments FOR the corporation controlling the litigation: derivative suits can act as a mechanism of managerial accountability; there is a potential for bias in interested director transactions the directors cannot realistically be expected to sue themselves the cause of action belongs to corporation - like all assets, litigation under control of BoD the shareholder may have interests diverse from those of corporation - shareholders lawyer often real party in interest Therefore BoD should have some say in whether to go forward Statutory authority re: the demand 55

      

MBCA 7.42: No shareholder may commence a derivative proceeding until a written demand has been made and 90 days have expired from the date the demand was made unless irreparable injury to the corporation would result by waiting for the expiration of the 90-day period Only carve-out is if the corp. would be injured by waiting 90 days (only waiting period can be made) No exceptionseveryone makes a demand Only a few states have adopted this Statutory authority re: the demand FRCP 23.1: The complaint shall allege the efforts, if any, made by the plaintiff to obtain the action the plaintiff desires from the directors and the reasons for the plaintiff's failure to obtain the action or for not making the effort Here you can get out of making a demand if you can show the reasons for not making such an effort Here, like most states, demand itself can be waived

How are they differe t?


  MBCA 7.42: No shareholder may comme ce Impli s that l a derivative proceedi g until th a writtenperi d has waiti demand ca be waived been made and 90 days have expired from the date the demand was made unless irreparable injury to the corporation would result by waiting for the expiration of the 90-day period

Implies that dema d itself ma FRCP 23.1: he complaint be waived shall allege the efforts, if Law i most states any, made by the plaintiff to excuses dema d if obtain the action the futile plaintiff desires from the directors and the reasons for the plaintiff's failure to obtain the action or for not making the effort

 

What is The Demand? Typically a letter from shareholder to the board of directors. Must request that the board bring suit on the alleged cause of action Must be sufficiently specific as to apprise the board of the nature of the alleged cause of action and to evaluate its merits: j At a minimum, a demand must identify the alleged wrongdoers, describe the factual basis of the wrongful acts and the harm caused to the corporation, and request remedial relief. Allison v. Gen. Motors Corp., 604 F. Supp. 1106, 1117 (D. Del.), affd mem., 782 F.2d 1026 (3d Cir 1985). Demand futility What is the most accepted excuse for not making a demand? It is a waste of time Both the Delaware and New York approaches to the demand futility inquiry boil down the following inquiry: Is the board of directors so clearly disabled by conflicted interests that its judgment cannot be trusted? j If so, absent intervention by a special litigation committee, the shareholder should be permitted to go forward. j If not, the board should be allowed to decide whether the litigation should proceed. Note: By judgment we mean what to do what to do w/ the demand What are the courts trying to balance when dealing with demand futility litigation? We are trying to protect the directors right to run the company and the shareholders right to hold the directors accountable Direct v. Derivative: The initial issue was whether Grimes complaint was derivative or direct. 56

43

The court tells us that although tests have been articulated many times, it is often difficult to distinguish between a derivative and an individual action. The distinction depends on the nature of the wrong alleged and the relief, if any, which could result if plaintiff were to prevail.

Grimes v. Donald (Del.Sup.Ct. 1996)p. 241-49THE DELAWARE TEST FOR DEMAND EXCUSAL y The deal in the employment agreement was really good for Donald y He has the ability to unilaterally determine he has been constructively terminated y Grimes is the shareholder of the co. who is a little irked about this K y The complaint alleged: abdication by the board of its duties [direct claim] as per the court, but is the court correct?  Abdication means giving up power  If it is direct, no demand is necessary breach of the duty of care [derivative claim],  When you paid so much for something such that it could not possibly have been a good business decision, you have breached the duty of care waste via excess compensation [derivative claim]. y The P is seeking to have the K declared invalid and is also seeking damages. y Were plaintiffs claims direct or derivative? Plaintiffs abdication claim: the severe financial costs [penalties] that the corporation will incur if it tries to interfere in Donalds management will inhibit and deter the Board from exercising its duties under 141(a). With respect to the abdication claim, Grimes sought only to have the agreements declared invalid and, as such, that claim was deemed direct. Is that correct? Or is this really derivative?  Albert thinks it is really derivative y Can directors delegate duties without triggering an abdication claim? When you abdicate, you are giving someone control; you cannot delegate all your duties or you have abdicated Can the abdication argument ever be a winner?  You would really need some brutal facts  Is it a winner in this case? No, but its close y What is the courts opinion on the language of the contract dealing with unreasonable interference? y Why is it not an impermissible delegation? What makes this a valid delegation? What made it a valid delegation was the fact that the Bd could take him out and put someone else in y What standard does the court use to evaluate the BODs decision to enter into the employment contract with Donald? Look to the time K was entered into and see if it could be the fruit of a rational business thought This is the Business Judgment Rule They would have run into a problem if the ct. has viewed this as an abdication; it cannot be a proper business process to take away someones power to run a corp. y Quick and dirty on the BJR: courts should not second-guess the business judgment of the directors; the principal function of the business judgment rule is, or at least ought to be, to protect officers and directors from liability for violation of their DOC. Think of it as a rebuttable presumption It projects business decisions that directors made in good faith If the shareholders can show the officer or director breached a duty, he/she will rebut the presumption 57

y y

Could show a conflict of interest to rebut, e.g., 9/10 of the Board are owners of a co. that breached a K  What do we have to show? It is about the decision to litigate and not the underlying K.  Need to show that the decision to not litigate was wrongful, e.g., only a cursory examination which shows a breach of the duty of care Under the BJR, a board decision to incur the risk of such a financial penalty in order to attract a senior manager was entitled to protection. If an independent and informed board, acting in good faith, determines that the services of a particular individual warrant large amounts of money, whether in the form of current salary or severance provisions, the board has made a business judgment [that] will normally receive the protection of the BJR unless the facts show that such amounts, compared with the services to be received in exchange, constitute waste or could not otherwise be the product of a valid exercise of business judgment. Why do we even get to the discussion of demands and excusal here? Did the plaintiff satisfy the statutory requirement of a demand to the Board? B/c the P made a demand How did the company respond to his pre-suit demand? The Board refused it On what basis did the BOD make the refusal? They say that they seriously considered it Were these steps sufficient to satisfy the BJR? Yes, obviously enough to satisfy the BJR What was the legal effect of Grimes making this demand on the BoD before filing suit, as required by Delaware version of FRCP 23.1? When you make a demand you are conceding that demand is reqd You are waiving the rt. to litigate the need for a demand So what does plaintiff have to do to get his day in court, after his demand has been refused? You will litigate the wrongful refusal Must show the decision of the board was not covered by the business judgment rule If demand is made and rejected, the board rejecting the demand is entitled to the presumption of the BJR unless the stockholder can allege facts with particularity creating a reasonable doubt that the board is entitled to the benefit of the presumption.  Almost always a losing arg. If there is a reason to doubt that the board acted independently or with due care in responding to the demand, the stockholder may have the basis ex post to claim wrongful refusal. The stockholder then has the right to bring the underlying action with the same standing which the stockholder would have had, ex ante, if demand had been excused as futile.  This is the possible escape hatch that lets you proceed on the merits Did Grimes successfully show that demand was wrongfully refused? No, he did not So heres the problem for plaintiffs from the BJR presumption: 1. Under Delaware law, where demand is made the plaintiff is deemed to have conceded that the demand was required [waiver of any claim that demand is excused], 2. which in turn makes the decision of the board on whether to dismiss a matter of business judgment, 3. which in turn means that the plaintiff loses. And where demand is required, or made, the plaintiff is not entitled to discovery. Since derivative plaintiffs are denied discovery, how can they ever show the factual basis for the excuse? Very difficult to overcome the BJR w/o discovery 58

y y y

y y

y y

y y y

Under Delaware law, can plaintiffs demand ever be excused? Plaintiff must allege particularized facts showing why he was justified in not making the demand.  Why does plaintiff have to allege particularized facts? It saves corporations from shareholders who have lawyers who are pushing them to sue or who have different agendas Grimes The three usual bases for excusing demand as futile are: (1) a majority of the board has a material or familial interest in the challenged transaction; (2) a majority of the board is incapable of acting independently, maybe because it is dominated or controlled by the alleged wrongdoer; or (3) the challenged transaction was not the product of a valid business judgment. A board setting its own fees would be an example of a conflicted transaction If 6/10 were shareholders in the target co., that could constitute a conflict such that the business judgment rule would not be applied There may be circumstances where 1 & 2 are not true, but 3 is, so that is why we need 3 even though it seems extra and unnecessary when you have 1 & 2 As a result, well-advised plaintiffs in Delaware almost never make demand, since the board always says no and the BJR presumption supports the board. So the issue to be litigated is whether demand is excused. Aronson v. Lewis (1984)p. 246 This is the case that preceded Grimes (see p. 246 of the CB) Aronson introduces the concept of reasonable doubt into this analysis. The case creates a test for trial courts to see if the failure to make a demand was justified by using discretion to determine whether the particularized facts alleged in the complaint create a reasonable doubt that:  the directors are disinterested and independent for purposes of responding to the demand; OR [Aronson court says and, but acts like its or; and the NY courts in Marxspecifically interpret the branches of this test as disjunctive]  the challenged transaction was otherwise the product of a valid exercise of business judgment. Self interest, for these purposes, is defined in terms of direct financial interest in the challenged transaction; the fact that a majority of directors voted to approve the transactionand therefore are named as defendantsdoes not constitute self-interest and will not excuse demand. To summarize: If demand is required and made, it will invariably be refused; then BJR applies to the decision to dismiss the lawsuit and plaintiff loses. Demand may be excused if plaintiffs allegations raise reasonable doubt about self interest or the protection of the business judgment rule, but then the directors can appoint a special litigation committee as in Zapata [see footnote 13]. where demand is excused, the corporation may still move to dismiss the suit as not in the best interests of the corporation. The recommendation may come from a special committee of independent directors, as in Zapata. NOTE: On EXAM, if Delaware Corp., walk through Grimes

Marx v. Akers (N.Y. 1996)p. 250-55NY TEST FOR DEMAND EXCUSAL y Whats the big factual difference between Grimes and Marx? Here, P (the shareholder) does NOT make a demand. So that makes the lawsuit about demand excusal In Grimes, since P make a demand, the lawsuit was about whether the refusal of the demand was wrongful under the BJR 59

y y

y y

Here, we are able to get into the standard for demand excusal under NY So P files a lawsuit and D makes a motion to dismiss for failure to make a demand What is this courts view on derivative actions in general? By their very nature, shareholder derivative actions infringe upon the managerial discretion of corporate boards Consequently, we have historically been reluctant to permit shareholder derivative suits, noting that the power of the courts to direct the management of the corporations affairs should be exercised with restraint.  A derivative lawsuit IS getting into the internal workings of a corp. the object is for the court to chart the course for the corporation which the directors should have selected, and which it is presumed that they would have chosen if they had not been actuated by fraud or bad faith. Due to their misconduct, the court substitutes its judgment ad hoc for that of the directors in the conduct of its business.  So in this situation, the ct. will step in and do what should have been done  Only when the Boards decision does not practice business judgment muster will we get to this point What cases does the NY court cite to? Aronson and Barr This is a case of first impression in NY Why does the Marx court talk about Aronson and not Grimes? Grimes was happening at the same time and that is why they use Aronson and not Grimes Is it sufficient to simply name a majority of the directors as defendants? No the ct. tells us correctly that conclusory allegations are not the same thing as breaches of fiduciary duty When you sue the corp. derivatively you are going to name all the directors Not enough to allege all directors were involved, not enough to allege all directors were directors when the action alleged occurred, really need to show the breach of the duty of loyalty. Rule of law: under NYBCL 626(c), plaintiff has to set forth with particularity the efforts of the plaintiff to secure the initiation of such action by the board, or the reasons for not making such effort. Three prong disjunctive standard: Majority of directors interested in challenged transaction  If the majority of the Board of Directors are involved in the transaction that the shareholder is upset about, isnt it suspect? Arent we concerned about the ability of that group of people to be impartial and discharge their duty of loyalty in a decision about what to do about that challenged transaction?  That decision should not get business judgment protection.  Fact that you were named as a D is not enough, need to show you breached your duty Directors failed to inform themselves to degree reasonably appropriate  Saying that we are take away the reqt of demand if you have shown us a breach of the duty of care of the duty of loyalty w/ respect to the underlying transaction Challenged transaction so egregious that it could not have been the product of sound business judgment of the directors  Sounds like the 3rd prong in Grimes  When your directors have breached duties and the like we worry about their business judgment on their ability to handle litigation on the underlying act In demand excusal cases, we talk in the hypothetical. Could the Board have decided or were they too conflicted? How does the Marx court view plaintiffs allegations about the compensation paid to IBMs executive officers/inside directors? Inside directors are good news and bad news One of the claims here is about money to these individuals 60

Arg.: You are paying these officers so much money that it couldnt be the product of a good business judgment and thus breached the duty of care and therefore, they couldnt have possibly properly evaluated a demand Since only 3 directors are alleged to have the benefit of the exec comp scheme, P has failed to allege that a majority of the BOD was interest as such w/ respect to these officers/directors, Ps complaint is dismissed for failure to make a demand How does the Marx view plaintiffs allegations concerning the compensation paid to IBMs outside directors? If you are just a director, you are an outside director We think they will be more impartial/neutral; they may not have the allegiance issues the inside directors have P also alleged that a majority of the board was self-interested in setting the comp b/c the outside directors comprised a majority of the board: here, demand excused! 3/18 is not enough; 15/18 is a majority and will be enough If you dont make a demand and you plead demand futility you are going to at least begin to explore the conduct of the board and why they cant make the decision If you lose, you make the demand Does that help plaintiff in the long run? No fails to state a CoA Good news for P on the outside directors claim, the ct. found 15/18 satisfies the standard for demand futility and therefore P is not reqd to make a demandthose guys are too involved in the underlying transaction such that we are concerned about their ability to make a proper decision on that demand Bad news is P is unable to show the money is excessive What would be the result if the court found the opposite way -- that the demand was required, and the complaint stated a cause of action?

Derivative Litigation Decision Tree

  
85

  

Di

        

Di

D iva iv ? D iva iv

 
In a di i,P and y a all done 61

Derivative Litigation Decision Tree


irect or irect

sues

Is de and futile? If no, de and required

If yes, demand excused and sues


86

Direct or Deriv tive? Direct Derivative

es

Was demand made?

No was there a colorable claim? Marx Analysis Q4

es

Is demand futile? If no, demand required

If yes, demand excused and sues

Was demand Made?


87
88

Deri ti e iti ree


Demand es es

ti
ade?

Deci i

Derivative Litigation Decision Tree (2)


Demand Made? If demand is required, Yes plaintiff almost certain Demand to end up here Refused? Yes No BOD sues No

n sui

E C G FCED C

OD sues

Re usal wr ngful? es sues


89

Demand Re used? N

Marx Anal sis Q4

Refusal wrongful?

Stop; no suit

62

2 52

24

2 42

16 32 4 321 0

"%"

"$

'

"$"

#"$ #"!

e a ee

eL

ga on

ec

on

 9 I

erivative? Derivative

)(

  C !& A

This is the situation in a derivative suit


e a eL ee (2) ga on ec on

Standard? No BJR as applied Marx Analysis to decision to Q4 refuse demand

Yes
sues
90

Direct or Derivative? Direct Derivative

De a req ire

The Task - SLC


preserving the discretion of directors to manage a corporation without undue interference Marx v. Akers permitting shareholders to bring claims on behalf of the corporation when it is evident that directors will wrongfully refuse to bring such claims Marx

92

What effect a court should give to any board decision not to sue (especially in demand excused cases)? Here we get to the role of the special committee. Should the courts defer under the business judgment rule? Or should they instead reexamine the BOD or committee decision?

C. THE ROLE OF SPECIAL COMMITTEES Auerbach v. Bennett (N.Y. 1979)p. 256-61: Special Litigation Committee y What did the SLC do in the scope of their review? They hired special counsel not connected to the firm (goes to the fiction of impartiality) They were back through everything the audit committee did and tested it again They talked to outside counsel hired just to help them They interviewed all directors present when the alleged action occurred They send questionnaires to everyone y What did the SLC conclude? They found that there was bribery going on. However, the rejected the demand y Issue: How much deference should court accord SLC recommendation to dismiss suit? y we confront a special instance of the application of the BJR and inquire whether it applies in its full vigor to shield from judicial scrutiny the decision of a three person minority committee of the board acting on behalf of the full board not to prosecute a shareholders derivative action. y Rule: In evaluating the decision of a special committee, the court must examine the Selection of procedures used to undertake its project, and 63

Needed: A filter to separate cases in which the board is disabled by conflicts of interest from making an independent decision in good faith

Majorit of board has material interest; or Majorit of the board lacks independence; or hallenged transaction not product of valid exercise of business judgment

p p

Majorit of directors interested; or Directors failed to inform themselves; or hallenged transaction could not have een the roduct of sound usiness judgment

` W `Y X ` X `Y X

De a De a

` W cb a

De a s es Board disabled by some conflict, but De a e c se can board regain control anyway via s es SLC?

W `Y X

W ` W d `Y X

RP

D riv tiv Tr

iti

ti

isi

f tility

UT
a e? ref se ?
91

UT

Q W

PP P

The Demand Requirement as Filter




NY standard:

Delaware standard: Reasonable doubt as to:

93

y y y

The ultimate substantive decision predicated on the chosen procedures.  This falls squarely within the application of the BJR and is thus outside scope of review (necessarily requires the weighing of legal, commercial, public relations, and fiscal factors). Did the court find the members of the Special Litigation Committee to be interested or independent? Independent What does plaintiff say about that determination? What does the court say about that? Relevance of the BJR? As all parties and both courts below recognize, the disposition of this case on the merits turns on the proper application of the business judgment doctrine, in particular to the decision of a specially appointed committee of disinterested directors acting on behalf of the board to terminate a shareholders' derivative action. . . . In this instance our inquiry, to the limited extent to which it may be pursued, has a two tiered aspect. Effect of Second-Tier Decision What did the defendants argue with respect to the effect of the second-tier decision (the committees recommendation)?  The committee did an impartial job and that decision is bullet-proof under the BJR Does court agree?  Ultimate decision covered by business judgment rule  But judicial inquiry permitted with respect to: Disinterested independence Adequacy of investigation j This speaks to an examination of the duty of care  Burden of proof? On plaintiffto show the SLC is not disinterested and their investigation procedures were not adequate While the court may properly inquire as to the adequacy and appropriateness of the committees investigative procedures and methodologies, it may not under the guise of consideration of such factors trespass in the domain of business judgment.  They are not getting into the merits of the decision; they are getting into the process of the decision  As long as it is a decision that doesnt breach fiduciary duties, it stands Hold a sub-set of a Board to the same standards Application Factors considered re: disinterested independence?  Someone w/ no financial interest in the transaction disconnected Would be great if they had no connection w/ the company at all Should be assisted by a new set of lawyers, new set of bankers, etc.  Someone with no prior affiliations  Certain degree of competency Factors considered re: adequacy of procedures  Directors need to follow proper procedure  Need to make a good faith inquiry  Thus, decision not to litigate will be permissible Demand Really Excused? Court assumes demand excused, but was it?  Auerbach is in NY so will use the Marx test for demand excusal Marx v. Akerstest:  Majority of BOD Interested? Allegation was that foreign officials were being bribed to benefit the company Not really seeing a direct financial gain 64

BOD failed to inform itself? or through its committee? j Board didnt undertake the investigation as a whole; had a sub-set do it j Committee did a lot so the committee did not fail to inform itself and as such, the Board did not fail to inform itself  Transaction egregious? Wont be a way to excuse demand here Doesnt seem like the test for demand excusal is fulfilled, so why is there no discussion about demand excusal here?  Was a demand even made? P can make a demand or file a lawsuit; here, the P filed a lawsuit Corp. can answer Ps conduct depending on what P does If the P doesnt make a demand and files a complaint, one option for the corp. is to answer by saying where is your demand or bond posting j Seen where corp. wants this to go away quickly j Seems like here, demand would not have been excused, but then it sort of looks like the D has something to hide So instead the D set up a special litigation committee to deal w/ the lawsuit Zapata v. Maldonado (Del. 1981)p. 261-68Delawares Case Law Holding on SLCs y Did Maldonado make a demand? No, he doesnt He claims the demand is excused claims he named all the directors as Ds, therefore, they are all liable and not in the position to rule on the demand, would have been futile to make the demand (we know this isnt a valid reason) y So what does the corporation do about this what are its choices? They can create a special committee to evaluate his claims; or they can wait till he files the lawsuit, and defend with his failure to make a demand. y The lower court opinion found BJR does not confer power on BOD to terminate a derivative suit. y Is that supportable? BJR is a rebuttable presumption The BJR itself creates nothing y Does the BJR create authority? It does not create authority, it confers nothing on the Board It is a case law defense y How does the BJR apply to this case? Here, the evaluation of the Committee/BOD decision not to sue only becomes relevant when plaintiff protests it. The BJR then is raised as a defense to the attack on the decision.

This court views the case as having 3 focuses: Whether the lower court was right in finding that the shareholder had an individual right to maintain the derivative action. What is the corporate power under Delaware law of an authorized board committee to cause the dismissal of litigation instituted on behalf of the corporation. What is the role of the Court of Chancery in resolving conflicts between stockholders and SLCs? ISSUE #1: the lower court found that once a shareholder made a demand and it was refused, the shareholder somehow had an independent individual right to continue the derivative action over the objections of the corporation- is this an accurate statement of the law? Did our plaintiff make a demand?  No, initial decision about whether to litigate was posed to the corp. via the Ps complaint 65

What would have happened if he made a demand?  They could have decided to bring a lawsuit or they could have refused to do so Does this process bear on the issue of who controls the litigation?  Doesnt matter, ball still moves to Ds ct.  Once P does whatever he does, next move will always be on the corp. who will determine how the litigation will unfold Does it make sense to you that the court says that if the BOD refuses a demand, their authority to choose to litigate is not challenged, even if its ultimate conclusion is not respected as wrongful?  It takes some thinking, but it works. They have the authority to make the decision, but the decision is subject to review.  The only time they get into a situation if that decision is ultimately proved to have been wrongfully decided.  Not taking away their ability to make a decision. A demand, when required and refused (if not wrongful), terminates a stockholders legal ability to initiate a derivative action. But where demand is properly excused, the stockholder does posses the ability to initiate the action on his corporations behalf. ISSUE #2: What is the corporate power under Delaware law of an authorized board committee to cause the dismissal of litigation instituted on behalf of the corporation? Where would a committee get power to do anything?  From 141(a), corp. BOD runs the co.  From 141(b),provides for the qualifications of the directors, the # of directors, etc.  From 141(c), committee can do anything BOD can do  So the committee has the color of authority that the Board has Who appointed the committee?  The interested directors appointed the committee What is the result of the courts analysis of section 141(c)?  Ct. correctly notes that the committee can do anything the Board can do b/c the committee was appointed by some of the Board members A BOD decision to cause a derivative suit to be dismissed as detrimental to the company, after demand has been made and refused, will be respected unless it was wrongful. Absent wrongful refusal the stockholder in such a situation simply lacks managerial power ISSUE #3: What is the role of the Court of Chancery in resolving conflicts between stockholders and SLCs? Rule of law: Delaware standard for reviewing SLC recommendations is a two step approach:  Zapata Two-Step Step 1:Inquiry into the independence and good faith of the committee j Inquire into the bases supporting the committees recommendations j The corporation has the burden of proving independence, good faith, and a reasonable investigation Step 2: Court may go on to apply its own business judgment as to whether the case is to be dismissed j The court will undertake an independent inquiry into whether the suit should be dismissed. j The court can consider matters of law and public policy in addition to the corporations best interests.  What is the function of step #2? It allows meritorious suits to go forward, and helps account for structural bias problem.  How is step #2 different from Auerbach? Court decides whether there was a reasonable basis for the decision 66

Albert sees this as an overstepping; she is concerned about courts getting into the daily operations of corporations
y

Analysis Zapata far more intrusive judicial review than usual. Why?  Context: Demand was excused because board disabled from acting due to conflicted interests  Committee appointed by the disabled board  Potential for structural bias there but for the grace of God, go I Concerned that these seemingly impartial directors now have to point the finger at the person who appointed them. The benefit of SLCs has been removed; the court seems to have effectively gotten to the same place as the legislature getting rid of SLCs Why does the court distinguish between cases where the independent litigation committee investigates whether to sue after a derivative plaintiff has made demands on the firm, and cases where the plaintiffs demand on the firm has been excused? If a demand has been made, the issue of the Boards impartiality is waived So the court distinguishes btwn. Ps who gave up the issue of Board impartiality when they made the demand and the non-demand Ps did not Those are two very different Ps

CHAPTER 4: THE LIMITED LIABILITY COMPANY SECTION 1. FORMATION y Limited Liability Companies 67

Cross between partnership and corporation (hybrid)  Tax advantages of partnerships Partnership does not get taxed at the entity level (only gets taxed once at the partner level); corporation does (when you have a taxable entity, gets taxed at the entity level and the individual leveldouble layer of tax)  Limited liability of corporations Until a partnership, dont have unlimited personal liability of all partners  None of the restrictions (e.g., number and type of shareholders) applicable to S corporations Need to figure out whether need to make new law for the hybrid, apply partnership law or apply corporation law. History LLC is the first new legal business concept since the S corp (1950s) LLCs introduced in Wyoming in 1977 Created as a vehicle for ownership of real estate and development of oil, gas and other mineral rights Tax treatment of LLCs  The IRS initial position [pre-1988] was that LLCs should not be treated as a partnership  In 1988, IRS ruled that LLC could qualify for partnership-like tax treatment  In 1997, IRS further liberalized with check the box regulations  Check-the-Box Regulations give unincorporated associations the ability to choose their tax status without regard to the entitys nontax legal characteristics Why are LLCs so popular? They provide a standard form contract that incorporates many of the most attractive features of partnerships and corporations. The LLC is an unincorporated business organization that can provide its members with pass through tax treatment, limited liability, and the ability to actively participate in firm management. The Limited Liability Company Funding  Members typically contribute capital  Contribution may be cash, property, services rendered, a promissory note, or other obligation to contribute cash, property, or to perform services. See ULLCA 401. Liability  Members stand to lose capital contributions, but their personal assets are not subject to attachment Tax Consequences  Income and losses pass through to members  LLC does not pay taxes at the entity level, so profits are not subject to double taxation  Capital gains flow through to owners and retain their tax attributes (i.e., are subject to lower rates) Financial Interests  Profit and Loss Sharing Absent contrary agreement, most LLC statutes allocate profits and losses on the basis of the value of members' contributions Compare partnership laws equal division default rule  Withdrawal Member may withdraw and demand payment of his/her interest upon giving the notice specified in the statute or the LLC's operating agreement Assignment of LLC Interest  Unless otherwise provided in the LLC's operating agreement, a member may assign his financial interest in the LLC An assignee of a financial interest in an LLC may acquire other rights only by being admitted as a member of the company if all the remaining members consent or the operating agreement so provides. See ULLCA 501-503  Analogous to partnership rules 68

Management Rights  absent contrary agreement, each member has equal rights in the management of the LLC; see ULLCA 404(a)(1) Most matters decided by majority vote, see ULLCA 404(a)(2) Significant matters require unanimous consent, see ULLCA 404(c) j E.g., merger, admission of new member, dissolution, etc...  Manager-managed LLC option available. See ULLCA 404(b)  Member-managed LLC is the other choice. See ULLCA 404(a) Fiduciary Duties  Manager-managed LLCs The managers of a manager-managed LLC have a duty of care and loyalty Usually, members of a manager-managed LLC have no duties to the LLC or its members by reason of being members  Member-managed LLCs All members of a member-managed LLC have a duty of care and loyalty Derivative Actions  Member may bring an action on behalf of the LLC to recover a judgment in its favor if the members with authority to bring the action refuse to do so Liabilities  No member or manager of a limited liability company is obligated personally for any debt, obligation, or liability of the limited liability company solely by reason of being a member or acting as a manager of the limited liability company  Members of an LLC are not liable simply b/c they are members Formation  File articles of organization the designated State office. ULLCA 202(a) Required and optional contents set forth in ULLCA 203 Filing fees plus $800 minimum franchise tax  Other formation tasks: Choose and register name: LLC statutes generally require the name of the LLC to include the words limited liability company, the abbreviation LLC, or similar phrases. ULLCA 105 j Effect of, see Water, Waste & Land v. Lanham Designate office and agent for service of process Draft operating agreement the basic contract governing the affairs of a limited liability company and stating the various rights and duties of the members j Takes the place of the partnership agreement Default rules for LLCs  Typical LLC statutes provide default rules, but with flexibility.  The default rule is comparable to the general partnership form, (1) vesting management in the LLCs members, (2) except that the number of votes cast by each member is voting is determined by their proportional share in the book value of the membership interests.  In general, both of these rules are subject to any contrary provisions of the articles of organization and operating agreement. The LLC thus provides substantial flexibility in structuring the firms decisionmaking processes. Water, Waste & Land, Inc. d/b/a Westec v. Lanham(Colo. 1998)p. 300-04 y Preferred Income Investors, L.L.C. (PII) was an LLC organized the laws of Colorado y Clark is interacting w/ Westec; hires them to perform engineering services; they fully perform the services before getting paid.
y

What is the significance of Clark telling Westec to send a written proposal to Lanham? Creates some notice issues 69

y y

y y

y y y y y y

y y

They dont ask who Lanham is, they send a written proposal to Lanham who does nothing Even though they hear nothing, they fully perform This puts Westec on notice that someone named Lanham exists; this creates a little more reason than just the PII, for Westec to exercise some due diligence Starting to send like apparent authority being created Oral K that is executory, Westec performed and have not been paidthis is the only binding arrangement here; the written proposal was never signed Question becomes who is responsible for the breach PII admits liability, but they are a totally unfunded LLC Westec realizes a judgment against the LLC is not the way to go  Will either have the pierce the LLC veil (which we will later learn about); or  Take the agency route Whats the bottom line at the trial court [county court] level? Clark is an agent for Lanham; Clark is not personally liable Whats the holding at the district court level? It reverses the trial ct. The notice provision of the LLC provides that files of the Articles of Organization serve as constructive notice of a companys status as a LLC. What is the result of the district court decision? Dist. Ct. interpreted the LLC Acts notice provision as putting Westec on constructive notice of Lanhams agency relationship w/ the co. This course of analysis assumed the LLC Act displaced certain common law agency doctrines, at least insofar as these doctrines otherwise would be applicable to suits by TPs seeking to hold agents of a LLC liable for their personal actions as agents. The Col. SC holds: the statutory notice provision applies only where a TP seeks to impose liability on an LLCs members or managers, simply due to their status as members or managers of the LLC. When a TP sues a manager or a member of an LLC under an agency theory, the principles of agency law apply notwithstanding the LLC Acts statutory notice rules. Rule: When a third party sues a manager or member of an LLC under an agency theory, the principles of agency law apply notwithstanding the LLC Acts statutory notice rules. How do these facts work out under standard agency law? Under common law agency rules, A has liability on a partially disclosed agency to avoid liability, A must fully disclose both the agency and the identity of P. Well who did Westec think it was dealing with? They thought they were dealing w/ Clark as an agent for Lanham. What conclusion does the court reach? They find this is be a partially disclosed principal. How is this a partially disclosed agency? It sounds undisclosed - and both Clark and Lanham are agents for the undisclosed LLC! The court tells us that the partially disclosed agency exists because Clark and Lanham failed to disclose the existence OR identity of the LLC. Does this make sense? Yikes. This sounds much more like an undisclosed agency. The court seems a little mixed up on the definition of partially disclosed agency, as evidenced by this language from the full opinion: For these reasons, we conclude that where an agent fails to disclose either the fact that he is acting on behalf of a principal or the identity of the principal, the notice provision of our LLC Act, section 7- 80208, cannot relieve the agent of liability to a third party. We get to the right conclusion, but the court seems to be a little mixed up. When a third party deals with an agent acting on behalf of a limited liability company, the existence and identity of which has been disclosed, the third party is conclusively presumed to know that the entity is a limited liability company and not a partnership or some other type of business organization. True, just doesnt happen to be the facts here. 70

y y

Where the third party does not know the identity of the principal entity, however, the situation is fundamentally different because the third party is without notice and the law does not contemplate that he has any way of finding the relevant records. Here, have a TP, Westec who does not know the existence or identity of the Principal. So we cant hold them responsible for notice. Does it matter if the agency is partially or undisclosed? They are the same in the respect that the TP gets to choose who to sue. What is the applicable state statute? It is a constructive notice statute.

SECTION 1. THE OPERATING AGREEMENT Elf Atochem North America, Inc. v. Jaffari (Del. Sup. Ct. 1999)p. 305-11 y Operating agreement is like the partnership agreement y Can the parties say whatever they want in their LLC Agreement? Can pick a state law that works for you (choice of law), a forum that works for you (forum selection clause), can pick arbitration.  Note: The arbitration and forum selection clauses were only in the big Agreement.  Should always have a choice of law provision, but that doesnt always mean it will be honored  Recall that what law ends up being applied will matter since different states often have different tests  Here, the agreement specifies CA law  Forum chosen was also CA Freedom of Contract  Default rules under ULLA  There are some limits on freedom of K, but for the most part, freedom of K will carry the day  Cant eliminate the duty of loyalty or duty of care  Just cant leave out certain mandatory statutory provisions Can have an arbitration clause  Claimed the arbitration clause is unenforceable.  Arbitration clause requires any dispute arising under the agreement to be arbitrated These decisions have no precedential value You can decide who your judges are good news if you have a colorable claim j Depends if specialization helps your situation y Does plaintiff claim direct or derivative claims? Can LLCs be sued derivatively, like corporations?  Dont need derivative suits in partnership b/c people already have personal liability y Why in Delaware? y How does Elf hope to get around the arbitration/choice of law issue? By arguing breach of distribution or the employment agreement However, you cant arbitrate an arbitration provisionwill have to litigate in CA y What about his claims tied to the Distributorship Agreement? Distribution agreement was deemed to arise out of the operating agreement y What about plaintiffs argument that the Delaware Chancery Court somehow still has special jurisdiction over this matter since it involves a Delaware LLC? Parties agreed these cases would be heard in CA DE have absolutely no jurisdictionspecial or otherwise What P is trying to get at is if you dont specify at all and the agreement is silent, then the default rules kick in Here, the ULLA constitutes the default rulesin the ULLA, it gives the DE Chancery Court special jurisdiction; but since the parties provided otherwise, the default rule will not apply 71

y y

Strong public policy aspects Strong public policy favoring arbitration in CA Holding: (1) agreement binding on LLC as well as the members; (2) since Act does not prohibit members of an LLC from vesting exclusive SMJ in arbitration proceedings (or ct. enforcement of arbitration) in CA to resolve disputes, the contractual forum selection provisions must govern.

SECTION 3. PIERCING THE LLC VEIL PCV [in the corporate context]: we typically need: (1) a total disregard for the separate existence of the corporation Need unity of interest Example: Comingling of bank accounts; argument that if you are not going to separate yourself from the corporation, why should a creditor? (2) Most of the cases and the commentators state that there is a second element that must be proved: injustice or some such concept. y To what extent should the corporate law rules apply to LLCs? 302. Limited Liability Company Liable for Members or Managers Actionable Conduct:  A limited liability company is liable for loss or injury caused to a person, or for a penalty incurred, as a result of a wrongful act or omission, or other actionable conduct, of a member or manager acting in the ordinary course of business of the company or with authority of the company.  Saying that the company is liable for the acts of its owners in the same way a corporation would be  This wont help w/ piercing 303. Liability of Members and Managers.  (a) Except as otherwise provided in subsection (c), the debts, obligations, and liabilities of a limited liability company, whether arising in contract, tort, or otherwise, are solely the debts, obligations, and liabilities of the company. A member or manager is not personally liable for a debt, obligation, or liability of the company solely by reason of being or acting as a member or manager.  Keep in mind that there are two different flavors of LLCs  This is the beginning of a piercing statutethis statute tells us that LLCs are responsible for bad stuff their people do and that the people who do them are not going to be liable just b/c they are owners.  Makes us think there are other tests we will put these people through to figure out when a member or manager is going to be liable 303. Liability of Members and Managers.  (b) The failure of a limited liability company to observe the usual company formalities or requirements relating to the exercise of its company powers or management of its business is not a ground for imposing personal liability on the members or managers for liabilities of the company.  The uniform statute actually statutorily indicates that failure to observe formalities is not a ground for personal liability 303. Liability of Members and Managers.  (c) All or specified members of a limited liability company are liable in their capacity as members for all or specified debts, obligations, or liabilities of the company if: (1) a provision to that effect is contained in the articles of organization; and (2) a member so liable has consented in writing to the adoption of the provision or to be bound by the provision.  Basically unless the individuals agree otherwise, they are not going to be liable for the things the LLC does  This sounds very much like a corporation as opposed to a partnership b/c the partnership owners have unlimited personal liability Kaycee Land and Livestock v. Flahive(Wyo. 2002)p. 312-14
y

72

y y

y y y y

y y

When people seek a roundabout way of litigating, there is usually a reason for it Issue: in the absence of fraud, is a claim to pierce the LL entity veil or disregard the LLC entity in the same manner as a court would pierce the corporate veil, an available remedy against a Wyoming LLC Essentially, can you pierce the corporate veil in Wyoming?  If no, then we are done  If yes, then will determine whether appropriate to do so here Wyoming law: Section 17-15-113: Neither the members of a limited liability company nor the managers of a limited liability company managed by a manager or managers are liable under a judgment, decree or order of a court, or in any other manner, for a debt, obligation or liability of the limited liability company. So have piercing in the corporate context, but it is not in the LLC statute Requested that the court say there are no circumstances in which the LLC veil can be pierced Argued since the ULLA was adopted, the legislature did not mean to allow piercing the corporate veil in Wyoming However, the Wyoming law does not prohibit an LLC veil from being pierced, it doesnt preclude it P says lack of statutory disposition is not dispositive ULLCA 303(a): A member or manager is not personally liable for a debt, obligation, or liability of the company solely by reason of being or acting as a member or manager. MBCA Cf.: MBCA 6.22(b): a shareholder of a corporation is not personally liable for the acts or debts of the corporation except that he may become personally liable by reason of his own acts or conduct The Wyoming legislature based their statute on a statute that had a carve-out for piercing but they adopted it in a form that didnt address piercing Although the legislature has not expressly provided for veil-piercing to apply w/ LLCs, piercing the corp. veil has traditionally been a judicially created remedy to avoid an otherwise unjust result. Minn. Stat. 322B.303(2) case law that states the conditions and circumstances under which the corporate veil of a corporation may be pierced under Minnesota law also applies to limited liability companies. This is a typical LLC statute Many states have statutes that mirror corporate law Differences btwn. a corporation and a partnership that would come in on this discussion? Much less statutory law on partnership than corporations Big differences in terms of liability of the owners So if we allow you to pierce, arent we really giving you two bites at the apple? As a public policy matter, it would be illogical to allow person to abuse LLCs in a manner that has been prohibited for corps

SECTION 4. FIDUCIARY OBLIGATIONS McConnell v. Hunt Sports Enterprises(Ohio 1999)p. 317-22 y McConnell et al.: Local business leaders with lots of political pull y Hunt: Kansas City businessman with pro sports experience (owns NFL Kansas City Chiefs) y Form LLC to develop NHL franchise in Columbus, Ohio Deal runs into trouble McConnell takes deal for self y Looks less like tortuous interference b/c they came to him y Operating agreement said members of the LLC are allowed to compete directly It is very broadalmost limitless When you bring people together w/ common interest, they may also automatically be in direct competition 73

Is Sectio 3.3 of t e A ree e t Ma ifestly U reaso a le ?

y y

Why is it so important for the court to figure out whether Section 3.3 is manifestly unreasonable? So we will allow parties to compete w/ an LLC up to a limit Cant eliminate the duty of loyalty or care; can only get right up against it  So cant scrap the duty of loyalty but you can get close; can come up w/ things that are deemed not to be violating the duty of loyalty (conduct and activities; types or categories of activities that the parties in freedom of K agree are not going to be violating the duty of loyalty) So if the provision is manifestly unreasonable can bring in all other kinds of issues What would make it manifestly unreasonable? Is it manifestly unreasonable?  Manifestly unreasonable sounds like something that is so egregious on its face that we are not going to let it go forward  Doesnt sound like that b/c the LLC had the opportunity to take on the project in the beginning; but Hunt kept saying no on behalf of the LLC So it is that the provision doesnt provide activities that are manifestly unreasonable So from a K point of view:  If K is ambiguous, will bring in extrinsic evidence Seems more like a partnership here; really picks up on UPA language as opposed to a corporate statute Holding: The plain language of the K states that members of the LLC may compete. An LLC, like a partnership, involves a fiduciary relationship. But the operating agreement allowed competition, and so limited the scope of fiduciary duty Rule: An operating agreement of an LLC may limit or define the scope of fiduciary duties imposed on its members, so long as it is not manifestly unreasonable.

NOTE: ON EXAM y If you get these provisions: need to interpret K provision in context of what statute says. But remember that you can limit the DOL. y HOW YOU WOULD DO IT: Here is the provision, defend under ULLCA 103 We have drafted a superbroad provision, and the statute allows us to take categories out of the DOL unless manifestly unreasonable. Its not manifestly unreasonable because XYZ.  Think about whether the provision is too broad, and satisfies the statute. The way to do this is to look at the application. Start with talking about the language, then assuming the language of the K is enforceable, start talking about the conduct j 1. Look at the language of K, and ask if its manifestly unreasonable make the argument: they can do whatever they want j 2. If a ct does find its not manifestly unreasonable, then test the conduct to see if the conduct is manifestly unreasonable Could strike, reform, separate 74

x y

g e e g fg g g g e f f e d d d

ULLCA 103( )(2): o erati a ree e t ay ot eli i ate t e ty of loyalty t ay i e tify s ecific ty es or cate ories of activities t at o ot violate t e ty of loyalty, if ot a ifestly reaso a le ....

Sectio 3.3 of t e o erati a ree e t: Me ers May Co ete. Me ers s all ot i a y ay e ro i ite fro or restricte i e a i or o i a i terest i a y ot er si ess ve t re of a y at re, i cl i a y ve t re ic i t e co etitive it t e si ess of t e Co a y.
41

r u

v r

wy y y y y y y y w

MUST SATISFY THE JUDGE THAT ITS NOT MANIFESTLY UNREASONABLE. Counterargumenttoo broad and therefore unenforceable. Albert thinks this was too broad. This is a close case could go either way, maybe for policy reasonable.  Broad completely eliminates theDOL SECTION 5. DISSOLUTION y Recall when we talked about events of dissolution in the partnership context; whether partnership will close up and go away or will reconstitute itself y This looks more like the corporate context y Events of Dissolution By operation of law:  Upon the happening of any event specified in LLC operating agreement  Vote of members (as specified in operating agreement)  It becomes unlawful to carry on the business Upon court order:  Economic purpose frustrated  Misconduct by members New Horizons Supply Cooperative v. Haack (Wis.App. 1999)p. 323-27 y Set up like an LLC. y Articles of Organization never filed. Since she didnt produce it, it was deemed not to exist. y Kickapoo goes out of business and it owes money to New Horizons. y They keep calling and trying to get their money. Haack, one of the LLC members, keeps saying that payment of $100/month will start. It never does. y Agreement signed by Haack in her own name. No designation so it creates impression of personal liability. y Haack says she will personally pay the $100/month. y She had also said that there were some assets of the business: a truck, secured by the bank and some accounts receivable that they were trying to collect. y The LLC is dissolved. Under Wisconsin law, dont have to file Articles of Dissolution. Key is that letting the creditors know serves the same purpose of giving the creditors notice. That way the creditors can make sure there are enough assets when it is their turn. Puts the creditors on alert that they need to take steps to get their money. y The trial court screws up a lot of bus. org. law. y This is an LLC and if done correctly, she will have no personal liability. y Purpose of this case is to figure out what happens to the assets when an LLC dissolves. y As per the statute, she will liable personally for the lesser of her share of the assets or the creditors claim. What is her share?  $500 If there are assets that belong to the entity that you took, your limited liability is now open for question. We are going to hold you responsible for you amount you took up to your share of the debt.  HYPO: If they each put in $2,000. If on dissolution, she takes out $500 on dissolution. She will have to give back that $500 up to her share which is 50%. Will have to give back the assets up to the amount you would be responsible for the debt.  This is only if you take assets without telling the creditors. y See another circumstance where an LLC owner can by his or her conduct incur some liability Not dipping into personal liability; essentially just liable for what they took up to their share  Done in a way in which your responsibility to give back to the assets up to your proportionate share of the assets 75

Note: Big Essay will not be on LLCs Odds are agency/partnership for the essay Also, duties of care which we will discuss next

DUTY OF CARE

Two W of Thinking bout the Business Judgment Rule




As standard of liability to measure D&O onduct


No liability for negligence Instead liability based on things like:


As an abstention doctrine for the courts Court will not review BoD decision if: Preconditions:
 


 

Fraud Illegal conduct Self-dealing

No fraud No illegality No self-dealing

y y y

y y

Think of it as a defense you can raise when you have NOT misbehaved; have exercised your duty of loyalty/duty of care Rebuttable presumptionone way to rebut is by showing breach of duty of loyalty and/or duty of care What is the duty of care? MBCA 8.30(a):Each member of the board of directors, when discharging the duties of a director, shall act: (1) in good faith, and (2) in a manner the director reasonably believes to be in the best interests of the corporation Can illustrate this concept best in the absence What would a reasonable director do in the same circumstances? What is the duty of loyalty? Directors and officers have a fiduciary duty to put the interests of the corporation ahead of their own. See issues involving conflict of intereste.g., someone on both sides of the transaction creates a duty of loyalty issue Before client seems to invoke the BJR rebuttable presumption that client better be sure he or she committed no breach of either of these duties; defense doesnt work if there is a breach Why doesnt the BJR offer protection for an uninformed decision? Duty of care is triggered by an uninformed decision If you breach duty of care, your decision isnt entitled to protection We dont know what is going to happen The people who are going to be making these decision were put in officers by the shareholders who have a stake in the corporation; presumably they did so b/c they figured these men and women knew about the business and were smart and so on Since we dont know the future, something that seems like a good decision, may turn out to be a bad one As long as you can show that when you made decision you were not conflicted, you made an informed decision, you didnt breach your duties, etc., we will provide you w/ the protection of BJR 76

These individuals are in the best position to make decision for the company so we will protect their informed decisions Who has burden of proof on this issue? BoP on plaintiff Will sue derivatively, the Board will answer w/ the BJR and it shifts back to you to show the Board wasnt shielded by the BJR rule What must that party prove? Will have to prove for breach of duty of care: directors must avail themselves of all reasonably available material information What is the result of an uninformed decision by a BOD is the decision void or voidable? They capable of being saved by the shareholders

Kamin v. American Express (N.Y. 1st Dept. 1976)p. 310 Sued b/c of corporate waste of assets which comes from the duty of careidea is that the corporate spent so much on something that the decision to spend that much could not have been carefully thought out Amount paying when compared to what you are getting is so disparate that you couldnt have expressed your duty of due care y Typical for corporations to have investmentshere, AMEX buys DLJ stock y 2 Options for AMEX: Could have sold the stocks at a loss (would incur a capital loss of $25 million)  But, could have gotten a tax credit for their bad investments and that would decrease the tax liability on their good investments (gains) Could have declared a special dividend to all stockholders of record pursuant to which the shares of DLJ are distributed in kind  Property dividendb/c not cash Some companies give other things like turkey, food, wine, etc.  Downside is not only does the co. not save taxes; this is taxable to the shareholder  If AMEX had taken the hit instead, the value of the stockholders shares might have increased, but it wouldnt have been a taxable event for the shareholder unless and until that stockholder sold his shares y This is a derivative suit b/c something the co. did affects the shareholders It is not directwould be direct if everyone recd shares and he didnt for example y Complaint? If the company had sold the stock, it would have recognized a $26 million tax loss that could have been offset against its taxable income.  When it issued the DLJ stock as a dividend, it lost that tax benefit.  The shareholders recognized dividend income equal to the fair market value ($4 million) of the DLJ stock, and got a basis in the DLJ stock equal to that amount. Plaintiffs calculated the cost of the companys scheme at $8 million. y He wants rescission Would have been easier if they had moved for a TRO They decided preliminary relief was moot; thing about preliminary relief is that you have to do it BEFORE y Any DOL issues here? Have to put the companies interests ahead of their own It deals w/ an individual director vs. the company These directors decided to give the stocks to the shareholders and give up the tax savingsdirectors put themselves ahead of the company Have to show:  If own the DLJ stock, would have a conflict of interest 77
y

y y

If they dont own the DLJ, then could show their compensation is based on how well the company performs; thus, if the co. lost money their compensation would go down Not taking the loss leaves the companys income higher and thus, their compensation as well Any evidence of self-interest here?  Ct. says only hint of self-interest is that 4 of the 20 directors were officers & employees of AmEx & members of its Exec. Comp. Plan.  So it was suggested by virtue of their action taken earnings may have been overstated & their compensation affected thereby  But the ct. found such a claim highly speculative & standing alone could hardly be regarded as sufficient to support an inference of self-dealing Any DOC issues here? Did the P pose the issue to the Board?  Yes, made a demand and the Board rejected it after one day during which they investigated it  Board had a special meetingmeeting was outside the ordinary course Court decides that they honored their duty of carebad business decision doesnt automatically mean they breached their duty of care Decided it was based on market value consideration for the stock in general which is a relatively solid business decision  So when evaluating this decision w/ the BJR, they decide the presumption has not been rebutted What standard does the court adopt for the DOC of directors? the court will not interfere unless the powers have been illegally or unconscientiously executed; or unless it be made to appear that the acts were fraudulent or collusive, and destructive of the rights of the stockholders. Mere errors of judgment are not sufficient grounds for equity interference, for the powers of those entrusted with corporate management are largely discretionary. The Court will not interfere unless a clear case is made out of fraud, oppression, arbitrary action, or breach of trust. No evidence of any of this here; they had 2 options and decided to go with one They had a full meeting and determined the potential results of each decision and came up with a goodfaith decision to go w/ a certain approach P is basically suing b/c they didnt take his idea and that is NOT a viable lawsuit Any allegations of fraud, self-dealing, bad faith or oppressive conduct here? Noct. explicitly says there is not Didnt the BOD understand this option of selling the shares and then using the tax loss? Did the BOD even consider the plaintiffs suggestions? Court states:  The minutes of the special meeting indicate that the defendants were fully aware that a sale rather than a distribution of the DLJ shares might result in the realization of a substantial income tax saving. Nevertheless, they concluded that there were countervailing considerations primarily with respect to the adverse effect such a sale, realizing a loss of $25 million, would have on the net income figures in the American Express financial statement. Such a reduction of net income would have a serious effect on the market value of the publicly traded American Express stock. What if defendant directors had failed to inform themselves about the adverse tax consequences? Ct. seems to apply it would be different if the defendant directors totally overlooked the facts called to their attention It would have to go beyond a mere error in judgment Is the BOD decision whether to declare a dividend open to debate? Can only declare a dividend if it makes sense There are solvency tests in every state statute which says what you have to have to declare a dividend and what you have to have after the dividend is done 78

Thus, there are times when you cant declare a dividend even when you want to since you dont have enough money to do so; cant bankrupt the co. over a dividend First, question is whether the BOD can do it  Can do it if statutorily permitted  If solvency test is satisfied Second, ask should you declare a dividend  Shareholders can get value in the form of dividends or appreciation  There are definite choices a co. can make and there are case-by-case decisions  Is this open to shareholder discussion and should it be? The BOD is in the best position to make the decision The decision to declare a dividend is NEVER open to shareholder discussion Otherwise, it would be a usurpation of 141(a) Thus, a decision to declare a dividend is ALWAYS w/in the discretion of the Board Could sue if a dividend was declared and not paid, but cant sue about whether one should be declaredthat is the definition of the business judgment the Board is the appropriate entity to make the determination Strong Abstention Version of BJR Court says: A complaint which alleges merely that some course of action other than that pursued by the Board of Directors would have been more advantageous gives rise to no cognizable cause of action. Courts have more than enough to do in adjudicating legal rights and devising remedies for wrongs. The directors room rather than the courtroom is the appropriate forum for thrashing out purely business questions which will have an impact on profits, market prices, competitive situations, or tax advantages.  Ct. will not get into second guessing business judgment decisions of Boards Most objective observers would probably conclude that what the directors of American Express did was dumb. So why was there no cause of action?  Dumb is ok but wholly lacking business purpose is not

Smith v. Van Gorkom (Del. Sup. Ct. 1985)p. 332-42SEMINAL CASE ON FAILURE OF DUTY OF CARE y Trans Union is a holding co. y Van GorkomCPA and officer, CEO, and chairman of the Board; he was approaching mandatory retirement age [he is an attorney] y Tax benefits that companies have that they can use to offset certain kinds of income Bad news for Trans Union is that they had investment tax credits that could be used to offset income, but they didnt have the right kind of income In the hands of someone else who had the right kind of income, they could have tax savings That decreases the value of the stock to less than it would be had they had the right kind of income

The Deal: Legal Structure


Pritzker Entities

Trans Uni n

Timeline 79

Jay Pritzker

Marmon Group, Inc.

Ne wCo, Inc.

12

TU stock ~ $38 (probably didnt reflect the true value of the co.)  Sept. 5: CFO Romans runs feasibility study not a valuation study- difference? Feasibility study says lets say the # is $55 and looks to see what the end conclusion is w/ respect to a deal with that dollar amt., will that work while a valuation study says what is the # (price) [value of company divided by price] j So one, the feasibility study, gives us an opinion as to whether a price is viable Could run at a number of different prices and get different outcomes j The other, the valuation study, gives us a price Just b/c Van Gorkom says he would take $55 doesnt mean thats the best they could go I feel like $50 is too low and $60 is too high is not a valuation study He invents the $55 Need the process to get you to the number instead of the number getting you through the process  Before Sept. 13: VG meets with TU Controller Carl Peterson  Sept 13-19: Van Gorkom approaches Pritzker Sept 20: Senior management meeting  Reaction was very negative Sept 20: BOD meeting  Board was equally unimpressed  He makes a 20 min. presentation to the Board  He is a CPA, but he isnt wearing that hat  Tells everything he knows about $55, but no one asks where that $55 is from  Copies of the proposed merger agreement were delivered too late for study before or during the meetinghow can you vote on an agreement that doesnt appear until after that meeting? That is a violation of the duty of care They didnt even have a proposed summary!  After the presentation, they only deliberated for 2 hours!  Lock-Up Agreement-locks up that number of shares for the bidder If he doesnt take over the co. b/c someone outbids him he is protected b/c he gets the spread btwn. $38 and $55 on the million years  Ability to shop around for a better offer: 90-Day Market Test j Would validate a decision by the Board that $55 was a fair price j Nominally a great idea j But fact that market did not come back w/ a greater price does not validate the $55 just shows that the valuation was faulty Oct 1: First public wrinkle: within 10 days of the press release, key executive officers threatened to resign.  They persuade the dissidents to remain on the Co. payroll at least 6 months after consummation of the merger.  Important that the team be on board. VG meets with Pritzker who agrees to some modifications as long as VG could convince the dissidents to stay on the TU payroll for 6 months after the deal closed Oct 8: TU BoD approves revised deal VG gets the senior management team calmed down and back in favor of the deal; they agree to hire Salomon Bros. to solicit other offers for TU, and they approved the proposed amendments to the Merger Agreement which had not yet even been drafted. Oct 10: amendment to merger agreement delivered to VG and signed by VG, apparently without reading them Dec. 19: this litigation was filed, and within 4 weeks, 8 of the 10 BOD were deposed Jan. 26: BOD has lengthy meeting and votes to proceed with the merger Jan. 27: BOD approves supplement to proxy statement detailing the merger 80

When you want your shareholders to vote on something you send them an information statement Reqd information proxy statement here did not explain the $55 Feb 10: TU shareholders approve merger by 69.9% to 7.25% (22.85% abstained) What Happened to the Old Trans Union Shareholders? They have Trans Union stock that gets turned into cash and goes away A merger is where a co. turns into and is swallowed up by an existing co. DGCL 251(b)(5):  The plan of merger shall specify the manner of converting the shares of each of the constituent corporations into cash securities of any other corporation or entity which the holders of such shares are to receive in exchange for [their] shares
What Happened to the Old Trans Union Shareholders?

$38

$55
19

What does the court look at first? The application of the BJR to the 9/20 BOD meeting  Was the business judgment made by BOD informed? No b/c they did not adequately inform themselves of the price No valuation study on $55 No expert That is not an informed decisionit violates the duty of care and these directors are subject to liability  Who has the burden of proof BoP on plaintiff to show breach Easy enough to doget the director on the stand and ask questions Were the plaintiffs here able to show this?  Able to show breach of duty of care fairly easily Ct. will reach the decision that Board would have reached had they not been conflicted  Will back out of the true number that the co. was valued at as of the day of merger and will give them the difference The court decides BOD failed to make an informed decision at its 9/20 BOD meeting about:  VGs role forcing the sale and creating the share price;  the intrinsic value of TU the company they sat on the board of, some for many years What factors helped the court to reach this decision?  Failed to investigate the offer adequately and consider any other options  Didnt have a valuation study, had a feasibility study  2 hours meeting  Market valuation testdidnt make any difference  NO DOCUMENTATION AT ALL  Shareholders approved deal based on proxy statement that breached DOC What relevance, if any, does the court find for the fact that the shareholders approved the board action? 81

 Court says they didnt have all the information  Would have been different if they approved it after knowing the $55 was unsupported What about defendants arguments?  What about defendants argument that the substantial premium over the market price makes their decision an informed decision? Why is this alone insufficient? j B/c it is a better deal than $38 doesnt automatically mean it is the highest deal j $55 might not have been a premium at all in the hands of someone can use the tax credits j If the stock was that undervalued, it may not have been a premium at all  What about defendants argument that the market test confirms the reasonableness of their decision to accept the proposal? The constraints on it did not allow it test the market  What about the defendants argument that their business savvy should be sufficient to save them from liability? Fact that relied on premium that might not in fact have been one and market test that doesnt actually test the market undermines the statement that they are geniuses about Trans Union Even though they did do a lot and came pretty close; need more to back it up What does the court look at next? The application of the BJR to the BOD conduct after the 9/20 BOD meeting  Was their conduct [in approving the amendments] informed and not grossly negligent; and if so, could it cure the 9/20 violations of the business judgment rule? The amendments could have fixed it if it was based on a valuation of $55 Would still have had a breach of duty of care on the first meeting but the damages would have gone away; proxy statement would have validated the price But they didnt have the amendments in time and read them properly Next The court looks at the issue of shareholder approval of the merger:  Was the proxy material fair and complete enough to classify the stockholders as fully informed? Not reqd to a do a valuation study but reqd to tell the shareholders what they need to make the decision What about 141(e) A member of the BOD or a member of any committee designated by the BOD shall in the performance of such members duties be fully protected in relying in good faith upon the records of the corporation and upon such information, opinions, reports or statements presented to the corporation by any of the corporations officers or employees, or committees of the BOD, or by any other person as to matters that the person reasonably believes are within such other persons professional or expert competence and who has been selected with reasonable care by or on behalf of the corporation.  If the records turn out to be wrong you are not going to have liability if you rely on the records in good-faith  Allows you do discharge your duty of care by hiring an expert to opine on their area of expertise How does 141(e) figure into all this?  Was Van Gorkom opining in his CPA capacities? No, he was opining as a CEO So the Board cant rely on him as a 141(e) expert What constitutes a report for 141(e) and what reports were made here? One very serious problem is that Van Gorkom himself was uninformed. Suppose that instead of himself briefing the board of directors, Van Gorkom had the corporations lawyer brief the board. Would the board be protected under 141(e)in that case? 82

Now we have an expert who was presumably hired w/ reasonable care. He must opine w/in his expertisehe is an expert in the law and not in this area so he cannot constitute a 141(e) expert Legal standard from the Smith v. Van Gorkom What do BOD need to show to be informed?  Directors need to gather all material information reasonably available to them: By material, we essentially mean a big deal Evaluating the Standard  The reasonably available standard requires an in-depth study: Valuation study j This the fiduciary duty to the shareholders to get them the best price Discussion of course of the negotiations j Here, they only had a couple of short meetings Review of actual contract j Deal was not memorialized in writing Information about the terms of the offer and their fairness j Had no data on the fairness of the offer; putting the $55 in context Thus, here, the Board failed to avail themselves of all material information reasonably available to them ALI Principles of Corporate Governance 4.01(c)(2): The ALI standard only requires directors to be informed to the extent that they reasonably believe to be appropriate under the circumstances How does the ALI standard differ from the courts standard?  The ALI approach permits directors to make decisions on less than all reasonable available information, if they reasonably believe doing so is appropriate given the situation.  Gives more wiggle room  However, wouldnt probably change the result in Van Gorkom b/c they probably cant even meet this standard What does Cinerama v. Technicolor Inc.(p. 342-44)add to this jurisprudence? The facts seem similar to Van Gorkom, but here the defendants had an easier time since their CEO had done a thorough job investigating the deal, had bargained hard and gotten the share price raised, and had hired experts who did a thorough job in support of the fairness of the deal for Technicolor. Plaintiffs were shareholders who opposed the merger of Technicolor into a subsidiary of MacAndrews and Forbes, and perfected their appraisal rights. While they were pursuing their appraisal remedy, they learned facts that led them to file for rescission.  Ps could have gotten a preliminary injunction if the deal had not gone through yet, they could sue to rescind, or they could opt out individually (protect appraisal right)this is a statutory right given to corporate shareholders that allows them to tell the judge that they are not getting a good deal  Here, they ultimately file for rescission but they had exercised their statutory right to opt out They were ultimately unsuccessful lower court held that since the price was fair, there was no harm, no foul. Up and down on appeal, ultimately affirmed- and we get an example of the entire fairness doctrine. Entire fairness from Cinerama Will use this standard to evaluate certain kinds of breaches that are nonetheless fair transactions Entire fairness:  Timing  Initiation  Negotiation  Structure of the transaction 83

 

 Disclosure to and approval by BOD  Disclosure to and approval by the shareholders. If you can satisfy the entire fairness standard, the burden shifts back to the D who must show the entire fairness of the conflicted transaction; if he cant, his breach is a big deal and he is done; if he can, then the breach isnt a big deal If the transaction is otherwise fair and reasonable, it cleanses the taint of the breach of duty y Question 2-p. 344 He was approaching retirement so he may have had different goals than the rest of the shareholders; he is not a good representative He may be more interested in cashing out than someone who is a little bit younger Fair price by definition has to be the highest in price and in this case they missed it by a couple of bucks Francis v. United Jersey Bank: The Players (N.J. 1981)p. 328 y Lillian Pritchard: Widow of Pritchard & Bairds (a reinsurance company) founder She is a shareholder Director, but:  Inactive  Listless  Drank rather heavily As a director, means she will take over fiduciary duties She knows nothing about the business  But when you agree to be a director of a co., you have to know what is going on y Pritchard boys: Charles Jr. & William Sons of founder Active in management; dominant figures Father had said Charles would take the shirt off my back Systematically embezzled large sums in form of nominal loans y United Jersey Bank Plaintiff Trustee in bankruptcy for Pritchard & Baird Represents creditors: y Whats the big deal with a couple of loans?

Sample Balance Sheet


Assets
Cash Accounts receivable Land Total assets
Assume Pri rd boys take $20 out as salary what happens?

Sample Balance Sheet


s o n
$ 50 150 $200 $100 40 $140 $340
60

Lia ilities
$ 40 100 200 $340 Accounts payable Notes payable

Cash Accounts rece vab e Land Tota assets


Res lt s l ryecrease i cas and s r l s

$ 20 100 200

Accounts a ab e otes a ab e

Shareholders Equity
Capital Surplus Total liabilities and owners equity

$320 Shareholders Equity Ca ta $100 ur us 20 $120 Tota ab ties and owners equity $320

84

o rq o rq

no n n o o

oq t o nq

u| u z y x x wv u

Assets

Liabilities
$ 50 150 $200

mlkj

61

Sample Balance Sheet


Cash ccounts receivable Land $ 40 100 200

Sample alance Sheet


Assets

otal assets

Total assets Result of lo ecrease i cash, i crease i loansboth assets

$340

62

y y

y y y

All they did was document their embezzlements by characterizing them as loans The Litigation Is Pritchard automatically liable, because BJR doesnt apply?  No. Plaintiff still has to show that Pritchard breached her duty of care. Did Pritchard breach her duty of care?  Yes inattentive facts that she was old, depressed, drunk, and ignorant of business were no defense What did Court Expect? Directors have a statutory responsibility to manage the corporation Directors cant give a proxy Duty to be informed:  Obligation of basic knowledge and supervision  Read and understand financial statements  Object to misconduct and, if necessary, resign She could have avoided liability if she had never accepted the director role Her estate is liable for her lack in following the duty of care; her estate is available to the creditors Sons dont get sued b/c they wouldnt have been lucrative Ds; they had no money left New Federal Legislation Post-Enron scandal, Congress passed Sarbanes-Oxley act, which:  Makes it easier to prosecute securities fraud, particularly financial fraud.  Imposes greater responsibility on senior management and directors, particularly independent directors and audit committee members, by requiring them to take a substantially more proactive role in overseeing and monitoring the financial reporting process, including disclosure and reporting systems and internal controls  Does not purport to change the common law duty of care, but increases civil and criminal enforcement authority over the conduct of corporate officers and directors, No question that potential civil liability for directors will be greater after Sarbanes-Oxley Statute articulates what directors need to do to exercise their duty of care and provides penalties for failure to do so  Dont change the duty of care just articulates it and adds in statutory penalties Sarbanes-Oxley Audit Committee Requirements Section 301 of Sarbanes-Oxley orders SEC to adopt rules mandating that:  At least one member of audit committee must be a financial expert  The audit committee shall receive reports from the independent auditors regarding critical accounting polices and practices, discussions that have taken place with management regarding alternative treatments of financial information under GAAP, and any accounting disagreements and other material written communications between the auditors and management

85

Ass itc ys t k $20 s l s?

t t

Capital Surplus

Total liabilities and owners equity

$340 Shareholders Equity Capital $100 Surplus 40 $140 otal liabilities and o ners equity $340

 }

Assets

Liabilities
ccounts payable otes payable $ 50 150 $200

Li
$ 20 100 20 200

Cash Accounts r ceivable Loan Land

Accounts payable Notes payable

Sharehol ers

ilities
$ 50 150 $200

it

$100 40 $140
$340
63

The audit committee must establish procedures to receive and address complaints regarding accounting, internal control and audit issues, and to provide company employees an opportunity to make confidential, anonymous submissions regarding accounting and auditing matters

SECTION 2. DUTY OF LOYALTY A. DIRECTORS AND MANAGERS

Direct intere ted direct r transactions


ACME Corporation

Indirect interested director transactions


Susie Smith an Acme director

contract Susie Smith


an Acme director

AJAX, n .

DOL conflict of interests transactions History: In the late 19th century, any shareholder could void ANY contract between a director and the firm, regardless of the fairness of the contract or ratification. It didnt matter if it was a good deal for the corporation or not, and it didnt matter if the interested director approved the contract or not. Benefits of the old rule: simplicity and easy to apply; shareholders faced smaller risks that directors would defraud the corporation Drawbacks to old rule: corporation often lost advantageous deals they could otherwise have made with a director. Two common fact patterns for DOL breaches:  D or O enters into a contract with the corporation;  D or O learns of a business opportunity that may be of use to the corporation, but they take it for themselves Corporate Opportunities DoctrineExample of violation of Duty of Loyalty Why doesnt the BJR apply to conflict of interest transactions?  B/c these are not the kind of directors that we are going to presume are making good decisions  Just will mean that now they have to explain themselves; doesnt necessarily mean liability Sarbanes-Oxley Act In 2002, Congress passed the federal Public Company Accounting Reform and Investor Protection Act (popularly known as the Sarbanes-Oxley Act). Section 402 of the Sarbanes-Oxley Act bars publicly held corporations registered with the SEC from directly or indirectly lending or arranging for the extension of credit to their own officers or directors. In addition, Sarbanes-Oxley 406 requires a corporation to adopt a code of ethics applicable to its CEO, CFO, controller, and chief accountant. If the company fails to do so, it must disclose that failure and explain its reasons for not adopting the required code. In order to pass muster, the ethics code must provide for the ethical handling of actual or apparent conflicts of interest between personal and professional relationships.

Bayer v. Beran (N.Y. Sup. Ct. 1944)p. 336 SHOWS THAT BREACH OF DOL CAN BE CLEANSED BY ENTIRE FAIRNESS DOCTRINE 86

contract ACME Corp.

Bayer v. Beran

spouses

Dr. Dreyfus

Jean Tennyson
9

y y y

Plaintiffs complaint about type of show: High-end artsy opera program Maybe plaintiff wants something that would appeal more to the masses Standard of review? BJR Again note abstention approach: it is not the function of a court of equity to review these matters or even to consider them Does the court talk about the BJR? Note the courts strong focus on balancing authority and accountability  To encourage freedom of action on the part of directors, or to put it another way, to discourage interference with the exercise of their free and independent judgment, there has grown up what is known as the business judgment rule. ... Indeed, although the concept of responsibility is firmly fixed in the law, it is only in a most unusual and extraordinary case that directors are held liable for negligence in the absence of fraud, or improper motive, or personal interest. ... The business judgment rule, however, yields to the rule of undivided loyalty. This great rule of law is designed to avoid the possibility of fraud and to avoid the temptation of self interest. Who has the burden of proof re: breach of duty of loyalty? BoP on Defendant  Their dealings with the corporation are subjected to rigorous scrutiny and where any of their contract or engagements with the corporation are challenged, the burden is on the director not only to prove the good faith of the transaction but also to show its inherent fairness from the viewpoint of the corporation and those interested therein.Pepper v. Litton Were the decision to choose radio advertising, and the cost thereof subject to protection by BJR? It is a decision that the directors make based on their knowledge That is not something the court ought to be getting involved in And were the decisions undertaken in compliance with the BJR? Absolutely They did what they thought was best for the corporation  All these decisions were fine; w/ respect to every opera singer except for her there is no breach of the duty of care  W/ her we have an added level of scrutiny b/c of the conflict The corporate housekeeping wasnt really taken care of here Ct. wasnt that jazzed up about it b/c even though they didnt necessarily have the formal documentation they did have the level of awareness Just note that the corporate housekeeping was not exactly kosher Was it a breach of the DOL for the corporation to hire Miss Tennyson? Because of the conflict of interest, the business judgment rule does not apply. Hence, Dreyfus and the other directors bore the burden of proof on the issue of the fairness of the transaction. 87

y y

y y

y y

In other words,if P has shown a breach of the DOL the burden shifts to the D (the director) to show why the transaction is still inherently or intrinsically fair to the corporation (thus, P doesnt necessarily win)  Can be called the entire fairness or intrinsic fairness doctrine Did they meet that burden? Yes Why no breach of DOL? Tennyson did not get unreasonable pay The program was not designed to further her career The company obtained its moneys worth from the radio campaign Here, the breach of DOL was cleansed by the entire fairness of the transaction Plaintiffs argue that the transaction was ultra vires (illegal as beyond corporations power) because no formal board action. What does court say about that argument? One mandatory provision is the purpose of the corp., you have to say that To change purpose, would need to amend the Articles of Incorp. by bringing it to the shareholders So will draft purpose clause really generally so dont have to go to shareholders to do this and can get around the statute; thus, dont see too much of this litigation anymore Here, the corp. has the power to hire the CEOs wife, it just has to stand up to scrutiny that it is a good decisionmust show it is an arms-length transaction and otherwise fair and reasonable to the corp. Can one director bind the corporation? Think of the concept of agency law  When hire agent, give them the power to bind you in K The BOD can bind the corp.; for any given decision, when the corp. decides to enter into a K w/ anyone, will want an officer to sign the K Shareholders elect the directors who appoint the officers; thus, the officers are typically the agents of the corp. and can bind the corp. (e.g., CEO, VP, etc.) A majority of the BOD can bind the corp. in the sense that they will approve transactions  It would be unwieldy to always require the entire Board to sign a K, e.g., every time a coffee machine is purchased so instead officer will sign in a corp. capacity  Done by authorizing the agents Point is that one director cannot bind the corp. unless the BOD is one person which is unlikely So why doesnt the court care about that here? Ct. says Board was made up of executive officers who were all over this; form a corp. housekeeping view it is problematic, but they will look at the result instead of how they got there Did the board really need to formally ratify the conduct? Ct. says they accepted the benefits of the radio ads so in essence it was implied ratification What advice you would have given Dreyfus if he had asked you how to validate the hiring of his wife? Full disclosure and abstentiontell everyone what the K is and then shut your mouth Better, leave the room! Variant: Quorum Issue Will always be about Delaware law in this course Variation on Bayer: Assume a Delaware corporation with 5 directors: Dr. Dreyfus, plus Alice Adams, Bob Brown, Charlie Conners, and Ed Edmond. There are 1 million outstanding shares; each director owns 5,000. At a scheduled BOD meeting, only Dr. Dreyfus, Alice, and Ed show up to discuss and vote on the radio advertising idea. Do they have a quorum?  Have directors who are also shareholders which is very common  At the scheduled BOD meeting, they are all being directors; doesnt matter they are shareholde

88


y

If 3/5 show up at the meeting, what is your confidence level about their ability to make a decision? At what point would it become questionable that it is really critical mass? j Bottom line is that if all 5 showed up, 3/5 agreeing would carry the vote (would be a simple majority) unless the Articles of Incorp. state otherwise j So the quorom is the minimum # of directors you need present to have a valid meeting j Less than a majority is problematic b/c they couldnt get a resolution passed if there was one Problem w/ making the quorum higher is that it will be harder to get everyone in a room So, need at least a majority as a minimum unless provided otherwise This applies to the ENTIRE duration of the meeting, e.g., if someone goes to the bathroom and there is no longer a majority, thus, nothing happens as a matter of law until that person walks back in OR phones back in j So the key is BODIES in the room j Meeting freezes and nothing counts until you have the quorum back in the room or on the phone If you have a quorum, items come up for vote and the majority of those present need to approve it With a ten-person Board, 6 people have to show up, 4 must agree to pass the resolution if 6 show up Statutory Notice Provisionscant have a secret meeting even though you have a quorum there So a sub-set of the Board is able to pass resolutions Here, in the above hypo, we have critical mass so there is a good meeting. There was also statutory notice since this was a scheduled meeting. Doesnt mean anything will get passed, but it is a valid meeting. 141(a): Total number of disinterested directors

Quorum statutes What is a quorum?  141(b) . . . A majority of the total number of directors shall constitute a quorum for the transaction of business unless the certificate of incorporation or the bylaws require a greater number. . . . Does the fact that Dreyfus is one of the three directors present matter for quorum purposes?  144(b): Common or interested directors may be counted in determining the presence of a quorum . . . The Idea of a Quorum in Context: 141(b)tells us you have to have critical mass in the room (Always look for the 141(b) quorum first on one of these questions)  To have that you need half plus one (that is the minimum)  You can make that number bigger but you cannot make that smaller By present we mean in the room or on the phone Assume we have 10 members  The statute tells us we need six of them presentin person or on the phoneto have a good meeting  Before and after the vote a lot of stuff happens that doesnt actually trigger a vote  If during the course of the meeting, people come and go and you lose your quorume.g., person on phone gets disconnectedthat is not a valid meeting during that time 89

If during a meeting no vote takes place, you dont have to worry about a quorum We really look to the time of the vote to see if there was a quorum at that time and thus, a valid meeting  Items up for a vote will be interested or disinterested items (this is step 2characterize the items to be voted on) Interested meaning one or more of the director has some form of interest in the transaction being voted on; e.g., director of the co. w/ a fiduciary duty to shareholders of the company but she is financial interested in the other co. Need to do this for EACH transaction (evaluate whether directors are interested or disinterested) Wont say you cant do the deal b/c director is on both sides But IF you do it, we will double-check and make sure it is fair to the corp.  If it is a disinterested item: A simple majority works to pass an item j If have 6 people in the room, need 4 to pass it  On any item where a director or more than one director has an interest, we take a different approach Lets say 7/10 directors are interested on a particular transaction; 3 are disinterested Still need 6 people in the room How many of the disinterested directors do we need to say yes to do this deal? j So if 3 disinterested show up and 2 of them say yes, then it will pass j Need the affirmative vote of the majority of the total number of disinterested directors even if that majority is less than a quorum j Every disinterested director that votes is subject to liability for that vote j If the disinterested directors are not in the room, it cannot be approved If you dont have any disinterested you cant use (a)(1); same if you only have one disinterested Voting Statutes: Do you See the Difference? 141(b): The vote of the majority of the directors presentat a meeting at which a quorum is present shall be the act of the board of directors unless the certificate of incorporation or the bylaws shall require a vote of a greater number. 144(a)(1): the board or committee in good faith authorizes the contract or transaction by the affirmative votes of a majority of the disinterested directors, even though the disinterested directors be less than a quorum Variant: Vote Okay, now Alice and Ed vote for transaction. Dr. Dreyfus abstains. Has it been approved?  So have a 141(b) quorum b/c 3/5 directors are present. This is a majority out of those present so this has been approved by a valid vote. Okay, this time all 5 directors are present in person or by phone  Dr. Dreyfus, Alice, Bob, and Ed vote for transaction. Charlie abstains. Has it been approved? Have a 141(b) quorum since all are present. 4/5 voted for it which amounts to a majority of those present so you have a valid vote. Ratification What is the effect of ratification per 144(a)(1) or 144(a)(2)?  Allows us to take a conflicted transaction and cleanse it  Allows the disinterested directors to weigh in and cleanse the taint  144(a): No contract or transaction between a corporation and 1 or more of its directors or officers ... shall be void or voidable solely for this reason, or solely because the director or officer is present at or participates in the meeting of the board or committee which authorizes the contract or transaction, or solely because any such directors or officers votes are counted for such purpose, if: 90

 

(a)(1): approved by a majority of the disinterested directors Really means the total number of disinterested directors So if there are 5 members, 3 interested, need the remaining 2 interested to say yes to have that majority Can see that something smaller than a majority of the Board can suffice (a)(2): approved by a majority of the shareholders  Thus, there are 3 ways to take a transaction in Delaware and cleanse it (3rd way is discussed below)  Unlikely 1 would be true and 3 would not be; also unlikely 2 would be true and 3 would not be  Will be circumstances where you cant satisfy 1 or 2, then will go to (a)(3)this is more complicated than 1 or 2 so will only do it if you have to Any differences between MBCA 8.61(b) and 144(a)(1)? Has the same concept The MBCA says a majority of the disinterested shareholders which has to be more than 1 And what about 144(a)(3)? What would be an example of situation in which it would be relevant? Only going this route if nothing else works 144(a)(3) Issues  Hallmarks of a fair transaction? Within the range of terms that parties bargaining at arms-length might reach Defects in disclosure highly relevant, however j If undisclosed facts had been known, firm might have gotten a better deal even if terms were in the range of fairness j This can really bare on whether a 141(a)(2) shareholder vote is really valid 144(a)(3) Issues in Bayer  Nothing to show that some other soprano would have enhanced the program  No suggestion that the present programs cost is disproportionate  Tennysons contract was on a standard form, negotiated through her professional agent  Her compensation was in conformity with that paid for comparable work She received less than any of the other artists on the program  Although she appeared with greater regularity than any other, she received no undue prominence  Subordinated to the advertisement of the companys products Exam Answer to the Question Is the vote Binding?: We have to look at two things 1) 141b and 2) whether it meets 144a. [do IRAC for 141bexplain why need to do 144a (interested transaction breach of the DOLwife is singer)then do IRAC for 144a1] Step 1. 141(b) a maj of total number of directors be in the room Step 2. This is a conflict transaction becausewife is Dreyfus wife and therefore, hes a not a disinterested dirctor. 3. Step 3: Majority of disinterested directors = . Here we only had 2. Thus, nonbinding 4. Step 4: how could we fix it? Call another disinterested director and get his vote. How it Works: 1. P alleges conflict 2. Burden shifts to corp to do one of three things (144a1-3) [see below] 3. If they can do any of the 3, they have met their burden of proof and BJR applies. Approaching a Conflicted Transaction: Step 2 1. 144a1: BOARD [Easiest Way] if you can show that a majority of disinterested directors approved a K, then the BJR should kick back in. 91 1. 2.

a. The material facts as to the director/officers relatonship or interest and as to the contract or transaction are known or disclosed to the board and the board in good faith authorizes the contract/transaction by the affirmative votes of a majority of disinterested directors whether or not its a quorum b. NOTE: if you have only one disinterested director, that is fine under the MBCA, but not Delaware law

2. 144a2: SHAREHOLDERS a. The material facts as to the director/officers relationship or interest and as to the contract or transaction are known or disclosed to the shareholdersand the contract/transaction is specifically approved in good faith by vote of the shareholders whether or not its a quorum b. NOTE: this doesnt say disinterested shareholders, but courts have read it in 3. 144a3: INHERENT FAIRNESS: contract or transaction fair at time it is authorized, ratified, apprd a. Hallmarks of a Fair Transaction: i. Within the range of terms that parties bargaining at arms-length might reach 1. Business as usual, no prior connection 2. Look at comparable offers ii. Defects in disclosure are highly relevant: if undisclosed facts had been known, the firm mightve gotten a better deal even if its terms were in the range of fairness Benihana of Tokyo v. Benihana (SEE SLIDES ON TWEN FOR MORE ON THIS CASE) y Benihana owns/operates y Aoki involved in insider trading o Transferred interest to a trust o Trustees were originally kids, changed it to go to his new wife instead o Interfamily struggle y 3 million shares of common outstanding shares (1 vote) y Class A Stock (1/10 of a vote) y Why did Benihana need the money? o Wanted to do renovations to the restaurants y Classified Board: Staggered group o Only once a year, part of the board will be re-elected o Why would you do this?  Creates stability y John Abdo: o Negotiates on behalf of BFC to possibly purchase preferred stock o Director, Vice Chairman of BFC, owns some stock y Investment Bank runs different scenarios o Abdo hears all of this o He does not hear anything super-confidential o How did the investment bank come up with their model?  Look at the company  Where do the scenarios come from? y Look at other companies that needed to finance y Abdo made presentation for BFC o Board book clearly stated that he worked for BFC y How did the stock wind up getting issued? o Lowered the trusts voting power y Unsuccessful in showing that Abdo used confidential info in negotiations o If he had, the result would have been different 92

o In reality, there was no conflict  Everyone knew he was on both sides  He did not use confidential info B. CORPORATE OPPORTUNITIES y Corporate Opportunities Doctrine: D&O cant take for themselves opportunities presented to them in theircorporate capacity that:  the corporation is financially able to take advantage of; Seems less egregious is you took something away that the corp. didnt have money for Looks like the beginning of a defense to the doctrine  are within the corporations line of business and is of practical advantage to it; More in line w/ what corp. does, more it looks like a corp. opportunity  is one in which the corporation has an interest or expectancy; and Interest: Something to which the firm has a better right Expectancy: takes something which, in the ordinary course of things, would come to the corporation j If officer bought land to which the corporation had a contractual right, the officer took an interest j If the officer took the renewal rights to a lease the corporation had, the officer took an expectancy  by embracing the opportunity the officer or director would create a conflict between his or her self-interest and that of the corporation. Braz v. Cellular Information Systems, Inc. (Del. 1996)p. 347 y Facts: Plaintiff: CISprovider of cell phone service Defendant: Broz, President and sole shareholder of RFBCanother provider of cell phone service; Broz was also an outside director of CIS, a competitor of RFBC and the plaintiff in this case  Clearly a huge conflict of interest for Broz Dispute arose b/c an FCC licensing area was offered to RFBC to purchase, but was not offered to CIS. Broz had purchased this license for RFBC. CIS had no interest in purchasing this license; they wouldnt have bought it even if it was offered to them for nothing An outsider would think this was precisely in their line of businessbut, they are moving out of the cellular industry so that changes things Broz never presented to the Board formally  All the CIS directors say if this was put for a voteit wasntI would have said no b/c they couldnt have and wouldnt have taken it  But, taking it to the Board can get rid of the bad faith of a decision PriCellularthey are tendering for CIS  Tender OfferOffer to buy shares of stock from shareholders, who are invited to tender their shares to the offeror for purchase at a specified price w/in some specified period of time.  Some contingencies can be built into it.  It is NOT a closed transaction.  They were bidding for the Michigan License against Broz in his individual capacity  They were actually close to bankruptcy themselves  Said if someone comes along who will pay more than X, they will get it Broz makes the bigger bid and takes it away from PriCellular  When we took the license, he has a problem w/ CIS because he is on their Board, but he has no problem w/ PriCellular b/c it is co. seeking to take over a co. whose Board he serves upon PriCellular sues for the taking of a corporate opportunity 93

y y

y y

y y

 By the time of suit they own CIS Held: No breach of the DOL. Under Broz, when can a D or O take an opportunity for himself? If the director or officer believes, based on one of the factors articulated above, that the corporation is not entitled to the opportunity, then he may take it for himself.USE 4-PRONG TEST The D or Os belief is CRITICAL. Corporate Opportunity Doctrine Are these factors or elements?  Its not entirely clear; the test uses and. The court calls them factors but then implies that absence of any one is enough to avoid liability  Some are more problematic than others  So you really need all of them for it to be a CO  Court must look at all info Does the fact that this opportunity came to Broz in his individual capacity end the analysis? That is not the end of the discussion It is complicated by the fact that he doesnt bring it to the corp. which he probably should have It is a better fact for him, but it not dispositive Is the purchase of this license a corporate opportunity under the 4-pronged test? CIS is not financially available to take this opportunity In terms of line of business; they were moving out of the cell industry No interest or expectancygetting out of that business No conflict b/c just on Board; showed up a few times for meetings What about the fact that he was competing with PriCellular, who was about to buy CIS? What is the relevance of BOD approval of the D&Os actions? Just provides a safe harbor Nice to get Helps you move through the factors What if PriCellular had no financing difficulties and could have invested enough money into CIS to buy the license. Same result? Closer PriCellular is to buying CIS, the bigger the conflict he has What if Rhodes the original broker did not distinguish between Brozs role at CIS and his role in RFBC? Why didnt PriCellular just outbid RFBC? Because they couldnt; they didnt have the money to do it

In re eBay, Inc. Shareholders Litigation (Del. Ch. 2004)page 352 y What is spinning? A practice that involves allocating shares of lucrative public offerings of stock to favored clients. y How many IPOs did Goldman allocate to eBay CEO Omidyar? At least 40 He sold them right away and made tons and tons of money y What is the defendants argument? Goldman Sachs IPO allocations to eBays insider directors were collateral investments opportunities that arose by virtue of the inside directors status as wealthy individuals. It was not a corporate opportunity w/in the corp.s line of business or an opportunity in which the corp. had an interest or expectancy. y Is this a corporate opportunity under Delaware law? Ct. decides that it isDs usurped a corp. opportunity to their co. & their IB helped Rule: Line of business may be interpreted broadly 94

Analysis:  (1) EBay was financially able to exploit the opportunities in question  (2) EBay was in the business of investing in securities. This was critical here We see a broadening of the line of business in this case  Facts alleged suggest that investing was integral to EBays cash management strategies and a significant part of its business  No answer to say as Ds do that IPOs are risky investments; undisputed that EBay was never given an opportunity to turn down the IPO allocations as too risky. See a definite conflict of interest here  They clearly invested in the shares (is this within definition of corporate opportunity? Yes  Right result? Notes on page 392 y In Beam ex rel. Martha Stewart Living Omnimedia v. Stewart (Del. Ch. 2003) affd (Del. 2004), the plaintiffs alleged that Martha usurped a corporate opportunity when she and her family partnership sold 3 million shares of MSO to ValueAct. y The argument was that Stewart and another BOD member breached their fiduciary duty of loyalty when they sold large blocks of MSO stock to ValueAct.
y

What additional gloss do we get on the 4 factor test set out in Broz? No single factor is dispositive. Instead the court must balance all factors as they apply to a particular case. On which of the 4 factors do you think the court in the Martha Stewart case relied on to conclude that there was no usurpation? Chancellor Chandler held that: (a) is the corporation is financially able to exploit the opportunity prong:should be analyzed in this context by asking whether MSO had enough authorized but unissued stock to satisfy the investor, which it did. (b) within the corporations line of business prong: an opportunity is within the corporations line of business if it is an activity as to which [the corporation] has fundamental knowledge, practical experience and ability to pursue. Chandler held: Simply stated, selling stock is not the same line of business as selling advice to homemakers. Further, I would presume that a companys line of business is one that is intended to be profitable. By definition, a companys issuance of its stock does not generate income. j How is this different than the situation in the eBay litigation? Here the contrast to eBay is particularly significant. In contrast to the sale of stock at issue in Beam, eBay apparently did intend to make a profit from investing in marketable securities. (c) Interest and expectancy prong: Chandler held that for MSO to have an interest or expectancy in the transaction there must be some tie between that property and the nature of the corporate business. In Beam, plaintiff does not allege any facts that would imply that MSO was in need of additional capital, seeking additional capital, or even remotely interested in finding new investors. In contrast, eBay apparently was looking for investments in marketable securities. (d) conflict of interest prong: The fourth prong of Broz asks whether, by taking the opportunity for his own, the corporate fiduciary will thereby be placed in a position inimical to his duties to the corporation. 95

In Beam, Chandler held that Delaware courts have recognized a policy that allows officers and directors of corporations to buy and sell shares of that corporation at will so long as they act in good faith.  When Stewart sold stock, she was getting money If company was selling stock, they would be getting investor  Ebay: If Ebay bought shares and flipped them, they would have income  Martha usurped the stock by taking away the opportunity of MSO to raise new capital by selling stock Suppose the independent members of eBays BOD had authorized the defendants to accept the allocated shares. What result on those facts? Broz makes clear that rejection of a corporate opportunity by the independent board creates a safe harbor. Presumably, by analogy to the case law under DGCL 144(a)(1), within this safe harbor, the standard of review is the business judgment rule and plaintiff must show that the rejection constituted waste. We like the Board to approve or disclaim interest in a CO as a safe harbor but we still love at the four prongs

ProblemPage 356 y Prong 1Nothing to tell us otherwise, so assume they are financially able y Prong 2Sense is that they the line of business seems different Zapco does video game software Software engineers do voice recognition y Prong 3Want to know what he was wearing Zapco shirt? Entire defense cannot be that they came to you in personal capacity If he came into the situation and said We need Zapco to market this, and he says no, they are too busy, I will do this. y Prong 4Want to look at where the interaction happens and in what capacity If they come up to him in the Zapco booth, it is harder to prove If he says I knew that they were not going to bring it to Zapco, so I took it myself. Not a good defense If he gets board approval, then he has a nice safe-harbor to rely on Example Suppose IBM enters into a merger agreement with Apple, and submits the merger to a shareholder vote. We would expect directors of both with significant conflicts of interest not to participate without disclosing their conflicts of interest.

y y

C. DOMINANT SHAREHOLDERS y When we have a shareholder who owns a lot of stock they start to pick up some fiduciary-like duties y When you have a shareholders who owns so much stock that he/she is deemed to be able to control the Board, you pick up some Board-like responsibilities to the minority shareholders y But is every shareholder who holds significant stock in both companies disqualified from voting, absent disclosure? Of course notgiven that many investors hold large stock portfolios, such a rule would be completely unworkable. y Nonetheless, courts do impose some fiduciary duties on some shareholders, for two closely related reasons.

96

1. controlling shareholders (majority shareholders and, sometimes, shareholders with less than a majority) can control the board. Because such a board will not act as an independent monitoring institution, courts may impose the boards fiduciary duties directly on the shareholders. So controlling shareholders owe fiduciary duties to the minority 2. Some corporate actions require a direct shareholder vote. Where a controlling shareholder uses that vote in a manner the court considers unfair, a court may hold that the shareholder violated fiduciary duties to the other shareholders.

Intrinsic fairness In general, transactions between a controlling shareholder and the corporation are subject to an intrinsic/entire fairness test. Under it, the controlling shareholder has the burden of proving that the transaction was fair to the corporation. Nonetheless, the test applies only when a potential for self dealing inheres in the arrangementwhen the controlling shareholder can receive something at the expense of the corporation (or the minority shareholders).

y y y y

Sinclair Oil Corp. v. Levien, Delaware (Del. 1971)p. 357 Parent-Subsidiarycorp. that owns shares of another corp. is a subsidiary Sinclair owned about 97% of Sivens stock. Sinclair nominates all members of Sivens BOD. The plaintiff was a minority shareholder in the company, and complained that (i) Sinclair caused Sinven to issue excessive dividends, (ii) Sinclair usurped Sinvens corporate opportunities [denial to Sinven of industrial development], and (iii) Sinclair allowed another Sinclair subsidiary, Sinclair International, to breach various contracts with Sinven. y What two standards does the court identify? Business judgment rule: BoP on plaintiff to rebut the BJR Intrinsic fairness:BoP on defendants to show transaction was fair to Sinven y Burden ALWAYS starts w/ the D to prove the breach Always start w/ the BJR, that is the defaut Unless there is a demonstrated conflict in which case you will talk about heightened scrutiny  Here it will be heightened scrutiny b/c there are Sinclair directors on both sides of the transaction. y How does court select standard of review? Intrinsic fairness is ONLY used when parent has received a benefit to the exclusion of the minority shareholders of the subsidiary and at the expense of the minority shareholders of the subsidiary  When a parent controls the transaction and fixes the terms, intrinsic fairness is triggered, and the burden shifts to the parent.  The test to apply intrinsic fairness is when P has received a benefit to the exclusion and at the expense of S this amounts to self-dealing [P on both sides of a transaction]. y BoP shifts to the Ds who are unable to show that the transaction is fair. Thus, the D has liability. y Sinclair Wins y Expansion Policy Issue: Opportunities outside of Venezuela  Did not use Venezuelan subsidiary to do deal  His agument is that Sinclair used SinVen exclusively in Venezuela and that is unfair  But this is within BJR: how you set up your business is under your own control Breach of K: 97

 

Sinclair owns 100% of Sinclair Intl Oil Company Parent advantages wholly owned subsidiary at detriment of partially owned P claims that D caused K to be breached j Lag of 30 days payment after receipt j Purchase of fixed min amt of crude/refined products from Sinven If Sinclair owned both, there is no cause of action j Here, we have the perfect scenario j Parent allowing wholly-owned subsidiary the ability to buy what it needs j Parent benefits at expense of minority shareholder j Self Dealing j Sinclair unable to satisfy intrinsic fairness
j

Stock Terminology: Common Stock  a type of stock that represents the RESIDUAL value of the corporation; common stock has no special contract rights or preferences, so creditors and preferred shareholders must receive their required interest and dividend payments before common stockholders get ANYTHING.  Common stock has no fixed maturity date; if the corporation is liquidated, the creditors and preferred shareholders are paid the value of their interests first, and common stockholders are paid the value of their interests, if any, last. Common stockholders have no personal liability UNLESS they are up to no good  Ordinarily, the common stockholders have the exclusive right to elect the board of directors who manage the corporation.  They may also get to vote on mergers and other important matters.  In return for their investment, common stockholders receive DIVIDENDS declared by the board of directors.  Common stock is frequently the only class of stock outstanding;  it generally represents the greatest proportion of the corporations capital structure and bears the greatest risk of loss should the enterprise fail.  But in return for this position of subordination, the common stockholders have an exclusive claim to the corporate earnings and assets that exceed the claims of creditors and other shareholders.  Therefore, the common stockholders bear the major risks of the corporate venture, yet stand to profit the most if it is successful. Preferred Stock  an equity security that is given certain preferences and rights with regard to assets or dividends over common stock. They have contractual rights superior to those of the common stock; it is preferred to the common-- it takes priority over the common stock in some way, usually either in terms of dividends or liquidation preferences or both. Liquidation Preference  the right to be paid before the common stockholders if the corporation is dissolved and liquidated. The liquidation preference is usually stated in a dollar amount.  Example: a corporation issues a stock that has a liquidation preference of $200. This means that if the corporation is dissolved and liquidated, the holder of each preferred share will receive at least $200 before the common stockholders will receive anything. Note that since the corporation must pay its creditors first, there may be insufficient funds to pay even this preference. Dividend Preference  the right to receive a fixed dividend at set periods during the year, like every quarter. The dividend rate is usually a set percentage of the initial offering price. 98

Example: if a stockholder purchases $10,000 of a preferred stock that pays an 8 % annual dividend. The stockholder has the right to receive $800 each year as a dividend on the preferred stock. Cumulative Dividend Right: corporations must pay a preferred dividend if they have the earnings to do so. Cumulative preferred stock provides that any missed dividend payments must be paid in the future to the preferred stockholders before the common stockholders can receive any dividends. Non Cumulative Preferred: dividends do not cumulate; the corporation does not have to pay any missed dividends. For these shares, only the current years dividends must be paid to preferred shareholders prior to the payment of dividends to common shareholders. Participation Preference  allows the stockholder to participate in the profits of the corporation, along with the common stockholders. Participation is in addition to the fixed dividend paid on preferred stock. The terms of the participation can vary widely. Usually, the common stockholders must be paid a certain amount of dividends before participation kicks in. Conversion Right  permits the stockholders to convert their shares into another security, typically, common stock. The terms and exchange rate of the conversion are established when the shares are first issued. The holders of the convertible preferred stock usually exercise this option if the corporations stock increases significantly in value. Redemption Right  permits the corporation to redeem/buy back the preferred stock at some future date. The terms of the redemption are established when the shares are first issued. Corporations usually redeem shares when the current interest rate falls below the dividend rate on the preferred stock.  This is the only feature we have looked at that makes preferred stock less valuable Zahn v. Transamerica Corp.(3d Cir. 1947)p. 361 y Transamerica owned a controlling block of the Class B shares and dominated the management of Axton Fisher. Transamerica had elected majority of the board. y Zahn owned Class A common stock in Axton-Fisher y Transamerica owned an interest in both y Dividend preference: Class A was entitled to a cumulative annual dividend of $3.20, then the Class B was entitled to $1.60 and then the two classes shared equally any excess dividend. y Redemption/conversion preference: company could call the Class A shares for redemption upon 60 days notice at $60 per share plus accumulated dividends, but Class A shareholders could convert each Class A share into one share of Class B instead. The B shares are not convertible or callable. y Liquidation preference: upon liquidation, the division of assets was 2:1 in favor of the Class A. y Voting rights: All the votes were in the class B except when dividends had not been paid for four successive quarters, then the classes of stock with the defaulted dividends pick up voting rights like class B (not an issue in the case). y Lets play with the numbers to see what was going on in this case: Assume that there is only one A share and one B share and that the amount available on liquidation is $60. What result?  In the absence of conversion, because the liquidation preference for division of assets is 2:1 in favor of the A share, the A share gets $40 and the B share gets $20. Now assume that the amount available on liquidation is $240.What result?  In the absence of conversion, the A share would get $160 and the B share would get $80. What happens if the corporation seeks to redeem the A share for $60?  If the directors issue the notice of redemption for the A share for $60 and the holder decides to take a B share instead, each will receive $120.  What is the purpose of this redemption provision? 99

y y

The redemption provision was included to benefit the B shareholders in situations in which the value of the company had risen to the point where redemption would force conversion. Assume there are 100 A shares and 100 B shares, but no preferred. The company has $6000 in cash and nothing else. Suppose the company liquidates. What will the A shares and B shares get?  The liquidation ratio is 2:1 in favor of the A shares, so the A shares will get $40 per share and the B shares will get $20 per share. Do the B shares have an incentive to cause the firm to redeem the A shares at $60 before liquidating?  No. Redeeming the 100 A shares for $60 would leave nothing for the B shares. So when will the B shares have an incentive to redeem the A shares?  When the company is doing well and has more than the $6000 cash on hand. Still assume there are 100 A shares and 100 B shares, but no preferred. If the company has cash of $30,000. What will the A shares and B shares get upon liquidation if the company does not redeem the A shares?  The A shares will get $200/share and the B shares get $100.  Do the B shares now have an incentive to redeem the A shares? Absolutely. If the company redeems the A shares for $60, the firm will be left with $30,000 ($60)(100) = $24,000, yielding $240 per B share. This is a short-hand account of what apparently happened in the case.  But if the company calls the A shares for redemption and the A shares know that the company has $30,000, what will they do? Using their 60-day notice period, theyll convert to B shares. Hence, there will be no A shares, 200 B shares, and each will get $150. So why didnt the A shares in Zahn convert? B/c they didnt know the value was underrepresented. But shouldnt the fact that the company called the stock necessarily have signaled to the A shares that they would be better off converting? Happens all the time and doesnt necessarily signal anything Tobacco asset Axton-Fisher possessed as its principal asset leaf-tobacco, which cost it about 6.3 million It is actually worth about 20 million,  Axton Fisher knew this  Transamerica knew this Call the stock (send letter to shareholders) that Class A is going away Does it make sense that shareholders who took cash knew about value of tobacco? Rule: The majority shareholder has the right to control, but when it does so it owes a fiduciary duty to the minority and to the corporation Holding: Transamerica was a controlling shareholder. Transamerica owed a fiduciary duty to AFT and to the minority stockholders of AFT. The corporation charter gave the BOD the right to call a redemption of the Class A stock, but in this case the BOD was controlled by Transamerica, which owned the Class B stock (the voting stock) and would benefit from a decision to redeem the Class A stock. The Board in therefore not disinterested. If all the allegations in the complaint are proved, Zahn may recover from Transamerica the value of his stock Example Suppose, for example, that the company is worth $10,000.  If it liquidates without redeeming the A shares, the A shares will get $66 and the B shares will get $33 because of the liquidation split. 100

y y

If the company redeems the A shares, the A shares will get $60 [assuming no accrued dividends] and the B shares will get $40.  If the A shares convert to B shares, the A shares and B shares will all get $50. Hence, in this example, the B shares have an incentive to have the A shares convert, but the A shares do not have an incentive to convert. So the As and Bs have different financial interests here. The Bs are dominant shareholders are definitely calling the shots and the Board determines when and if to call the shares. So a dominant shareholder cannot disadvantage a minority shareholder to benefit himself. D. RATIFICATION In Re Wheelabrator Technologies, Inc. Shareholders Litigation (Del. Ch. 1995)p. 370 y Waste Management owned 22% of WTI stock and had 4 directors on the WTI Board. Waste and WTI decided to merge. The Board voted unanimously to merge. Went through all the corporate housekeeping. A good meeting took place. The Board issued proxy statements to the WTI shareholders. WTI shareholders (not included Waste) voted to merge. y Wheelabrator issues: Dissident shareholders sued claiming: (i) that the proxy disclosure was inadequate (ii) that the directors violated their duty of care (iii) that the directors violated their duty of loyalty y DOC (i) Disclosure. The plaintiffs argued that the proxy material was materially misleading because it didnt disclose that the board meeting lasted only three hours.  Ct. held this argument lacked evidentiary support. The assertion made by P that the WTI Board could not have considered the merger proposal carefully rests upon an unsupported inference from one fact: the 3 hour length of the March 30, 1990 meeting.  Duration of the meeting is not necessarily dispositive. (ii) Duty of care. Even if (hypothetically) the board didnt exercise due care, the shareholders ratified the merger after full disclosure. Thats enough to kill the claim.  Even though they didnt, it was taken to the shareholders who approved this transaction based on the information in the proxy statement; unlike Van Gorkum, the shareholder vote was fully informed. y DOL (iii) Duty of Loyalty.Even if (hypothetically) the board violated its duty of loyalty in voting for the merger, ratification changes the BoP. But nothing is simple here. The court sets out a two-part standard: Does it matter whether the shareholders are interested or not? And what about the language of 144(a)(2) -- does it require DISINTERESTED shareholders?  No. The section provides that an interested transaction is not void if (a) there is full disclosure and the disinterested directors ratify it, (b) there is full disclosure and the shareholder ratify it, or (c) it is fair. The drafters inserted a requirement of disinterest in (a) but not in (b). Accordingly, ratification ought to be effective to shift the burden of proof of unfairness to the plaintiffs (under 144(a)), even if the shareholders are not disinterested; Courts have held otherwise. Although perhaps not faithful to the statutory language the decision does appeal to common sense. The Obligation of Good Faith Disney Case (page 376) y Shareholder derivative suit 101

y y y

Was Ovitzs firing a violation of duty of care or duty of loyalty and a breach of duty of good faith? Procedural Posture in this case is endless (litigation went on for 10+ years) This is after TransUnion where DE gives corporate world huge smackdown where board had to avail themselves to all information o This was a bit of DE saying that they are not terrible all the time How was Disney set up and why were they looking for Ovitz? o Wells died, and Eisner was there as CEO o Eisner is notoriously difficult to work with, but he is good at what he does  He had health conditions  They needed someone smart to learn the ropes in case Eisner left, got sick, died, etc.  Shareholders needed to know that the company would be in good hands o Ovitz was working at CAA, making $20 million + a lot of money on the table that he would e walking away from  He needed an incentive from Disney to walk away from this amount of money o Eisner and Irwin Russell, chairman of compensation committee, negotiated with Ovitz  Under 141 they can create a committee and delegate authority  Board members elected to committee by shareholders  Chairman of compensation committee part of negotiations y They had an expert in compensation, under 141(e) this is allowed  Terms of Ovitzs K y Only submitted to board the TERMS of the board, NOT the actual K y The Compensation Committee ceded some of the power back to the President y A 5 year term o This is risky: Ovitz had never run a studio in the past  But the Court said that he had lots of experience running other entertainment agencies and that other companies wanted him o This is an odd choice o Business Judgment Protection: it was made in good faith y A couple of options to end his K o If initial 5-year term expired and no renewal offered, he got $10 million in a termination provision  This is good as long as both parties are happy, the K will help the parties stay in place o They could fire him for cause  Cause would have to be very strong in the view of Ovitz (must have done something demonstrably wrong)  Corporation would want it to just be something basic y It is MUCH cheaper to fire someone for cause  If they fired him for cause, he probably would have FOUGHT y Shareholders argument: you should have fired him for cause o Violated duty of care: you didnt negotiate it properly; you should have investigated further o Ovitz was a habitual liar (not actionable) o They wound up getting rid of him without cause  To terminate him for cause or without cause is PROTECTED by BJR  This was a weird situation it might have been difficult to terminate him with cause this would drag Disney through litigation o The Disney Directors Conduct  Shareholders did NOT rebut the BJR 102

The Directors were not liable y Nice decision for our directors o The Claims Against Disney:  A due care and bad faith claim could be treated on separate grounds  Full Disney Board was not required to consider and approve the OEA y Disney had authority to give committee jurisdiction to handle all compensation issues y Authorized by the full Board approved this in advance  The compensation board did not do as good a job as they should have y The documentation would have been stronger if best practices were followed y Despite imperfections, evidentiary record said that compensation committee had generally informed itself about the situation o The options that Ovitz got were identical to options that Eisner got o They DID have presentations, spreadsheets, etc. Whats the big deal with good faith? y Only directors who act in good faith are entitled to indemnification for legal expenses under Del Section 145. y Only directors who rely in good faith on corporate records and experts can be relieved of liability under Del Section 141(e). y Only disinterested director and/or shareholder approval of interested director transactions can insulate interested director transactions from judicial review. So is good faith a predicate for BJR protection? And what are the categories of bad faith? y Subjective bad faith: conduct motivated by actual intent to do harm which clearly constituted bad faith y Lack of due care: does this rise to the level of bad faith? Its gross negligence without malevolent intent. This is NOT bad faith. y Intentional dereliction of duty: a conscious disregard for your responsibilities this is a non exculpable nonindemnifiable violation of the duty fiduciary duty to act in good faith, so its Bad Faith. Waste y A plaintiff who fails to rebut the BJR presumptions is not entitled to any remedy unless the transaction constitutes waste y [if] the exchange was so one sided that no business person of ordinary, sound judgment could conclude that the corporation has received adequate consideration. y The court tells us that the payment of a contractually obligated amount cannot constitute waste, unless the contractual obligation itself is wasteful.

OVERSIGHT (page 395) y Directors not expected to know in minute detail everything that happens on day-to-day basis o At very least, director must:  Have rudimentary understanding of firms business and how it works  Keep informed about firms activities  Engage in general monitoring of corporate affairs  Attend board meetings regularly  Routinely review financial statements o Caremark Standard:  Directors obligation: 103

Duty to attempt in good faith to assure that a corporate information and reporting system exists o Failure to do so under some circumstances may, in theory at least, render a director liable for losses caused by non-compliance with applicable legal standards o Stone v Ritter (page 396)  Sustained or systematic failure of board to exercise oversight will establish lack of good faith y Utter failure to attempt to assure reasonable information and reporting system exists  Corporation had to pay fine What is a classic Caremark claim? o When a director liability claim is predicated on ignorance of liability-creating activities within the corporation, only a sustained or systemic failure of the BOD to exercise oversight will establish the lack of good faith needed for liability. What is an example of such a sustained or systemic failure? o An utter failure to attempt to assure a reasonable information and reporting system exists.  Graham: absent cause for suspicion, there is not duty on the BOD to install and operate a corporate system of espionage to ferret out wrongdoing which they have no reason to suspect exists. Caremark: absent grounds to suspect deception, neither corporate boards nor senior officers can be charged with wrongdoing simply for assuming the integrity of employees and the honesty of their dealings on the companys behalf. y Why does this standard for demand futility fare poorly with respect the important class of cases in which plaintiff alleges that the board failed to exercise proper oversight? o BJR is inapplicable where the board did not exercise business judgment. y Does acting in bad faith trigger an independent basis of liability under Delaware corporate law? o No;although good faith may be described colloquially as part of a triad of fiduciary duties that includes the duties of care and loyalty, the obligation to act in good faith does not establish an independent fiduciary duty that stands on the same footing as the duties of care and loyalty. Only the latter two duties, where violated, may directly result in liability, whereas a failure to act in good faith may do so, but indirectly. y

SECTION 3. DISCLOSURE AND FAIRNESS y Threshold Question: Does what we are talking about fall w/in the definition of a security y Primary and secondary markets: Primary markets/issuer transactions:sale of securities to investors by issuers seeking to raise capital for their businesses. Private placements are the most common form of transaction. But these sales can be in public markets or through private placements in negotiated transactions. Secondary markets/trading transactions: buy-sale transactions among investors of already-issued securities. y Recall: 2 Ways to Buy-In: Ownerbuy stock Creditorbuy a bond y Function of securities markets: (1) capital formation: bringing together investors and businesses through the issuance of equity and debt; (2) liquidity: the ability to readily sell an investment instrument- providing an exit strategy; and (3) risk management:permits diversification and hedging of investments y Participants in the securities markets: 104

Investors:seek a return on their investment; they own securities, either directly or indirectly; the trend has been towards indirect ownership of securities by institutional investors.  What does it mean to own something indirectly? E.g., putting stock in your kids name or husbands name Owning stock like that might count Also owning a mutual fund is an example of indirectly owning stock Issuers of securities: make a financial commitment to provide a return in the form of (1) dividends, (2) interest payments, (3) appreciation and/or (4) liquidation rights; Financial intermediaries:bring issuers and investors together; they come in various forms:  Dealer: when a firm buys and sells securities for its own account, taking title itself.  Broker: when a firm buys and sells as an intermediary for a customer.  Market maker:a dealer who keeps an inventory of a particular companys securities and holds itself open as a willing buyer and seller of those securities.  Underwriter:when a firm helps a company sell securities through an offering registered under the 1933 Act Overview of securities law state and federal Securities regulation is designed to protect investors, whether they are buying securities from an issuer, trading in the securities markets, exercising their voting rights or selling in a tender offer. For the most part, securities regulation in the United States is a matter of federal law, based on the idea that: (1) mandatory disclosure [Make sure that investors have all the information they need to make informed decisions] and  All the SEC looks for is whether enough disclosure was provided (2) anti fraud liability will equip securities investors and securities markets with the information to move capital to its optimal uses.  Disclosure is meaningless if not true or materially misleading So the laws have an affirmative component, through disclosure obligations on issuers and insiders; and a negative component, with prohibitions via the anti fraud rules -- no material misrepresentations or omissions in connection with the sale of securities.  Misstatements are not terrible, material misstatements are Seven statutes:Only 2 matter for us Securities Act of 1933: regulates the sale of securities to the public primary market  When the co. sells stock to the world, it goes through this Act Securities Exchange Act of 1934: establishes the SEC to administer the securities laws and to regulate practices in the purchase and sale of securities secondary market  When the world sells stock to the rest of the world The SEC The 1934 Act establishes the SEC to administer the securities laws and to regulate practices in the purchase and sale of securities secondary market  Independent agency  Enforce the securities laws  Promulgate rules and regulations to implement those laws more effectively State blue sky laws Every state has one, but they are all slightly different The states had already begun to legislate in this area, before the market crash in 1929, with the first state blue sky law in 1911. The blue sky laws are named because they protect the investors from having nothing standing behind their investment besides water or blue sky. The various state blue sky laws vary, but all contain the following three ideas:

105

(1) anti fraud provisions: the states attorney general is empowered under the statute to investigate fraud in the advertisement, purchase or sale of a security and has the authority to undertake criminal prosecutions and get injunctive relief. Individual investors are entitled to sue civilly as well. (2) disclosure provisions:the statutes requires pre-approval of the offering circular or prospectus, all advertising and any other information in connection with the sale or purchase of securities (3) licensing and registration provisions for securities and brokers and dealers of securities:the laws provide for the registration of securities offered for sale in that state, and for all persons selling securities A. DEFINITION OF A SECURITY y Threshold question Is the interest in question a security for purposes of the 1933 or 1934 Act?  If so, see if it is: An exempted security or If this will be an exempted transaction y Definition of a Security 2(a)(1): The term security means any note, stock, ... bond, debenture, ... investment contract ... or, in general, any interest or instrument commonly known as a security. Debenture: form of unsecured debt instrument; no assets; just companys promise to pay Bond: secured obligation; promising assets in addition to promise to pay y Why does this definition matter? In order to come within the registration requirements of 5 of the 33 Act, the offer or sale of a property interest must constitute the offer or sale of a security. And once you have a class of securities registered under 5, you must provide the SEC and the market with continuous disclosure under the periodic disclosure system of the 1934 Act. If the item in question is NOT a security as defined in 2(1) of the 33 Act, then the registration requirements of 5 are not applicable. So your first line of inquiry is to see if you are dealing with a security.  If so, look to see if there are any exemptions from the registration requirements available. Both the 1933 and 1934 Act definitions have a laundry list of examples and categories, plus a catchall at the end. The purpose of this two-part test is to include in the definition all of the many types of instruments that in our commercial world fall within the ordinary concept of a security.  Both sections start with unless the context otherwise requires which allows instruments that would otherwise fall within the definitions to be carved out, if the context otherwise requires. y The 1933 Act defines 3 categories of securities: (1) any interest or instrument commonly known as a security: things like bonds, stock, notes, debentures and warrants Warrants: right to buy stock; operate light options (2) types of securities specifically mentioned in the Act: things like pre-organization subscriptions; fractional, undivided interests in oil, gas or other mineral rights (3) investment contracts and certificates of participation: the two most important clauses in 2(1) are its broad, catch-all phrases investment contract and certificates of interest or participation in any profit-sharing agreement. The SEC and the courts have applied these phrases to include many financial schemes not specifically mentioned by the Act as securities. Most of the case law has arisen in connection with the phrase investment contract and, to a lesser extent, to the phrase unless the context otherwise requires. The courts have fleshed out the concept of the investment contract to cover any investment scheme that triggers the investors need for the protection of the federal securities laws. y Is a LLC or Partnership Membership Interest an investment contract and thus a security? Securities Act 2(a)(1): The term security means any note, stock, ... bond, debenture, ... investment contract ... or, in general, any interest or instrument commonly known as a security. 106

Robinson v. Glynn(4th Cir. 2003)p. 406 y The parties have vigorously urged us to rule broadly in this case, asking that we generally classify interests in LLCs as investment contracts (Robinsons view) or non-securities (Glynns view). y What is an Investment Contract - SEC v. W.J. Howey, Co. (US 1946)(see p. 417 on the CB): The Howey test seeks to identify transactions in which investors are counting on others to manage the enterprise that will produce financial returns on their investments. The definition isolates transactions in which ownership is separated from control, suggesting the importance of mandatory disclosure and higher liability standards to ensure that investors allocate capital to its highest-valued uses. [1] a contract, transaction or scheme whereby a person invests money, [2] in a common enterprise, [3] and is led to expect profits [4] solely from the efforts of the promoter or a third party For us, we turn it into a 3-prong test by collapsing 3 & 4 into one step: 1. Investment of money: the investment can be cash or non cash consideration, like checks or money orders or credit. The investment part is satisfied if you put out consideration with the hope of some financial return -- producing income or profit. This excludes the purchase of a consumable commodity or service.  Anything constituting legal consideration for purposes of contract law should satisfy the first prong of the Howey test 2. Commonality/common enterprise: This prong focuses on the question of the extent to which the success of the investors interest rises and falls with others involved in the enterprise. Courts have developed two tests, depending on the circuit:  Horizontal commonality [between investors]:looks to the relationship between the individual investor and the other investors who put money into the scheme. Requires investors share the risk of the enterprise, usually through a pooling of their funds; e.g., shareholders of a corporation. Multiple investors have interrelated interests in a common scheme - they must share in a single pool of assets; the investors fortunes are interwoven - they pool resources, share profits and share losses pro rata.  Vertical commonality [between an investor and the promoter]: looks to the relationship between the investor and the promoter of the scheme. Requires the promoter and at least one investor share the risk; a single investor has a common interest with the manager of his investment; this requires a profit sharing arrangement between the promoter and each investor -- the investors fortunes are inextricably interwoven with and dependent on the fortunes of the promoter/manager of the enterprise. 3. Expectation of profit from the efforts of others: the expected return must come from earnings of the enterprise, not merely additional contributions, and this return must be the principle motivation for the investment.  Howey says that the profits must be SOLELY from the efforts of others; lower courts wrestle with whether solely means only or predominantly or substantially. There is a split in the circuits about whether to go with the more flexible interpretation.  What is the test how much effort must the promoter put into the project, as opposed to the investors efforts, in order for the expectation of profits test to be met? The critical inquiry is whether the efforts made by those other than the investor are the undeniably significant ones, those essential managerial efforts which affect the failure of success of the enterprise According to the 4th Circuit: j The Supreme Court has endorsed relaxation of the requirement that an investor rely only on others efforts, by omitting the word solely from its restatement of the Howey test. 107

And neither our court nor our sister circuits have required that an investor like Robinson expect profits solely from the efforts of others. j Requiring investors to rely wholly on the efforts of others would exclude from the protection of the securities laws any agreement that involved even slight effort from investors themselves. It would also exclude any agreement that offered investors control in theory, but denied it to them in fact. Agreements do not annul the securities laws by retaining nominal powers for investors unable to exercise them j Backed down from solely to predominantly, substantially, etc.  Idea is that if the investment returns come from other peoples efforts, you need more protection. What are the plaintiffs arguments? Argument #1: his lack of technical expertise, relative to Glynn, prevented him from meaningfully exercising his rights.  Which prong of Howey is this going to? The third prong. If you are a passive investor with no ability to control anything, the more you are depending on efforts of others  Does Robinsons argument make sense under Howey? The Howey test is applied in a flexible manner, since the Supreme Court said that the definition of investment contract embodies a flexible rather than a static principle, one that is capable of adaptation to meet the countless and variable schemes devised by those who seek the use of money of others on the promise of profits.  So how does that apply to Robinson? He was not a passive investor heavily dependent on the efforts of others like Glynn.  What does the court mean by the economic reality of the investment scheme? The court is trying to determine whether an investor is able to exercise meaningful control over his investment. Economic reality is critical in determining whether an investment vehicle qualifies as an investment contract. The relevant question is whether an investor, as a result of the investment agreement itself or the factual circumstances that surround it, is left unable to exercise meaningful control over his investment. Robinsons membership interest in the LLC, which Robinson served as treasurer and vicechairman of the board of managers, did not qualify as an investment contract, and thus was not within the definition of a security under federal securities laws, because Robinson had negotiated for a level of control that allowed him to actively protect his interest, such as by requiring his approval on indebtedness incurred outside the normal course of business, which was inconsistent with passive investment. j This guy was both a director and an officer. He is the treasurer. j Based on the facts, it seems that he knew virtually everything about the company. j Essentially, he had a way to make this work; the potential profits of the co. where w/in his hands j This guy is not the kind of guy we need to protect Argument #2: he and Glynn considered his interest in GeoPhone a security based on language in their agreement to purchase and their operating agreement. He bases this argument on the language on the restrictive legend on certificates referencing shares and securities. Argument #3: his membership interests were investment contracts and also stock.

What are the characteristics typically associated with common stock? 1) The right to receive dividends contingent upon the apportionment of profits [this says shareholders are entitled to their share of any distributed dividends, not that they are entitled to dividends per se]. 2) Negotiability 108

3) The ability to be pledged or hypothecated 4) The conferring of voting rights in proportion to the number of shares owned; and 5) The capacity to appreciate in value.  What does the courts discussion of Landreth Timber Co. v. Landreth (1985) (see p. 419 of the CB) add to this? Landreth Timber Co. v. Landreth holds that real stock is always a security: Landreth deals with whether a single individual who buys all the stock of a company and has the power to actively manage it could state a claim for fraud in his purchase. The 9th Circuit used the Howey test to support its holding that when he purchased all the stock, he was really buying a business, and the economic reality of the transaction was that he was buying a business and not an investment in a security. Landreth Timber j The Supreme Court reversed, holding that it would be burdensome to apply the Howey test to traditional stock. If something is called stock, and has the 5 common elements from Forman, its stock and there is no need to get into the investment contract analysis.  So a nice rule is: stock is always a security, except where stock is not really stock, in which case it might still be a security if its an investment contract** Wont be tested on whether something is stock-will be told it is stock On Exam: ANSWER WHAT YOU ARE ASKED Essay questions will have sub-parts Will still need to issue-spot He is NOT a passive investor (Howey not satisfied) Not stock Not a security Whats our result? Can he move forward on 10b-5?  No he cannot prove that its a security

Threshold question ALWAYS: Is it a security?


Is it stock? Is it an investment K? What would be the result if a court allowed Robinsons argument to succeed? 10b-5 would be allowed to be used in cases where it wasnt intended to apply Definition of security would be meaningless any investment would be a security What if GeoPhone had still been a corporation when Robinson bought in? Same result? When you buy into a corp. characterize the ownership interest as stock which is ALWAYS a security assuming it satisfies the 5 factors of Forman; thus, wouldnt need to get into the whole investment K thing So if it had been a corp. when he bought in, never would have come out like this What if GeoPhone was organized as a partnership? Same result? If it was a partnership, he would have a lot more control Horizontal commonality would be present But, would not satisfy the 3rd prong b/c he would be doing a lot of the work himself In a general partnership, the owners are the partners, the managers are the partners Whereas in a corp., the owners are shareholders, the managers are directors

B. THE REGISTRATION PROCESS

109

Selli Securities u er t e Securities ct f 1933

No selli g activity

Selling Securities under the Securities Act of 1933 5 prohibits the offer and sale of securities through the mail or via interstate commerce without filing a disclosure document called a registration statement with the SEC; 5 requires that the securities cant be sold until the registration statement is declared effective by the SEC; and 5 requires the delivery of a prospectus to everyone who actually buys the securities, as well as to anyone just offered the securities. Because the registration process is costly in financial and time terms, we try to avoid it if possible. There are two primary ways: to sell anexempt security -- that by its nature not subject to registration OR to sell a security that would otherwise have to be registered in an exempt transaction. Exempt securities (What you are selling) 3: Exempt securities: exempt from registration, both when issued and later when traded.  These securities were thought to be inappropriate subjects of regulation either because they were already regulated by some other governmental authority or because of the intrinsic nature of the securities themselves.  Exempt offerings remain subject to the anti fraud provisions of the securities laws; so the SEC can take action against any seller of securities under 17 - the general anti fraud provisions for the offer and sale of securities, primarily used by the SEC and the US Justice Department in criminal actions.  Just because a security is exempt under FEDERAL registration requirements doesnt necessarily mean it is exempt from state blue-sky registration, and state anti fraud rules and liabilities.  Burden of proving exemption: is on the person claiming the exemption; if the burden is not met, the exemption is lost, and if they were counting on the exemption, they have probably now violated 5 which requires registration with automatic liability for selling unregistered securities under 12(a)(1). Exempt securities included:  Government securities: are those issued or guaranteed by governmental organizations under 3(a)(2) like U.S. and state notes and bonds  Commercial paper: short term commercial paper [with a maturity of less than 9 months] is exempt they tend to be high quality, negotiable notes issued for current business operations and typically purchased by banks and other institutional sophisticated institutional investors, not the general public.  Securities subject to non- SEC regulation: some securities are exempted on the theory that other regulation offers adequate protection to investors, including: 3(a)(2): securities issued by banks; 3(a)(5): securities issued by federally or state-regulated S&Ls; 3(a)(2): insurance policies and annuity contracts issued by state-regulated insurance companies 110

EC revie adeq acy of disclos re, ot erits

Prospect s st be delivered

ALWAYS

Offers per itted b t o sales

it

ales allo ed

Registratio tate e t Filed EC

Registratio tate e t Effective

44

Securities issued by not for profit issuers: available to for organizations that operate for religious, educational, benevolent, fraternal, charitable or reformatory purposes. Exempt transactions (How you are selling it) 4:Exempt transactions: these are one-time exemptions based on the type of transaction -- the way you sell the security. So on resale, the security may need to be registered, absent some exemption.  4(1): market trading exemption: exemption for persons other than an issuer, underwriter or dealer for ordinary trading transactions like on stock exchanges or between investors.  4(2): private placement exemption exempts from registration any offering by an issuer not involving a public offering of course the laws dont define public offering. Under SEC v. Ralston Purina, the court holds that the application of this exemption turns on whether the particular class of persons affected needed the protection of the Act. An offering to those who are shown to be able to fend for themselves is a transaction not involving any public offering. Reg. D: Really 3 separate exemptions [Rules 504, 505 and 506] for private and small offerings. The exemptions are available only to the issuer who files an informational notice called a Form D with the SEC within 15 days of the first sale. Which rule you use turns on (1) the dollar amount of the offering; (2) the number and kind of investors [NOT OFFEREES]; (3) whether the Reg. D offering is part of some other larger offering; (4) the kind of advertising used; (5) the kind of information provided to investors. Relevant definitions: Accredited investor: examples include institutional investors; big organizations, key insiders [directors, executive officers and general partners of the issuer], millionaires [net worth, with spouse, of more than $1 million]; fat cats [annual income of at least $200,000 alone or $300,000 with spouse], venture capital firms Reg. D offerings have a prohibition on general solicitation and general advertising: no newspapers, magazines, television or radio ads, open seminars or investment meetings are permitted for a private placement, because since people could just walk in off the street, the issuer wouldnt be able to show that everyone who attended satisfied the sliding scale of sophistication. Reg. D offerings have resale restrictions: investors who purchase in a private placement purchase restricted securities -- they cannot resell these securities to unqualified investors without registration or another exemption. Such resales would transform the whole offering into a public distribution, and the reselling investors would become statutory underwriters.  There is a cost in selling in a private placementthat is it is restricted Funds need to be very careful to identify any restricted securities in their record keeping systems, and place blocks or liens on their securities in their trading systems so that their traders dont inadvertently sell them. The Rules: Rule 504: small offerings registered or exempt under state blue sky laws: 3(b) exemption: $1 million limit on aggregate offering price with almost no conditions, in order to assist small businesses in their efforts to raise funds. No limit on the number or kind of investors.  The SEC figured that the blue sky laws would take care of these small offerings Rule 505: medium sized offerings subject to SEC conditions: 3(b) exemption for issuers that are not investment companies and no bad boys under Reg. A. $5 million limit in any 12 month period, plus no more than 35 non accredited purchasers.  No limitation on the nature of purchasers; informational requirements for non-accredited investors. Bad boy disqualification provisions of Rule 262 apply [make it unavailable if the participant in an offering has been bad- subject to SEC disciplinary action or convicted of violating certain laws within the prior 5 years]. Rule 505 stock becomes restricted. 111

Rule 506: private offerings subject to SEC safe harbor conditions: 4(2) exemption is called a safe harbor. There is no dollar amount limitation; general advertising and solicitation is not permitted, and the offering is limited to 35 unaccredited purchasers. Plus, all the unaccredited purchasers must be knowledgeable, sophisticated and able to evaluate and bear the risks of the prospective investment. Reg. D requires issuers to take reasonable care to prevent unregistered, non exempt resales, by:  (1) having purchasers sign letters of investment intent;  (2) disclosing the securities restricted nature;  (3) legending the securities as restricted;  (4) establishing stop order procedures with transfer agents; and  (5) requiring opinions of counsel for transfer. Rule 144: provides a safe harbor system for selling restricted stock, with procedures including holding periods and limits on how much stock you can sell at once- you have to dribble it out.

Liabilities under the 1933 Act: Before the 1933 act, defrauded investors had to prove the elements of common law fraud: material misrepresentation, scienter, causation, reliance and injury. This was hard to do if the misrepresentation was an omission not an affirmative misstatement. Also, causation can be hard to demonstrate how much of the loss was from the misstatement, and how much from other factors, such as general market conditions?

Important Civil Liabilitie


  

1933 Act 11 1933 Act 12(a)(1) 1933 Act 12(a)(2)

Implied private rights of action


1934 Act 10(b) and SEC Rule 10b -5 1934 Act 14(a) and proxy rules

Important Civil Liabilities

1933 Act 11 provides private rights of action for investors who bought under a materially misleading registration statement.
11 creates an express right of action for securities purchasers when a registration statement contains untrue statements of material facts or omissions of material facts. So this is a remedy for someone who purchased a registered security either in the offering or on a post offering trading market as long as they can show the securities were part of the block sold in the offering. 1933 Act 11 Possible defendants: this section imposes express liability on:  everyone who prepares or signs the registration statement [6 tells us who has to sign] -- [issuer, CEO, CFO, chief accounting officer];  all directors [even those who didnt sign the registration statement];  underwriters, and experts like lawyers, accountants, investment bankers who have consented to be named as experts their liability is limited to the information prepared or certified by them, and certainly, most of them would prefer not to have any liability. 112

11 liability is broader than a common law fraud action. All a purchaser has to do to succeed on a 11 action is to show (1) that he bought the security in the offering; and (2) that there was a material misrepresentations in the registration statement. The plaintiff does NOT have to show scienter, causation or reliance on the misstatement. Damages are limited to the total amount of the offering, even if the plaintiffs damages are bigger. 11(e) then specifies the damages recoverable (roughly, the difference between the original price of the security and its present price). It adds that the defendant may reduce those damages by showing that part of the damages resulted, not from the misstatements in the document, but rather from other causes. Defenses We should think of the registration statement as having two parts (the parts prepared by certified experts, and everything else), and the potential defendants as constituting two groups (the experts, and everyone else). The defenses available for a misleading registration statement vary according to those two variables. Defenses Ds have depend on who they are to the company Issuers are strictly liable unless they can show one of these affirmative defenses: (1) the purchaser knew of the untruth or omission when they bought the security; OR (2) the untruth or omission is not material; OR (3) the statute of limitations has run. So for the most part issuers have liability For non issuers, there are 3 additional affirmative defenses [their standard is essentially a negligence standard but they must show they were NOT negligent]: (1) resign or take steps to resign and tell the issuer and the SEC in writing that they have done so, and disclaiming all responsibility for the relevant sections of the registration statement; (2) if the registration statement has already gone effective, written notice to the SEC and reasonable public notice; and (3) due diligence/reasonable investigation defense -- absolves defendants from liability if they had reasonable grounds for believing and did believe that there was no omission or material misstatement. 11(c)gives, as a test for reasonable investigation and reasonable belief, the level of care that a prudent person would exercise if his or her own money were at stake. The due diligence defense is popular, but not always successful. The court will look at the defendants knowledge, expertise and status with regard to the issuer, its affiliates and underwriters, and the degree of participation by the defendant in the actual registration process and preparation of the materials. 12(a)(1): Strict liability for illegal offers and sales; rescission remedy The gun-jumping statutethat is what it is called when you sell securities under 5 12(a)(1)imposes civil liability on those who sell securities in violation of 5; the purchaser just has to show that he bought the security from the defendant and that it was unregistered, and he wins.  Plaintiff gets his money back, with interest or recover rescissionary damages if he has resold the stock. The securities sales contract is rescinded. This simple and powerful private remedy enforces the registration and gun jumping requirements of 5.  Purchasers can only recover if there is a direct link with the seller-- privity requirement. Even though the purpose of this section is to promote full disclosure in offerings, there is no proof of any misrepresentation required.  This section operates like a put back to the seller within one year after the alleged 5 violation. Plus, any action has to be brought within 3 years after the security was first offered to the public, to give sellers some final date to relax after. Recall: 113

Steps: Show it is a security Show it was unregistered Get your money back 12(a)(2): Fraud in a prospectus or oral sales communication This section picks up where 11 ends; this is an express private remedy for purchaser against seller of a security for material misstatement or omission in connection with the offer and sale. Plaintiff must show: (1) sale of security; (2) through the jurisdictional means [interstate commerce or the US mail]; (3) via a prospectus or oral communication; (4) containing a material misstatement or omission; (5) by a defendant who offered or sold the security [privity required]; and (6) defendant knew or should have known of the untrue statement [so there is the possibility of a due diligence-like defense here too] Defendant is liable to the purchaser for recission or damages. Again, a privity requirement. But no need to prove reliance. Plaintiff must not have known about the misstatement or omission when he bought the security. There is a one year statute of limitations, starting from the date of actual discovery or when discovery should have been made.

Doran v. Petroleum Mgmt Co. (5th Cir. 1977)p. 413 y Facts: Doran has lots of money, degree in petroleum oil mgmt! Petroleum Mgmt. Co. organized a limited partnership to drill for oil It contacted a few people, but only Doran bought an interest  8 offerees, only 1 sale  The 8 is important this makes it a public offering Doran agreed to assume one of the firms debts  Adds value to issuer if agree to take over debt issuer owes  Undertook liability on a loan for the company Investment was part money, and part taking on this debt They were overproducing, in violation of State Law.  When they played by the state law, they ended up not doing so well!  He bought in when they were overproducing and things looked good The business started losing money and eventually defaulted on the note that Doran guaranteed Doran sued for rescission y Threshold Issue #1: Did they in fact sell him a security? Did he invest money?  Yes paid $125k Common enterprise?  Horizontal commonality: do we need more than one investor for this? No cant do it alonethere were other investors in the partnership Profits commonly derived from efforts of others? Also claims that it was not registered cannot use Section 11 y Trial Court: No evidence of material misrepresentation Defense will say not a security and lose Then they will say it is a security, but there is an exemption from registration statement because it was a PRIVATE PLACEMENT we cannot worry about them because they get their information directly from the seller  Affirmative Defense under 4(2) that it was a Private Placement y Private Placement Test Four factors: 114

Number of offerees and relationship to issuer We count the offereesrelevant # is who you made an offer too; more offerees, less likely that it is a private placement If someone is deeply involved in the issuer vs. someone passively involved; the passive investor who is new to the company arguably needs more protection Want to look at his position in co. and level of sophistication  Number of units offered Not dispositive  Size of the offering Speaks to dollar amt. you are trying to raise Not dispositive There will be a dollar amt. that is too bigwill be declared public  Manner of offering Looks to general solicitation and advertising2 prohibited things in word of private placement Which factor is the most critical?  The court tells us that the first factor is the most critical and conceptually the most problematic for the defendant. What is so special about the number of offerees?  It allows the court to ascertain the magnitude of the offering and to determine the characteristics and knowledge of the persons identified. Is the number of offerees dispositive on the issue of private placement versus public offering?  No; an offering to a small number of offerees can be public, just as an offering to many can be private. Why does the test focus on offerees and not just on those who actually purchased the stock?  To be a private placement, the entire process must have been a private affair not just the ultimate sales. Why do the offerees have to be sophisticated?  The theory is that sophisticated parties are in less need of the protections of the securities laws. Why should the court care about the offerees relationship to the issuer?  The relationship can help us understand what kind of information was available to the offerees as a result of the relationship. What does sophistication mean in this context? Manolo shoes and birken bags?  No, it means sophistication in terms of business knowledge. Holding: Reversed. Not a private placement. Ct. focuses very much on fact that these guys, particularly Duran had information a registration would have afforded a prospective investor in a public offering.

Escott v. BarChris Construction Corp. (S.D.N.Y. 1968)p. 421 y How did it run its business? BarChris would enter into contracts with customers, take a small down payment and then build the bowling alley, with the customer paying the balance of the contract price in installment notes. BC discounted these notes with a factor and got back a portion of the face value in cash, and the factor held back part of the face value as a reserve. BC also engaged in a version of the traditional sale and leaseback arrangement. BC would construct and install the equipment in a building, and then sell the interior to a factor, who would pay the full contract price. The factor would then lease the interior to BCs customer or back to a subsidiary of BC, who would then lease it to the customer. y What are convertible subordinated debentures? 115

y y y y

They are unsecured debt instruments that can be converted into something else- presumably common stock. They are subordinated, meaning they are not in a priority position- there is some other debt ahead of them. Under what statute are they suing, and who are they suing? The debenture holders brought this class action under 11 of the 1933 Act. They sued: (i) the signors of the registration statement, including the issuer BarChris and its 9 directors and the controller,  What is the background of these defendants? (ii) the underwriters (Drexel), and (iii) the accountants (Peat Marwick). Under 11, the following parties are liable if the registration statement is materially misleading: those who sign the registration statement, directors, ex-perts, and underwriters ( 11(a)).  The issuer is liable because it must sign the registration statement; it has no defenses.  The other parties may have the so called due diligence defenses of 11(b). Did the registration statement contain false statements of fact or omit to state facts necessary to prevent it from being misleading? If so, were these misstatements or omissions material? If so, are there any available defenses for these defendants? Lets take a look at the various defendants, their arguments/defenses and how the court viewed them: Vitolo (President) and Pugliese (VP)directors Kircher (Treasurer)director Birnbaumdirector Auslanderdirector Grant (outside counsel)director Peat, Marwickoutside auditors What about damages? The court notes that the defendants may lower their liability by showing that the losses were attributable to events other than the misstatements. It tables the issue, however, for hearings on each plaintiffs individual claims. Arguably, the issue is common to all. To the extent that the misstatements were material, they affected the price of the securities: i.e., people paid more for the debentures than they would have paid had the statements been true. That premium attributable to the misstatements should not have varied across plaintiffs. If so, the court should have dealt with the issue in this opinion. Integrated Disclosure System: Both the 1933 and the 1934 Acts require disclosure; the 1933 Act requires disclosure with respect to a particular transaction. Thereafter, the 1934 Act requires disclosure about a particular issuer, on a continual basis. What is a registration statement? What forms are available to what issuers? Form S-1: info on the registrant and the transaction; this form contains the most detailed set of instructions and must be used by companies that do not qualify to use another form, typically companies making an IPO.  In practice, this form is used by issuers that have not previously filed periodic reports under the 1934 Act. Form S-1 has been the basic 1933 Actregistration form for many years and requires a full description of the business and finances of the issuer. Form S-2: this second tier registration form is not used very often; this form is available to seasoned companies that have been filing periodic reports with the SEC [reporting companies] for at least 3 years, must be free of defaults on debt and not in arrears on its preferred stock dividends. The form permits incorporation by reference of the information in the companys Form 10-K and other periodic SEC filings. 116

Form S-3: info on just the transaction; this form is available to large, seasoned US companies that have been reporting companies for at least one year Form 10: Once the Form S-1 is filed and effective, the issuer has to register the particular class of securities with the SEC on Form 10. Form 10-K: I put the form and the information sheet for this form on TWEN; its the annual report containing audited financial statements, Managements discussion and analysis [MD&A] and incorporates the glossy annual report to shareholders.  Annual report that looks like a registration statement  Picks up on and updates from registration statement  Once a year Form 10-Q: also on TWEN; quarterly report for the first 3 quarters of the year, containing unaudited financial statements, and managements report on any material developments. Form 8-K: also on TWEN; to be filed when certain important events happen, within 15 days. C. RULE 10B-5 y Section 10(b): It shall be unlawful for any person, directly or indirectly, by the use of any means or instrumentality of interstate commerce or of the mails, or of any facility of any national securities exchange. (b) To use or employ, in connection with the purchase or sale of any security registered on a national securities exchange or any security not so registered, any manipulative or deceptive device or contrivance in contravention of such rules and regulations as the Commission may prescribe as necessary or appropriate in the public interest or for the protection of investors. Remember: Threshold question is whether or not it is a security (do this first) Rule 10b-5 It shall be unlawful for any person, directly or indirectly, by the use of any means or instrumentality of interstate commerce, or of the mails or of any facility of any national securities exchange, (a) To employ any device, scheme, or artifice to defraud, (b) To make any untrue statement of a material fact or to omit to state a material fact necessary in order to make the statements made, in the light of the circumstances under which they were made, not misleading, or (c) To engage in any act, practice, or course of business which operates or would operate as a fraud or deceit upon any person, in connection with the purchase or sale of any security. Elements Jurisdictional nexus Transactional nexus Material misrepresentation or omission Reliance Causation Scienter Elements: Jurisdictional nexus (it is almost impossible today to NOT get the jurisdictional nexus) It shall be unlawful for any person, directly or indirectly, by the use of any means or instrumentality of interstate commerce, or of the mails or of any facility of any national securities exchange, Transactional nexus: It shall be unlawful for any person, directly or indirectly, by the use of any means or instrumentality of interstate commerce, or of the mails or of any facility of any national securities exchange, (a) To employ any device, scheme, or artifice to defraud, 117

(b) To make any untrue statement of a material fact or to omit to state a material fact necessary in order to make the statements made, in the light of the circumstances under which they were made, not misleading, or (c) To engage in any act, practice, or course of business which operates or would operate as a fraud or deceit upon any person, in connection with the purchase or sale of any security. Material misrepresentation or omission It shall be unlawful for any person, directly or indirectly, by the use of any means or instrumentality of interstate commerce, or of the mails or of any facility of any national securities exchange, (a) To employ any device, scheme, or artifice to defraud, (b) To make any untrue statement of a material fact or to omit to state a materialfact necessary in order to make the statements made, in the light of the circumstances under which they were made, not misleading, or (c) To engage in any act, practice, or course of business which operates or would operate as a fraud or deceit upon any person, in connection with the purchase or sale of any security. Reliance It is a showing of the causal connection between a defendants misrep and the plaintiffs injury. How can this reliance be demonstrated?  One way is to show a breach of a duty to disclose material information; the SC has held that the necessary nexus between plaintiffs injury and defendants wrongful conduct is established due to the breach.  Traditionally, courts bifurcated the reliance test. If the case involved an affirmative misrepresentation, courts required the plaintiff to show that he relied on the misrepresentation. If it involved a failure to disclose, they adopted a rebuttable presumption: absent proof by the defendant to the contrary, they presumed that the plaintiff relied. What is the fraud on the market theory?  Ex: 1000 people knew about this misrep, relied on it, etc.  Rebuttable presumption that our Ps relied on integrity of market price, and that they relied on the stock price  Do NOT need to prove that you relied on the misrepresentation  When is it invoked? Invoked when there is an efficient market Have a misrepresentation out in the market that has been incorporated into the price of the stock Presume person buying stock bought relying on that misrepresentation Must be a MATERIAL, PUBLIC misrepresentation (This gets you around the reliance piece) If market is properly informed then price of securities will reflect the information out there Counts on integrity of stock price, and that the market is reliable  How can defendant rebut fraud on the market presumption? Defendant needs to sever the link between the misrep and either the price of the stock paid to/received by plaintiff or the decision to trade at the fair market price. j Market not deceived j Corrective statements Might get yourself in trouble j Specific plaintiffs would have sold anyway Causation Plaintiff must show that the misstatement caused the damage. If the corporation issued an overly optimistic press release, for example, the plaintiff must show that the statement raised the price of the security. 118

The defendant could then try to rebut the claim with evidence that the misstatement nonetheless did not contribute to the plaintiffs loss. Scienter Plaintiff must show that the defendant acted with an intent to deceive, manipulate, or defraud. Reckless disregard of falsity of statement will satisfy this requirement too. Negligence does not suffice, though numerous subsequent lower court decisions cases hold that recklessness does

Basic Inc. v. Levinson(1988)p. 438 Combustion had been negotiating a merger with Basic for 2 years Rumors about the deal persistently circulated, but Basic consistently denied them Denial 1: October 21, 1977 (stock price at $20) Denial 2: September 25, 1978 Denial 3: November 6, 1978 y Merger announced December 19, 1978 priced at $46 y Who are the plaintiffs? Claim they would have obtained a higher price had Basic not issued those denials y Why did Basic deny the rumors? FYI: Basic and Combustion agreed to keep the deal a secret at least until an agreement in principle had been reached But why?  To prevent competitive bidders from getting wind of the deal at an early stage y Did the district court certify the class? Why? Yes, they adopt the presumption of reliance (the beginning of fraud on the market) th y Did the 6 circuit agree? Why? th The 6 Cir. looks at it in a different way than the Sup. Ct. ultimately does y Why does the SC grant cert? To resolve the circuit split among the Courts of Appeals as to the standard of materiality applicable to preliminary merger discussions, and to determine whether courts below properly applied a presumption of reliance in certifying the class, rather than reqing each class member to show direct reliance on Basics statements y What tests can assist the court? Agreement in principle  Problematic and winds up not being adopted by the Sup. Ct.  Works in the reverse; if dont have the price/dont have the payment, dont have an agreement in principle  Just b/c we do have the price, doesnt mean we have enough for an agreement  Terrific idea that doesnt actually wind up being executed quite as well Basic materiality  Sup. Ct. acknowledges some kind of reasonable investor standard  Basic test is the balancing test which is a test to determine materiality  It is an alternate test to the agreement in principle y Materiality General standard of materiality?  whether there is a substantial likelihood that a reasonable shareholder would consider the fact important TSC Indus., Inc. v. Northway Inc.(1976) But how do we apply when faced with uncertain and contingent facts? Basic standard?  a highly fact-dependent probability/magnitude balancing approach Balances probability and balances magnitude
y y

119

We balance the probability of the contingent event against the magnitude of the contingent event. j Bigger the magnitude, the more likely it is to be material. j If it is unlikely and wouldnt be a big deal, that isnt going to be material. j Need to balance! Look at likelihood that the contingent event will happen
j

 

The SC takes a highly fact-dependent probability/magnitude balancing approach


based on the 2nd circuit rule in SEC v. Texas Gulf Sulphur [the materiality of speculative events will depend at any given time upon a balancing of both the indicated probability that the event will occur and the anticipated magnitude of the event in light of the totality of the company activity.] Prelim transaction that if they happen would be a big deal (merger, for example) The bigger the deal, the more material it is! There are some things that a company gets into on a material basis
j

y y

We are evaluating a failure of a BOD to say some fact Have someone saying if I would have known that things would have changed j Claimant says material misstatementreasonable shareholder would have considered that important, thus, 10(b) claim Will have a dispute as to materiality  The balancing test informs the TSC test by helping us to figure out whether the particular shareholder would consider it important  Only use the balancing test for uncertain and contingent facts Dont need it for misstatement of incomethat is just an affirmative misstatement If talking about something uncertain/contingent, in order to flush out the TSC standard and help the ct. see a reasonable investor would think it is important, need to put that contingent information in context; need to balance it out (likelihood it would happen and if it happened, how significant it would be)  ALWAYS use TSC In rejecting the agreement in principle test:  Sup. Ct. says it is about disclosure  Security and secrecy will always take a backseat to materiality Side-issue  Where 10b-5 liability is premised on an omission of material fact, liability can only arise where the defendant had a duty to disclose  Basic may not have had a duty to disclose on these facts Footnote 17 Issue not decided  They brought materiality on themselves b/c they lied  If you speak you must speak truthfully Issues: Were Basics statements materially false?  Just b/c the statements were calculated to mislead doesnt mean they are mislead  Those that will ultimately recover are those that bought and sold Is this a proper class action, when proof of reliance is an issue?  Reliance is a piece What we really take out of Basic is the adoption of Fraud-on-the-Market J. White Concurring in Part and Dissenting in Part: Agrees right standard of materiality 120

He doesnt want to pass judicially on the concept of the efficient capital market (reasonable minds can differ as to whether we have one) Take away from Basic: Idea of a real reasonable investor Take away fraud-on-the-market as being a valid theory

West v. Prudential Securities, Inc. (7th Cir. 2002)p. 463-66 y Issue: Whether the action may proceed, not on behalf of those who recd Hofmans news in person but on behalf of everyone who bought Jefferson stock during the months when Hofman was misbehaving. y Cant use FOM here. y Key w/ FOM is that the misstatement must be public so that entire class gets balanced. y Options for disgruntled shareholders 1. Statutory right of dissent and appraisal: any minority shareholder dissatisfied with the terms of the offer can (1) reject the terms; (2) obtain an independent judicial appraisal of the value of her shares; (3) receive that value in cash instead of the consideration offered in the cash merger transaction. market out exception: no appraisal rights in a stock for stock merger if the corporations shares are traded on a public market before and after the merger; the disgruntled shareholders can cash out on their own, assuming the market price is a good approximation of fair value 2. Injunction: before the consummation of the deal, a dissenting shareholder can bring a lawsuit, seeking to have the transaction halted. Grounds include lack of authority [shareholder vote], lack of compliance with statutory requirements, fraud and/or abuse of power, violation of mandatory disclosure provisions under applicable federal securities laws. 3. Rescission: after the deal has been consummated, a dissenting shareholder can bring a lawsuit for rescission. The transaction must be voidable due to lack of authority, fraud or abuse of power. Or under 12, there is a one year put back to the seller for fraud in the sale of securities.
y

Short-form Merger No shareholder approval needed  Unnecessary  Dont care what the less than 10% folks say; they dont have a choice in this Board of parent corporation approves  Dont need Board approval from the Sub; same reason as above Copy of articles of merger filed with Secretary of State and mailed to each shareholder of subsidiary corporation Parent must own at least 90% of each class of stock of the subsidiary corporation Give notice within 10 days of the effective date of the merger Shareholders can seek statutory appraisal rights

Santa Fe Indus. v. Green (1977)p. 468-73 y Santa Fe Industries held 95 percent of the stock of Kirby Lumber Corp. Santa Fe merged Kirby Lumber into itself (at $150/share) using the Delaware short-form merger statute  No shareholder vote required  Shareholders can get appraisal rights  Shareholders claim fair price = $772 Plaintiffs claims merger violated Rule 10b-5 because:  Merger was effected without prior notice to the minority shareholders and was done without any legitimate business purpose Not legally entitled to this; thus, no COA Dont need a justifiable business purpose 121

y y

Their shares had been unfairly undervalued Arguing a fraudulent undervaluing Sounds like they are saying the BOD breached their duty of care/duty of loyalty They are trying to dress of fiduciary duty claims as 10(b)(5) claims For a 10(b)(5) claim, you need a material misstatement or omission No grounds for a 10(b)(5) claim here, nothing is misstated or omitted They are trying to bootstrap a state law claim into fed. cause of action 2d Cir. says: They are going to federalize state common law. This would circumvent the whole system. Sup. Ct. Holding: If appraisal is fraudulent that is a different situation; but here they gave every piece of information in full and made a decision as directors This belongs to the state ct. to decide whether there has been a breach of the fiduciary duties of care or loyalty. Manipulation What is manipulation?  Practices that artificially affect market activity for the purpose of misleading investors Examples:  Wash sales Manipulator enters a purchase order and a sale order at the same time through the same stockbroker j Ownership of the stock does not change but creates the appearance of activity in a security  Matched orders Manipulator enters a purchase order with one stockbroker and a sale order, at the same time and at the same price, with a different broker j Sometimes involves multiple manipulators acting together (so-called cross sales) j Matched purchase and sale transactions create the false appearance of active trading No Federal Fiduciary Principle What was the Congressional intent?  Congress purpose for the 1934 Act was to assure full disclosure  Once full and fair disclosure is made, the fairness of the transaction is a non-issue under federal law  Creating a federal cause of action here thus would not serve any of the central purposes of the Act Implied private right of action  Courts generally should not create an implied federal cause of action where the matter is one traditionally relegated to state law  Here the conduct in question was a breach of fiduciary duty, a matter clearly the subject of state corporate law Federalism  Allowing Rule 10b-5 to reach reach transactions which constitute no more than internal corporate mismanagement would displace state law  Corporations are creatures of state law  Courts should not use Rule 10b-5 to preempt state corporate law Santa Fe: Was there Deception? Were plaintiffs deceived?  No omission or misstatement in merger documentation  Plaintiffs are not claiming that they were lied to; rather, they are claiming a breach of duty because the transaction was unfair Breach of fiduciary duty without fraud not a violation of Rule 10b-5 122

Terminology Whats a call option?  right to buy: call option gives the owner the right, but not the obligation, to buy a specified number of shares at a specified price Whats a put option?  right to sell: put option gives the owner the right, but not the obligation, to sell a specified number of shares at a specified price What is the option premium?  The price paid to purchase an option What is the strike or exercise price?  The price specified in the option contract at which the underlying stock can be bought or sold  The issuer of the option need not be the issuer or even a holder of the stock itself Stock prices influence option prices for example: Put: Suppose you have the right to sell stock at $25 on June 1 and you dont own the stock. You have a naked put.  You make money if the market price goes to $20, since you could then buy at the market price and immediately resell under the put and pocket a $5 gain. If the market price is above $25 on June 1, you lose the amount you paid for the put.  If you own the stock, the put merely protects you from a decline in the value of the stock below $25. It is like an insurance policy. Call: Suppose you have the right to buy stock at $25 on June 1. You make money if the market price goes to $30, since you could then buy under the call and immediately resell at the market price and pocket a $5 gain. If the market price is under $25 on June 1, you lose the amount you paid for the call.

Deutshman v. Beneficial Corp. (3d Cir. 1988)p. 474-76 y Here, they are saying things that are not true. y They are inflating the price of the stock to protect their own investments. y The duty of loyalty is written all over this. y Deutchman is not even a stockholder, but he has a COA b/c his calls are w/in the definition of security. y So if there is a fraud in connection with his buying or selling of these options, it is a 10(b)(5). y Sup. Ct. Holding: Deutschman has standing as a purchaser of an option K to seek damages under 10(b) for the affirmative misrepresentations he alleges were made by D.

SECTION 4. INSIDE INFORMATION Insider trading! y Insider trading is simplytrading on non-public information by someone with a duty not to trade. The insider can exploit his advantage whether the information is good or bad. y Example: Exxon stock prices are down because of the environmental claims against the company. Suppose that an entrepreneur with a get tough attitude toward environmentalistssay, Donald Trumpdetermines that he could raise that stock price by fighting off the environmentalists. He decides to buy up Exxon stock at the present (low) price, beat the environmental claims, sell off the Exxon stock at the new (higher) price, and live happily ever after with his fifth wife and new son. How will this work?  E stock sells at $100/share. DT will announce a tender offer at $120, conditional on receiving at least 51 percent of the stock. 123

Exxon stock will now rise to a price between $100 and $120, depending on how likely investors think DTs tender offer is to succeed. Moreover, much of the stock will fall into the hands of the arbitrageurs who buy something in one place and sell it in another place at the same time in order to make a profit from the difference in price in the two places. y Enter the inside trader Lets say a guy named Ivan cheated by buying information about tender offers from Dennis Levine, who works at Drexel Burnham Lambert. In the case of the DT tender offer, Levine would have learned about the offer if DT went to Drexel to raise funds. In the real world, IBs ties to Drexel apparently gave him two advantages: (a) sometimes he knew about forthcoming tender offers in advance and could buy stock before its price rose, and (b) sometimes he had inside information about whether a tender offer in the works was going to succeed or fail and could beat the other arbs into or out of the market. What is the harm? Who will IB have harmed if he buys Exxon stock before DT announces his offer?  Hurts people that bought it for too much b/c he drove the price up  Hurts the seller  Also hurts the integrity of the stock market  The buyer (DT) will be hurt b/c he will have to pay more y Two primary forms of insider trading: Classical insider trading: Typically, a corporate insider trades [buys or sells] shares of his corporation, using material, non public information obtained through his position as an insider. The insider exploits his informational advantage [which is a corporate asset] at the expense of the corporations shareholders.  When you buy or sell stock of a co. for which you are an insider  This constitutes a deceptive device for 10(b) and triggers the disclose or abstain rule.  The classical theory applies to traditional insiders [directors, officers, 10% shareholders] AND to temporary fiduciaries as described in Footnote 14 to Dirks. 10% is an arbitrary number Misappropriation/outsider trading: an insider can also exploit an informational advantage by trading in other companies stocks. Basically, the insider learns that his firm or a related firm will do something that affects the value of another companys stock, trading on this material non public information can also be profitable. The insider misappropriates the information at the expense of his firm, through a breach of trust or confidence. PROBLEMSp.481 y Problem 1: Principal Agency RelationshipGroves-principal, Martha-agent Do cousins owe each other a fiduciary duty? No. How did Martha learn this information?  She hired the geologist b/c she had a vibe Anything special or inside about the information?  She had information that her cousin did not have Nothing wrong w/ the K as a matter of law Difference btwn. her saying how would I know and her hiding something Absent a relationship btwn. person w/ inside info and the co. or asset that it is connected to; there is no liability When insider in co., cant trade on information b/c it violates a fiduciary duty you owe to your shareholders/corp. When 2 people are dealing at arms lengths, it is different; instead we would look for outright fraud If takes an opportunity that belongs to principal for herself; that is different that is a principalagency issue. y Problem 2: Difference here is they are partners and are in a fiduciary relationship. 124

Who you are and what your relationship is to the source of the information will be a critical question Rule 10b-5 It shall be unlawful for any person, directly or indirectly, by the use of any means or instrumentality of interstate commerce, or of the mails or of any facility of any national securities exchange, (a) To employ any device, scheme, or artifice to defraud, (b) To make any untrue statement of a material fact or to omit to state a material fact necessary in order to make the statements made, in the light of the circumstances under which they were made, not misleading, or (c) To engage in any act, practice, or course of business which operates or would operate as a fraud or deceit upon any person, in connection with the purchase or sale of any security. Insider trading is a just a sub-set of this rule

Texas Gulf Sulphur (1969)p. 482-91 y Not visible from a corporate point of view who is buying this land y Timeline: Late 1950s: TGS begins exploring eastern Canada 10/29-30/63: Exploratory hole k-55-1 drilled  Visual assay promising 11/12/63: Based on core sample results, TGS begins land acquisition  President of TGS commands secrecy 11/12/63: TGS insiders begin acquiring shares and call options 3/27/64: Land acquisition complete 4/11/64: Unauthorized press reports 4/12/64: Misleading press release issued 4/16/64: Official statement made at 10 am 4/16/64: News appeared on Dow Jones ticker tape at 10: 54 AM
The TGS Mi e The TGS Mi e

12

13

Insider Transactions Between Nov. 12, 1963 and March 31, 1964, insiders bought 7,100 shares and 12,300 call options.  Whats a call option? A rt. to buy Was the company entitled to keep the discovery quiet while it purchased the land?  Yes. Did TGS have a duty to disclose this information prior to April 16, 1964?  Dont have to disclose immediately; timing of disclosure is a matter of business judgment  They were buying up land as part of a valid business purpose; had no obligation to tell everyone What valid business reason could support the decision to delay disclosure?  Didnt want to drive up the price and didnt want others to buy up the land b/c they wanted it for themselves 125

Will see the disclose or abstain rule in the caseeither have to disclose your material information or you have to abstain from trading  If those are your choices, if cant disclose b/c of your job, then you must abstain Is TGS legally permitted to require its employees to keep quiet about this?  Yes. If TGS wanted the information kept confidential, did insiders have a right to disclose it?  Agency Restatement 395: Unless otherwise agreed, an agent is subject to a duty to the principal not to use ... information confidentially given him by the principal or acquired by him during the course of or on account of his agency or in violation of his duties as agent, in competition with or to the injury of the principal, on his own account or on behalf of another .... Given that the corporation had no duty to disclose, and had decided not to disclose the information, what options did the insiders have if they wanted to trade?  Nonethey must abstain How long does the duty to disclose or abstain run?  The information has to be public  It must be effectively disclosed in a manner sufficient to ensure its availability to the trading public When can they start trading again?  The news has to have hit the buyers  Needs to be in the public domain What happened to the stock price while the drilling was going on? It went up a lot Probably due to speculation As a factual matter, was the 4/12 press release [page 481] false? The press release said: The work done to date has not been sufficient to reach definite conclusions and any statements as to size and grade of ore would be premature and possibly misleading.  What impression did it create? What did the experts have to say about how much ore had been discovered to this point? Wont know how much is there until all done giving it up At some point during digging, will have a pretty good idea of what is there Who is suing whom? The SEC, seeking an injunction and other remedies, sued the company for issuing a misleading press release and the officers for insider trading. Co. can be a D but only purchasers or sellers can sue The Corporate Defendant Why was TGS charged with violating 10b-5? TGS was not a purchaser or seller  Status as such is only relevant to private party plaintiff standing to sue. Is the issuance of this press release in connection with the purchase or sale of a security to satisfy 10b-5?  In connection with prong is satisfied if the press release would cause reasonable investors to rely thereon and cause [such investors] to purchase or sell a corporations securities How did the 2nd circuit come to that conclusion? Where is the sale or the purchase?  The language of Rule 10b-5suggests that the drafters intended it to apply to fraud committed by someone who was buying or selling a security. Here the company was doing neither, right?  According to the court, that did not matter: a company issues a press release in connection with a purchase or sale as long as it issues a press release upon which a reasonable investor would rely in deciding whether to buy or sell. The court looks to the dominant congressional purposes underlying the 34 Act:  to promote free and open public securities markets, and to protect the investing public from suffering inequities in trading, including, specifically, inequities that follow from trading that has been stimulated by the publication of false or misleading corporate information releases. 126

y y

What did the trial court say about that first exploratory hole K-55-1? The trial court thought the results were too remote to be material. They defined materiality quite narrowly, in the fear that to do otherwise would prohibit insiders from trading and would thus deplete the ranks of capable corporate managers. What does the 2nd Circuit have to say about the trial courts fears about no one wanting to serve as a corporate manager if they dont get to trade on their inside information? How do we know when a fact is material? This is the case that started the balancing test- this is the language that the SC picked up in crafting its balancing test for Basic. What standard does the court use to determine materiality here? The court applies the balancing test [the indicate probability that the event will occur and the anticipated magnitude of the event, in light of the totality of the company activity] and find that the results were material. The court announced a reasonable investor standard for materiality: information is material if a reasonable investor would consider it important. Such an investor will consider information important if it might affect the value of the stock. What is the result of this standard on insiders? Under this materiality standard, insiders will seldom be able to defend their trades by arguing that information is immaterial. According to the court, the fact that insiders trade on a piece of information itself demonstrates its materiality. This result makes intuitive sense, but also effectively eliminates the materiality requirement in insider trading cases: no one files an insider trading case unless an insider has traded, yet the fact that an insider traded will itself be evidence of the materiality of that information. What factors did the court look in undertaking this probability/magnitude balancing? Very fact sensitive analysis Look at all the information the geologists came up which helps us figure out the probability that there is a lot of ore there Then look at the size of the co. and what that event would mean to the co. (finding the ore) Information material here b/c it was likely to occur and magnitude was high Before this case we would need the shareholder to read the press release and buy/sell Now all we need is the press release to be out there and buying and selling to occur Takes away the direct connection we needed until this case Big question: what can insiders do or not do with the stock of their firm, and when can they do it? First question- who are insiders? The court refers to 16(b)- what is that all about? 16 of the 1934 Actrestricts the conduct of officers, directors and 10% shareholders. 16(b) deals with short swing profits, a concept we will come back to. Are insiders always prohibited from investing in their own company, because of their access to inside information?  Of course not. The court is very clear on this: insiders duty to abstain or disclose ONLY arises in those situations which are essentially extraordinary in nature and which are reasonably certain to have substantial effect on the market price of the security if [the extraordinary situation is] disclosed. Does an insider have to share the benefit of her superior financial or expert analysis by disclosing her guesses or predictions?  Of course not- the courts are not aiming to entirely level the playing field. When can an insider trade on inside information?  Insiders must wait to trade until the information has been effectively disseminated, until the information is effectively disclosed in a manner sufficient to insure its availability to the investing public. Thus, those who traded before the April 16th announcement violated Rule 10b-5. But what of TGS director Coates, who placed his orders immediately after the public announcement? 127

Did it too soon; didnt wait long enough Insider Defendants What was the legal rule announced in this case?  Where an insider has material nonpublic information the insider must either disclose such information before trading or abstain from trading until the information has been disclosed Rationale for rule?  The Rule is based in policy on the justifiable expectation of the securities marketplace that all investors trading on impersonal exchanges have relatively equal access to material information.  The essence of the Rule is that anyone who has access, directly or indirectly, to information intended to be available only for a corporate purpose and not for the personal benefit of anyone may not take advantage of such information knowing it is unavailable to those with whom he is dealing, i.e., the investing public. Theory is that there is an expectation by all investors that you have equal access to trading If non-material, then it is not actionable If it is not non-public, then it is not actionable So it has to material, has to be non-public, and you have to trade on it in breach of a duty

y y y

Chiarella v. United States(1980)p. 493-94THIS CASE LEADS TO 14(e)(3)MISAPPROPRIATION THEORY


C n

Chiarella

P nd k Pr ss

T rg

Ch r

Inv s or

y y y y y y y y

y y

Chiarella traded in the target stock No violation b/c he has no connection to the target If no relationship, no fiduciary relationship and no duty to disclose or abstain b/c not an insider of the target From Dirks: only some persons, under some circumstances, will be barred from trading while in possession of material nonpublic information. From Chiarella did is called misappropriation 14(e)(3)was enacted in response to Chiarellastatutory prohibition on this conduct While what Chiarella did wasnt legal, in theory/in practice it didnt violate classical insider trading Problem w/ Chirarella, is that he is like a tippee who learned the information from his boss who learned it from the client; very remote and hard to police these kinds of parties who are so far away from the source of the information He was not liable here, but today he would be under the theory of misappropriation This case is a prelude to OHagan

Tr d


128

Tipping

Tip

Trade
Tippee
3

y y

y y

The world is divided into two categories: those who have insider trading liability and those who dont Tipper/tippee issues: what are the possible approaches for the Supreme Court in tipper/tippee cases? Anyone who obtains information from an insider picks up the insiders duty this was typically the SECs argument. The SC instead divided people who receive inside information into 3 categories:  Temporary FN 14 [to Dirks] insiders: lawyers and accountants who enter into confidential relationships with corporations and are given access to inside information for corporate purposes.  Tipees[one who gets a tip from a tipper who expects that the tippee will buy/sell based on that information]: who receive information in breach of the insiders duty to refrain from profiting on undisclosed information  Non-temporary insiders, non-tippees: they can trade freely (everybody else). What is problematic is them having the information and acting on it Insiders These folks obtain material non public information because of their role in the corporation [officer, director, employee, controlling shareholder] They have a 10b-5duty not to trade [abstain or disclose . . . ] Temporary Insiders From Dirks footnote 14; these folks are temporary [or constructive] insiders, and they are prohibited temporarily from trading in the company stock where they are temporary insiders. Lower courts have put a quasi temporary insider status on family members in situations where there are expectations of confidentiality Outsiders Outsiders [no relationship to the corporation] under OHagan have an abstain or disclose duty when they are aware of material nonpublic information that they got from a relationship of trust or confidence. These outsiders breach of confidence to the source of that information is, as a matter of law, a deception occurring in connection with the purchase/sale of securities. Tippers Both inside and outside traders who have this confidentiality duty can be liable as participants in illegal trading if they knowingly make improper tips. What makes a tip improper is if the tipper anticipates a reciprocal benefit [selling the tip, giving it to family/friends, expects the tippee to return the favor, etc.] The tipper is liable even though he/she didnt trade his/herself, as long as the tippee does. Tippees Folks with no other duty of confidentiality pick up a 10b-5duty to abstain or disclose when they knowingly trade on improper tips. So tippees are liable for trading after obtaining material, non public information that he/she knows or has reason to know came from someone who breached a duty of confidentiality. Everybody Else 129

Insider

Corpor tion

Investor

Strangers with no relationship to the source of the material non public information have no 10b-5 duty to abstain or disclose. Doesnt matter whether they overhear the material non public information or develop it themselves. Note: Only insiders, temporary insiders, and their tippees can have insider trading liability

Dirks v. SEC (1983)p. 483 CASE HELPS US FIGURE OUT WHEN WE HAVE A TIPPER/TIPPEE SITUATION y Who is Secrist? Former officer of Equity Funding of America. Alleged assets of Equity Funding, a diversified corp., primarily engaged in selling life insurance and mutual funds were vastly overstated as the result of fraudulent corporate practices. He also stated the various regulatory agencies had failed to act on similar charges made by Equity Funding employees. He is either an insider who is liable himself or he is a tipper  Left the firm so doesnt really have the duty  He is disclosing fraud He is not really the kind of guy you want to catchnot tipping in the way that you normally want to hold tippers liable y Who is Dirks? Officer of a NY broker-dealer firm who specialized in providing instrument analysis of insurance co. securities to institutional investors. He received the information from Secrist. He was urged by Secrist to verify the fraud and disclose it publically. He investigated the allegations. y Did Dirks have a financial stake in the trades of his clients made based on the information he disclosed? y What did the trial court decide? at the first hearing, the administrative law judge found that Dirks had aided and abetted violations of 10 - where tipees -- regardless of their motivation or occupation come into possession of material corporate information that they know is confidential and know or should know came from a corporate insider, they have to abstain or disclose. But since Dirks played such a huge role in uncovering the fraud, the SEC only censured him y Issue in Dirks: Did Dirks violate the antifraud provisions of the federal securities laws by disclosing material nonpublic information he got from insiders to investors who traded on the information? IS HE LIABLE AS A TIPPER? y SECs position: Dirks breached a duty that he had assumed as a result of knowingly receiving confidential information from EF insiders. The insider himself could bring the fraud to light, but Dirk, standing in their shoes, breached a fiduciary duty when he passed the information on to traders. So he could have reported the fraud, no problem. But since he told folks who traded on the information, he somehow assumes this duty. y If the SEC won, and a duty to disclose or abstain was imposed on everyone who knowingly received material nonpublic info from an insider and trades on it, what might happen? This would have an inhibiting influence on the role of the market analyst they would no longer try to sniff out information. That is b/c they are looking for information advantages. y Holding re tipping? See TEXTBOOK PAGE 487 (VERY IMPORTANT BRACKETED TEXT] In general, the tippees liability is derivative of the tippers, arising from his role as a participant after the fact in the insiders breach of a fiduciary duty. A tippee therefore can be held liable only when: 130

 The tipper breached a fiduciary duty by disclosing information to the tippee, and  The tippee knows or has reason to know of the breach of duty y Thus, Dirks is liable only if Secrist breached a fiduciary duty to EF shareholders in telling Dirks, and Dirks knew (or should have known) of that breach. Because Secrist was trying to stop fraud, he breached no fiduciary duty. y So the way tipper-tippee liability works is they are either both liability or neither are liability y What has to be true for a tipper to breach this duty? Cant trade so will get someone to do it for you Breach duty to shareholders by telling tippee to do it for you Need a personal benefit Tipper is insider who cannot trade; tipper tips tippee who trades; tipper gets a personal benefit and thus, tipper is as liable as the tippee A fiduciary duty breach occurs only where a tipper earns a personal benefit from the tip. Absent some personal gain, there is no breach of duty.  Looking at objective criteria, the courts must determine whether the insider personally will benefit, directly or indirectly, from his disclosure  Examples of personal gain: tippee pays for the tip; 2 insiders at different firms trade tips; gift; Pecuniary gain; enhanced reputation that will translate into future profits; or gifts And absent a breach of duty by the insider, there is no derivative breach. y Who is a constructive insider? Footnote 14 tells us that certain professionals can become fiduciaries of the shareholders because of the special confidential relationship, and because of their access to information. They become FN 14 insiders when they (1) obtain material nonpublic information from the issuer with (2) an expectation on the part of the corporation that the outsider will keep the disclosed information confidential and (3) the relationship at least implies such a duty Analysis questions page 500 1. Why did the court absolve Secrist of wrongdoing? The Court views 10(b)as an anti fraud provision, not as a provision designed more broadly to ensure that all participants in the market have equal access to information or that there is a level playing field.  There is no fraud, says the Court, because Secrist violated no fiduciary duty in tipping.  He violated no fiduciary duty because he received no personal benefit. 2. What is the scope of the Courts doctrine on breaches of fiduciary duty? The Court requires some personal benefit to Secrist (presumably a tangible benefitthat is, something other than that he feels good about what he did).  If Secrist and Dirks routinely exchanged stock tips, their exchange would violate Rule 10b-5 since the tipper would be receiving a benefit (future information).  If Secrist tipped Dirks out of revenge, whether S would have received a benefit (and therefore violated a fiduciary duty) is less clear.  If S carelessly discussed the fraud in the elevator, that would not constitute a fiduciary duty breach under Dirks. 3. What if Secrist disclosed inside information when Dirks bribed him, and then Dirks tipped his clients in violation of 10b-5. Did his clients also violate 10b-5? y Is there liability? o He did not bribe him in the fact pattern; if he had bribed him, Secrist gets the benefit and Dirks should have known that it was a breach of fiduciary duty Whether Dirks clients violated 10b 5would depend on whether they knew (or had reason to know) that Dirks received his information through a fiduciary duty breach.
y y

Concept: You cannot use someone else to accomplish what you cannot do What would have to change for Dirks to be liable? Could look at his motivation 131

y y

Would be different if he still worked there and if the information was different, e.g., earnings information for example instead of fraud No way insider can be liable for insider trading if his tippee is not also liable Historical background: Chiarella and Dirks created significant gaps in the insider trading prohibitions coverage. Rule 14e-3(Statute Book: p. 321) dealing with tender offers was the SECs immediate response to Chiarella, but the rules scope is very limited. It is not triggered until the offeror has taken substantial steps towards making the offer and, more important, is limited to information relating to a tender offer.  Applying it to Chiarella: He is not allowed to purchase and sell, and he did He is not allowed to cause purchase or sale, and he did He cant use the information unless it is already public  The doctrine has evolved through case law to create a misappropriation theory  Before the theory was enforced, he have this which acts as a statutory bandaid  Dont need Rule 14e-3 anymore b/c of OHagan as seen below As a result, most types of inside information remained subject to the duty-based analysis of Chiarella and its progeny. The misappropriation theory filled part of the gap, although it too is limited in that it requires a breach of fiduciary duty before trading on inside information becomes unlawful. It is not unlawful, for example, to trade on the basis of inadvertently overheard information, as in SEC v. Switzer which we talked about earlier.

United States v. OHagan (1997)p. 490 DEMONSTRATES 14(e)(3) NOT NEEDED ANYMORE
O Hagan
Grand Met

Inves or

O agan

27

y y

y y

Call optioncontract right to buy stock at X price on X day; will only exercise if the market price is more than the right you have to buy it at Issue: Is a person who trades in securities for personal profit using confidential information misappropriated in breach of fiduciary duty to the source of that information liable under Rule 10b-5 and 10(b)? Also, did the SEC exceed its rulemaking authority under 14(e) by adopting rule 14e-3(a) which prohibits trading on inside information in a tender offer, even absent a duty to disclose? Holding: The misappropriation theory is a valid basis on which to impose insider trading liability. A fiduciarys undisclosed use of information belonging to his principal, without disclosure of such use to the principal, for personal gain constitutes fraud in connection with the purchase or sale of a security and thus violatesRule 10b-5. The SEC had authority to adopt Rule 14e-3 as a prophylactic measure against insider trading in connection with a tender offer. 10b-5 is about deceptive devices; the way the doctrine is set-up is the lack of disclosure is what becomes relevant Is this case distinguishable from Santa Fe [short form squeeze out merger]? In Santa Fe, saw Ps trying to take information like this and make it into a federal claim 132

y y

y y

y y

y y

10b-5 is not the common law fraud federalized principle It is really for fraud in connection w/ the sale or purchase of a security Here, there was that kind of deception Does the statute require the deception of an identifiable purchaser or seller? No, it does not (see p. 505 of the OHagan opinion) Did O'Hagan's conduct satisfy the requirement of 10(b) that the deceptive use of information be "in connection with the purchase or sale of a security? The misappropriation at issue here was properly made the subject of a 10(b) charge because it meets the statutory requirement that there be deceptive conduct in connection with securities transactions. What's the story with 14(e)? It deals w/ tender offers Insider traders arent limited to tender offers Misappropriation is much broader and covers much more than just tender offers Why did the SEC really enact rule 14e-3? Why did the 8th Circuit think the SEC went too far with this rule? B/c the rule contains no breach of fiduciary duty reqt so they think the SEC exceeded its rulemaking authority How concerned is the SC with this issue? Not much Under a classical insider trading analysis, is O'Hagan liable? No, he was not trading in GrandMet stock He is an outsider trader in Pillsbury b/c he had no connection to them What would have to be true for him to be liable as an insider trader? So is O'Hagan an insider trader or an outside trader? He is an outside trader in Pillsbury. He owed no duty to Pillsbury or its shareholders. But he did owe a duty of trust and confidence to his law firm, and to the firm's client. So the court says this misappropriation is properly the subject of a 10(b) action since it meets the statutory requirement that there be deceptive conduct in connection with securities transactions. Note 2-United States v. Chestman (2d Cir. 1991)p. 508 What were the facts of Chestman?  An insider president/controlling shareholder tipped his sister, who tipped her daughter, who tipped her husband, who tipped his broker Chestman who traded on the info for himself and his clients. What did the 2nd circuit hold?  a person violates Rule 10b-5 when he misappropriates material nonpublic information in breach of a fiduciary duty or similar relationship of trust or confidence and uses that information in a securities transaction.  The court found that there was no fiduciary relationship between the tipee daughter and her husband, absent a showing that the husband regularly participated in confidential business decision - the family relationship was insufficient to create a fiduciary relationship on its own. So since the husband's actions didn't constitute a breach of fiduciary duty, his tipee was likewise not liable. BOTTOM LINE: In the absence of a fiduciary relationship, no liability for misappropriation. What is the statutory assistance provided by the SEC in determining when you are a fiduciary for misappropriation theory purposes? Rule 10b5-2 provides a non-exclusive list of three situations in which a person has a duty of trust or confidence for the purpose of the misappropriation theory: 1. Whenever a person agrees to maintain info in confidence; 2. Whenever the person communicating info and the person to whom it is communicated have a history, pattern or practice of sharing confidences, such that the recipient of the info knows or reasonably should know that the person communicating the info expects the recipient to maintain confidentiality; 133

y y y

or 3. Whenever the info is obtained from a spouse, parent, child or sibling, unless recipient shows that history, pattern or practice indicates no expectation of confidentiality. To avoid liability under the misappropriation theory, the misappropriater needs to disclose to the source of the information that he planned to trade on the info. Who is the source, in the case of a partnership like D&W? Can liability for insider trading be imposed solely on those who traded on the basis of material nonpublic information, or can it be broadened to include anyone who trades WHILE IN POSSESSION of such information? Law said will only hold people liable who trade on the basis of 10b-5 helps us figure that out:  10b5-1: the preliminary note tells us that this rule defines when a purchase or sale constitutes trading on the basis of material nonpublic information. (a) tells us that the manipulative and deceptive device prong is satisfied by the purchase or sale of a security on the basis of material non public information: j in breach of a duty of trust or confidence that is owed DIRECTLY, INDIRECTLY, or DERIVATIVELY to the issuer or shareholders or to the source of that material non public information.  mostly this is interpreted to mean you were aware of the information when you made the trade. So who are we talking about? Insiders [D&O, employees, controlling shareholders] Temporary insiders under Dirks Footnote 14 [accountants, lawyers, investment bankers] OutsidersWITH A DUTY TO THE SOURCE OF THE INFORMATION the outsiders breach of confidence to the information source satisfies the deception in connection with the trade Tippers: insiders/outsiders who knowingly make improper tips meaning they anticipate reciprocal benefits: sells tip, gives tip to family/friends, expects tippee to reciprocate with information  Liability extends to sub tippers who know or should know the tip is confidential and came from someone who tipped improperly  Tippers can be liable EVEN IF THEY DONT TRADE, as long as a tippee down the line eventually trades. Tipees: those with no confidentially duty will INHERIT an abstain or disclose duty IF they knowingly trade on improper tips. If tippee knows or should know the info came from a person who breached a duty in sharing, tippee is liable for trading. Ditto for sub tippees. Who are we NOT talking about? Strangers with no relationship to the source of the material, non public information they have NO duty to disclose or abstain and can freely trade. Okay, back to misappropriation: Under OHagan, we see that there can be no 10b-5 insider trading liability if there is no breach of trust or confidence. With misappropriation, note that the trade takes place outside your company Insider tradingclassical or misappropriation will not be the essay topic on the exam; could appear on multiple choice 10b-5: prohibits purchases/sales based on material information that constitutes deceptive device Inside Trader: You do it alone (classical) You dont do it yourself, but you tip someone and get the benefit back (classical) You can also outside trade (misappropriator), which is when you as an outsider misuse information Temporary Insiders are included in this

134

SECTION 5. SHORT-SWING PROFITS y Section 16(a) Every person who is directly or indirectly the beneficial owner of more than 10 per centum of any class of any equity security . . . or who is a director or an officer of the issuer of such security . . . within ten days after the close of each calendar month . . . shall file with the Commission . . . a statement indicating his ownership at the close of the calendar month and such changes in his ownership as have occurred during such calendar month  This is public and available to all of us  Who cares? They may know something so we may notif see unusual activity, may be helpful to us as investors Want to make sure they are not up to no good y Section 16(b) any profit realized by [such beneficial owner, director, or officer] from any purchase and sale, or any sale and purchase, of any equity security of such issuer . . . within any period of less than six months . . . shall inure to and be recoverable by the issuer  Arbitrary time-period  Artificial measure of what a profit isbiggest profit that could have beenmay not actually be actual dollars in the big Designed to take the largest, highest, biggest profits and have these people give it back to the corporations  Makes it so these insiders have to give the money back to the corp.  Statute says we look at a six-month clip  Short period of time; NOT trading on inside informationillegal; this trading by insiders they are not up to no good in the trades  They are just not allowed to keep money that they make y Insiders The section applies only to officers, directors, or shareholders with more than 10% of ANY class of stock. This is a smaller group of insiders than under Rule 10b-5. (which covers any person) The statute applies to officers and directors if they are officers or directors at eitherhe purchase or sale;  it applies to a 10% shareholder only if he or she held more than 10% at boththe purchase and the sale [which we will come back to in the Foremost-McKesson case in a few minutes].  Statute is harder on directors and officersgives them 6 months from the time they left the positioncant just dump it No tipping liability, no misappropriation liability, no constructive insiders only TRUE insider y Companies: The section applies only to firms that must register under the 1934 Act. This, too, is a smaller group than under Rule 10b-5 which covers public and private companies - covers all issuers  This is ONLY public 12: companies must register under the 1934 Act...  If they have a class of stock trading on a national exchange, or they have 500 or more shareholders and assets of at least $10 million  Under 12(a), all securities traded on a national exchange need to be registered [which triggers the periodic disclosure system, proxy regulation, insider trading, take over regulation] y Equity securities: The section applies only to equity securities and convertible debt. 135

y y

y y

Again, this is narrower than under Rule 10b-5 which covers any security as defined in the statute, including investment contracts. Sale and purchase: The section applies whether the sale follows the purchase or vice versa. It thus does not require that the trader earn his or her gains from buying and selling specific shares of stock. Instead, if the trader unloads 10 shares of stock and buys back 10 different shares of stock in the same company at a cheaper price, he or she is liable. Six-month period: The sale and purchase must be within a six-month period. Disgorgement: Any recovery goes to the company. Shareholders can sue derivatively, and a shareholder's lawyer can get a contingent fee out of any recovery or settlement. So the lawyers make their living on 16(b) suits. So short-swing profits are not real profitsit is a statutory calculation of the maximum profits you could have earned Maximize gains Courts interpret the statute to maximize the gains the company recovers - the court will match the highest and the lowest trade. It is hard to explain this principle without doing specific problems (as we will soon do), but let's keep this idea in mind anyway. Form and substance: Form almost always triumphs over substance in 16(b) cases. 16(b)may take the profit out of trades on inside information, though not very effectively, but may also apply to innocent trades and thereby will discourage executives from the desirable action of investing in the shares of their own companies. This is the downside of 16(b)s protection.

Reliance Electric Co. v. Emerson Electric Co. (1972)p. 511-14 y So how did Emerson get rid of the Dodge stock? After consulting with counsel, and as part of an integrated transaction, it sold 3.24% and then the remaining 9.96%, both within 6 months of the initial purchase. 6/16: buys 13.2% [becoming an insider>10%] 8/28: sells 37,000 shares --- down to 9.96% [no longer an insider < 10%] 9/11: sells remaining 9.96% y Will look at each of the trades and figure out whether the purchaser or seller was an insider at the time of the trade y What was the status of Emerson just before each transaction? NOT AN INSIDER on 6/16 when it buys 13.2% INSIDER on 8/28 when it sells 37,000 shares NOT AN INSIDER on 9/11 when it sells remaining 9.96% y Will see on exams that the 10% can go up and down changing the insider status Will see that you cant match-up trades unless you were an insider when you did it Will take a 6-month window and take all trades made will insider and will match them up y Matchable means you were an insider when the transaction was made y Need a matchable buy and a matchable sell Here, there was only one match, so no 16(b) liability here y Was the June 16 purchase a matchable purchase? Will see that the transaction that takes you over 10% does not make the transaction matchable Since that case had not been decided yet, here, it was matchable The question was not before the Ct. here y Assuming the June 16 purchase is matchable, can it be matched with the Sep 11 sale? No 136

y y y

What about with the 8/28 sale? No As long as the sales are not legally tied together, you are not going to have a problem If Foremost came first, this wouldnt have even been a case

Foremost-McKesson v. Provident Securities Co. (1976)p. 514-16 y Rule: Transaction that takes you over 10% is NOT a matchable action y Facts: Oct 20: Provident acquires debentures convertible into > 10% of Foremost stock Oct 24: Provident distributes some debentures to shareholders Oct 28: Provident sells remaining debentures y 16(b)is triggered b/c it knows Provident bought and sold in a 6-month period and they are seeking a declaratory judgment here y Issue: whether a person whose purchase of securities puts his holdings over 10% is a beneficial owner for 12(b) at the time of such purchase [triggering 16(b) liability on profits on the sale of these securities within 6 months]. y Easier way to state the issue: does "at the time of purchase" mean before the purchase or immediately after the purchase? th y Can we match Oct 20 acquisition with Oct 24 disposition? Is that Oct.20 a matchable transaction? Note: This is issue left open in Reliance y Statutory issue:This subsection shall not be construed to cover any transaction where such beneficial owner was not such both at the time of the purchase and sale, or the sale and purchase, of the security involved Why does this issue matter?  It is critical to owners like Provident who sell within six months of the acquisition that made him a 10% shareholder in the first place. y Holding? In a purchase-sale sequence, the transaction by which the shareholder crosses the 10% threshold is not a matchable purchase Only purchases effected after one becomes a 10% shareholder are matchable  So here, only the 2nd transaction is matchable b/c the second before that transaction they owned more than 10% y Code for 16(b)ABC trades on the New York Stock Exchangeit only applies to public companies Short swing profit examples on page 519: 1. Make sure the issuer is registered under the 1934 Act; 2. Evaluate each trade within the 6 month period to see if the party was a 16(b) insider when each trade was made 3. Match the highest sale prices with the lowest buy prices 4. Insider must disgorge this amount 1. Bill is the CEO of SCLaw, Inc. So he is a 16(b) insider since he is an officer [and remember, if you are an officer or director for ANY trade within the 6 months, all trades within that 6 months are matchable, even if you are no longer actually an officer or director when the trade was made]. The corporation is registered under the 1934 Act and has 1,000,000 shares outstanding. ASSUME: on 1/1, Bill buys 200,000 shares of SCLI common stock for $10 per share. What is his liability under 16(b) if: (a) he sells all 200,000 shares on 5/1 for $50 per share? y Because Bill is the CEO of an issuer registered under 12, 16(b)applies to all of his trades since he is an 137

officer and thus an insider. Okay, lets look at the trades. 1/1 he buys 200,000 shares at $10/share this is a matchable transaction since he was an officer when the purchase was made. 5/1: he sells 200,000 shares at $50/share - this is a matchable transaction since he was an officer when the sale was made. Since we have a matchable purchase and sale within 6 months, and we match them; Bill owes:  [what he got for the shares: (200,000)($50)] [what he paid for the shares: (200,000)($10)] = $8,000,000 in short swing profits under 16(b) NOTE: In this case, his real profit is the same as his short swing, but it doesn't always work out that way; it's possible to be liable for short swing profits on transactions where you really lost money on an out of pocket basis. First look for 6 months Then Look at EACH trade and see if officer was an insider the second before the trade was made and label each as matchable or unmatchable Just b/c it is matchable doesnt mean it will have a match

y y y

Problem 1(a) variation: suppose instead of buying 200,000 shares on 1/1 and selling them on 5/1, Bill sells 200,000 shares that he already owned [for more than 6 months] at $50/share on 1/1, and buys 200,000 shares on 5/1 at $10/share. Different result? y Same result. It does not matter whether he buys and then sells, or sells and then buys. He is an insider for 16(b) and he bought and sold securities within the six month period, so he is liable for 16(b)profits. y Okay, lets look at the trades, which are IDENTICAL to the prior example, except its a sale followed by a buy here, and in the prior example it was a buy followed by a sale. 1/1 he sells 200,000 shares at $50/share this is a matchable transaction since he was an officer when it was made. 5/1: he buys 200,000 shares at $10/share - this is a matchable transaction since he was an officer when it was made. We have a sale and a purchase of 200,000 shares within 6 months, and we match them; Bill owes:  [What he got for the shares: (200,000)($50)] [what he paid for the shares: (200,000)($10)] = $8,000,000 in short swing profits under 16(b) Lets go back to the original facts, where Bill buys 200,000 shares on 1/1 for $10/share and sells them on 5/1 for $50/share, but now suppose Bill is not an officer or director. What would be his liability under 16(b)? y None. If he is not an officer or director, the only way he would have 16(b)liability is if he were a more than 10% shareholder when he made the first trade. y Here that is not the case. He would not be liable, as he was not an insider when he bought the stock on 1/1 (under Foremost) so there is nothing to match the sale on 5/1 against. We have two transactions here, but only one of the transactions is matchable, so there is no other transaction to match those shares against.
y

Okay, lets look at the trades. 1/1 he buys 200,000 shares at $10/share this is NOT a matchable transaction since he did not own more than 10% of a class of stock when he made the trade. As a result of the trade, he is now an insider, however 5/1: he sells 200,000 shares at $50/share - this is a matchable transaction since he owned more than 10% of a class of stock when it was made. So we have just one matchable transaction- the sale, with nothing to match it against.

(b) Okay, assume he is an officer again! What a life Bill leads officer one day, not the next. 138

ASSUME further: on 1/1, Bill buys 200,000 shares of SCLI common stock for $10 per share. What is his liability under 16(b) if: he sells 110,000 shares on 5/1 for $50/share, and the remaining 90,000 on 5/2 for $50/share. What result? y Same result as (a) above-- $8 million in short swing liability. Because B is an officer, the 10 percent shareholding [or lack thereof] is irrelevant.
y

Okay, lets look at the trades. 1/1 he buys 200,000 shares at $10/share this is a matchable transaction since he was an officer when it was made. 5/1: he sells 110,000 shares at $50/share - this is a matchable transaction since he was an officer when it was made. 5/2: he sells 90,000 shares at $50/share - this is a matchable transaction since he was an officer when it was made.  [What he got for the shares: (110,000)($50)] - [what he paid for the shares: (110,000)($10)] = 4.4 millionPLUS[What he got for the shares: (90,000)($50)] - [what he paid for the shares: (90,000)($10)] = 3.6 million  For a total of $8 million in short swing profits under 16(b)

Suppose B was not an officer. He would not be liable, for the same reason as in (a): he was not a 10 percent shareholder when he bought the stock on 1/1 (under Foremost) so there is nothing to match the sale on 5/1 against. y Okay, lets look at the trades. 1/1 he buys 200,000 shares at $10/share this is NOT amatchable transaction since he did not own more than 10% of a class of stock when he made the trade. 5/1: he sells 110,000 shares at $50/share - this is a matchable transaction since he did own more than 10% of a class of stock when he made the trade. 5/2: he sells 90,000 shares at $50/share - this is NOT amatchable transaction since he did not own more than 10% of a class of stock when he made the trade. So we have just one matchable transaction- the sale on 5/1, with nothing to match it against.
y

Variation on 1(b): he is still an officer and executes the following trades: 1/1: buys 200,000 shares @ $10 5/1: sells 110,000 shares @ $50 5/2: sells 90,000 shares @ $40 What is his liability under 16(b):
y

Okay, lets look at the trades. 1/1 he buys 200,000 shares at $10/share this is a matchable transaction since he was an officer when it was made. 5/1: he sells 110,000 shares at $50/share - this is a matchable transaction since he was an officer when it was made. 5/2: he sells 90,000 shares at $40/share - this is a matchable transaction since he was an officer when it was made. So we have three matchable transactions within a 6 month period, so his liability is:  [(110,000)($50) - (110,000)($10)] + [90,000($40) - (90,000)($10) = $7,100,000 Remember to match the highest sale price with the lowest buy price- here it doesnt matter since the shares match up exactly.

Further variation for 1(b): Jan 1: Purchases 200,000 shares @ $1 139

May 1: Sells 110,000 shares @ $50 What is his liability under 16(b):
y

Okay, lets look at the trades. 1/1 he buys 200,000 shares at $10/share this is a matchable transaction since he was an officer when it was made. 5/1: he sells 110,000 shares at $50/share - this is a matchable transaction since he was an officer when it was made. A purchase and one matching sale occurred within 6 months, and Bill owes:  (110,000)($50) - (110,000)($10) = $4,400,000 The other 90,000 he purchased, although in a matchable transaction, have no shares sold to match against and so on these facts cannot trigger 16(b)liability.

(c) If he sells the 110,000 on 5/1 for $50/share, then resigns from SCLI and then sells the remaining 90,000 on 5/2 for $50/share, what is his liability? y Same result as (a) and (b) - $8 million in short swing liability. Officers and directors are subject to 16(b)if they are an officer or director at the time of either the purchase or the sale. But purchases or sales effected before the officer or director assumes such status are not matched, as per Rule 16a-2. 2. Lets get fancy- our next fact pattern involves Renee. Before we get into the real facts, lets try this variation: Suppose Renee who is neither an officer nor director, but who has owned owns 200,000 shares for the past two years now makes the following transactions: -Sells on 1/1, 70,000 shares at $50/share. -Sells on 2/1, 130,000 shares at $10. -Buys on 3/1, 110,000 shares at $100. -Buys on 4/1, 90,000 shares at $8.
y

y y y

In terms of real money - real profits/losses and not 16(b) profits, Renee has lost money in this series of transactions: REAL PROFITS = (Sale proceeds) - (purchase costs) =[($50 x 70,000) + ($10 x 130,000)] - [($100 x 110,000) + ($8 x 90,000)] _ ($3,500,000 + $1,300,000) - ($11,000,000 + $720,000) = -$6,920,000 In other words, she actually is out of pocket 6.9 million bucks, in real profits. Nonetheless, she is liable under 16(b) and can still have 16(b) profits. Note first that she was not a 10 percentshareholder when she made the 3/1 purchase, but that she was a 10 percent shareholder when she made the other transactions. Lets look at the trades: 1/1: she sells 70,000 shares at $50/share: this is a matchable transaction since she did own more than 10% of a class of stock when she made the trade.  She did so as a 20% shareholder at this pointthat is matchable; she is selling as an insider 2/1: she sells 130,000 shares at $10: this is a matchable transaction since she did own more than 10% of a class of stock when she made the trade.  Still a 13% owner here, so this is matchable  But at this point she loses her insider status after this trade 3/1: she buys 110,000 shares at $10: this is NOT amatchable transaction since she did not own more than 10% of a class of stock when she made the trade.  The purchase makes her an insider for after this transaction 4/1: she buys 90,000 shares at $8: this is a matchable transaction since she did own more than 10% of a class of stock when she made the trade. Okay how many shares do we have in matchable transactions that were both bought and sold? 140

200,000 sold in two matchable transactions, and 90,000 bought in matchable transactions. We can only match the total number of shares both bought and sold. So here that number is 90,000. Under the general rule of maximizing trader liability, we first match the highest sale price with the lowest buy price. Here, the highest sale price was $50/share [we have 70,000 shares] and the lowest buy price is $8/share [we have 90,000 shares]. So we can match only 70,000 of the 90,000 at $50-$8 we still have 20,000 left from the 90,000, so we go to the next lowest price on a matchable transaction which happened on 2/1.  ($50 x 70,000) - ($8 x 70,000) = $2,940,000 We then match the remaining 20,000 of the 4/1 purchase with the 2/1 sale. That results in additional liability of:  ($10 x 20,000) - ($8 x 20,000) = $40,000 Thus, Renee is liable for $2,940,000 + $40,000 = $2,980,000 under 16(b). Even though she lost real dollars, she has immense liability to the co. b/c the statutory calculation of profits when matching up the lowest possible buys with the highest possible sells this is what occurs It is a strict liability statuteno intent reqd  If you trade while an insideryou have potential liability for the next 6 months
y y y

Note: you can have a whole six months series of matchable transaction but nothing to match it against b/c all were buys or something like that Matchable focuses exclusively on the interest as an insider; not about whether the trade has a match After figuring out whether it is matchable, look at the entire period of 6 months and look at the matchable transactionsif buy a lot more than sold, will have some buys that have no sells to match with

2. Okay, now on to Renees real facts. What is her 16(b) liability if: (a) she sells 200,000 shares on 1/1 at $50/share; buys 50,000 shares on 5/1 at$10/share; buys 110,000 shares on 5/2 at $10/share y Renee was a 10 percent shareholder when she sold, but not when she bought. Under 16(b), she would need own more than 10% on both the sale and purchase dates to have liability. Hence, we cannot match the sale with any purchase, and Renee is not liable. y Okay, lets look at the trades: 1/1 she sells 200,000 shares at $50/share this is a matchable transaction since she did own more than 10% of a class of stock when she made the trade. 5/1: she buys 50,000 shares at $10/share - this is NOT amatchable transaction since she did not own more than 10% of a class of stock when she made the trade. 5/2: she buys 110,000 shares at $10/share - this is NOT amatchable transaction since she did not own more than 10% of a class of stock when she made the trade. But this trade does take her over 10%, and shes an insider going forward from here. So we have just one matchable transaction- the sale on 1/1, with nothing to match it against. (b) She sells 200,000 shares on 1/1 at $50/share; buys 110,000 shares on 5/1 at $10/share [which puts her over 10%]; and buys 50,000 shares at $10/share. y Renee was a 10 percent shareholder when she sold, but she was not a 10 percent shareholder when she bought the first 110,000, but she was a 10 percent shareholder when she bought the next 50,000. Accordingly, she is liable for her gain those 50,000 shares- we will match them to the sale in January - the 1/1 sale and the 5/1 purchase of 50,000 both occurred when she was a 10% shareholder, so they are matchable to the extent of the 50,000 she bought on 5/1. Her 5/1 purchase of 110,000 was done when she was NOT a 10% shareholder, so no liability on that purchase; it's not matchable. y Okay, lets look at the trades: 1/1 she sells 200,000 shares at $50/share this is a matchable transaction since she did own more than 10% of a class of stock when she made the trade. 141

5/1: she buys 110,000 shares at $10/share - this is NOT amatchable transaction since she did not own more than 10% of a class of stock when she made the trade. 5/2: she buys 50,000 shares at $10/share - this is a matchable transaction since she did own more than 10% of a class of stock when she made the trade. So we have two matchable transactions- the sale on 1/1 and the buy on 5/2 to match, so her liability is:  ($50 x 50,000) - ($10 x 50,000) = $2,000,000  [what she got] - [what she paid] = 16(b) short swing profits We can only match up 50,000 shares; the remaining 150,000 of the matchable sale has nothing to match against. (c) If she sells 110,000 shares on 1/1 at $50/share; sells 90,000 shares at $50/share on 1/2; buys 300,000 shares on 5/1 for $10/share. What is her liability? y Renee was not a 10 percent shareholder when she bought her 300,000 shares on 5/1. Accordingly, we cannot match a purchase with her sale of her first 110,000 shares, and she is not liable. y Okay, lets look at the trades: 1/1 she sells 110,000 shares at $50/share this is a matchable transaction since she did own more than 10% of a class of stock when she made the trade. 1/2: she sells 90,000 shares at $50/share - this is NOT amatchable transaction since she did not own more than 10% of a class of stock when she made the trade. 5/1: she buys 300,000 shares at $10/share - this is NOT amatchable transaction since she did not own more than 10% of a class of stock when she made the trade. So we have just one matchable transaction- the sale on 1/1 with nothing to match it against, so her liability is 0 (3) Bill, still the CEO and thus an insider, buys 100,000 shares on 3/1 at $10/shares; he buys 700,000 shares on 4/1 at $90/share; sells 800,000 shares on 5/1 at $30/share. Did he make money [real money, real profits, not 16(b) profits]? y Bill makes no money here: (Sale proceeds) - (purchase costs) =($30 x 800,000) - [($10)(100,000) + ($90)(700,000)] =$24,000,000 ($1,000,000 + $63,000,000) = -$40,000,000 y Lets look at the trades: 3/1: he buys 100,000 shares at $10: this is a matchable transaction since he was an officer when it was made. 4/1: he buys 700,000 shares at $90: this is a matchable transaction since he was an officer when it was made. 5/1: he sells 800,000 shares at $30: this is a matchable transaction since he was an officer when it was made. We match the highest sell with the lowest buy first:[(100,000)($30)] [(100,000)($10)]  There are some short swing profits here Then we match the rest:[(700,000)($30) (700,000)($90)] < 0 no short swing profits to disgorge y Does he have 16(b) liability? Yes. Bill is liable under 16(b)on the 100,000 shares that he bought at $10: ($30 x 100,000) - ($10 x 100,000) = $2,000,000 y Why is there no 16(b) liability on the other 700,000? He bought them at $90, and sold them at $30, so when we match those, there is no profit to disgorge. 4. Renee owns only 5000 convertible debentures [and has owned them for years, i.e., longer than 6 months], with a face amount of $1000; purchased for $1000, convertible into 100 shares each. Renee buys 100 more of these debentures on 3/1 for $800/debenture. 142

Without converting, she sells 100 debentures on 4/1 for $900/debenture. What is her 16(b) liability? y Renee's debentures are convertible into 500,000 shares, so the total number of shares outstanding if she converted would be 1,500,000 and she would own 1/3 of them. So she is treated as owner of more than 10 percent of the equity. So she is a 16(b)insider when she buys the debentures on 3/1, and when she sells them on 4/1. y Thus, she is liable for her $10,000 profit on the purchase and sale of the 100 debentures: ($900 x 100) - ($800 x 100) = $10,000

5. If SCLI class A stock has 1,000,000 shares outstanding, and 1,000,000 shares of class B outstanding. On 3/1, Mary who is not an officer or director buys 110,000 shares of class A at $10/share; on 3/2, she buys 50,000 shares of class B at $10/share; on 4/1 she sells her 110,000 shares of Class A for $50/share and her 50,000 shares of class B for $50/share lets assume she sold the B stock first. What is her 16(b) liability? y Lets look at the trades: 3/1: she buys 110,000 Class A shares at $10: this is not a matchable transaction since she did not own more than 10% of a class of stock when she made the trade. 3/2: she buys 50,000 shares of Class B at $10: this is a matchable transaction since she did own more than 10% of a class of stock [the A class] when she made the trade. 4/1: she sells 50,000 shares of Class B at $50: this is a matchable transaction since she did own more than 10% of a class of stock [the A class] when she made the trade. 4/1: she sells 110,000 shares of Class A at $50: this is a matchable transaction since she did own more than 10% of a class of stock [the A class] when she made the trade. At the time of her 3/1 purchase, Mary does not hold 10 percent of the stock of any class. So there is no matchable transaction in Class A stock. But at the time of her March 2 purchase of class B stock, she is a 10 percent shareholder in Class A. According, she is a 10 percent shareholder of any class, and so is within the scope of 16(b)insiders, and is liable for her profits on class B stock:  ($50 x 50,000) - ($10 x 50,000) = $2,000,000 NOTE: we wouldn't match a class A sale with a class B buy, but all you need to be a 16(b) insider is to own more than 10% of the class "of any equity security" registered under 12 of the 1934 Act.
y y y y y y y

Concept w/ short swing profits is that intent is not necessary; it is strict liability To avoid liability need to make sure that matchable buys and sells are more than six months apart Make sure it is a public company or otherwise registered under the 34 Act Decide whether the second before the transaction the person was an insider or not As long as any trade happens after you have resigned w/in the 6mths after it has happened, it is matchable The only time a matchable trade wont catch you is w/in the 6 mths nothing will be matchable to IT Look for it to either be 9 mths and clearly outside the 6 mth period or 5 mths and clearly w/in it

SECTION 6. INDEMNIFICATION AND INSURANCE y Indemnification Saying a director is not liable for X gets you to the same place as saying that the director is liable for X but that the corporation will indemnify the director against X. Saying a director is not liable for X likewise gets you to the same place as buying insurance against that liability. Concept is to make someone whole w/ respect to liability y Liability Limitation Statutes 143

DGCL 102(b)(7)provides that a corporation's articles of incorporation may (but need not) contain:  A provision eliminating or limiting the personal liability of a director to the corporation or its stockholders for monetary damages for breach of fiduciary duty as a director . . . .  provided that such provision shall not eliminate or limit the liability of a director: (i) For any breach of the director's duty of loyalty to the corporation or its stockholders; (ii) for acts or omissions not in good faith or which involve intentional misconduct or a knowing violation of law; (iii) under 174of this title [relating to liability for unlawful dividends]; or (iv) for any transaction from which the director derived an improper personal benefit Noteworthy Points Applies only to directors.  Although officers also are subject to a duty of care, they are denied exculpation by charter provision. Arnold v. Society for Savings Bancorp, Inc. (Del.): As to a defendant who is both a director and an officer, a 102(b)(7)provision applies only to actions taken solely in his capacity as a director. Limits only the monetary liability of directorsequitable remedies are still available A 102(b)(7)provision is an affirmative defense for a suit for breach of duty Delaware Law: Mandatory versus Permissive Indemnification Under 145(c), the corporation must indemnify a director or officer who "has been successful on the merits or otherwise." Delaware Law: Advancement of Expenses Under 145(e), the corporation may advance expenses to the officer or director provided the latter undertakes to repay any such amount if it turns out he is not entitled to indemnification.

144

CHAPTER 6: PROBLEMS OF CONTROL SECTION 1. PROXY FIGHTS INTRODUCTION y Shareholder Meetings Annual: at a time a place proscribed in the bylaws; the shareholders vote on matters such as electing directors. Most corporate statutes require that a corporation hold a shareholders meeting at least annually.  MBCA 7.01 Special: outside the ordinary schedule of meetings.  MBCA 7.02 voting lists:prepared by the corporation prior to each shareholders meeting. record date:cut off date for ownership determination  The date that counts for the vote notice:  written announcement that is sent a reasonable length of time prior to the date of the meeting.  Notice of the special meetings must include a statement of the purpose of the meeting; business transacted is limited to the stated purpose. Resolutions: corporate business matters are presented in the form of resolutions, which shareholders vote to approve or disapprove.  Practical reality: action without a meeting Can get a written consent Typical Annual Meeting Nominating committee of the incumbent board of directors nominates a slate of directors to be elected at next annual meeting  Biggest right that common stockholders have Incumbent board identifies other issues to be put to vote At company expense:  Management prepares proxy statement and card  Management solicits shareholder votes (typically with aid of proxy solicitor) Key is if we didnt have proxies we would never have a quorum Cant use a proxy at the directors meeting b/c he/she has a fiduciary duty of care that he/she cannot delegate Unlike shareholders, they can only be present in person or on the phone 145

y y

No proxies w/ directors! Voting at annual (or special) meetings Most matters require a majority of shares present at a meeting at which there is a quorum quorum:the minimum number of shareholders required to transact business. This is usually satisfied by the presence, in person or by proxy, of 50% of the outstanding shares present, either in person or by proxy. simple v. super majority: at times, more than a simple majority will be required either by statute or by the corporate charter. Typical examples include mergers and dissolution and charter amendments. What is a plurality and when is it used?  It is like a run-off; means you got the highest number of votes which may not be a majority What is straight voting?  This is one of the two different ways to look at the election of a seat  Each seat is a separate election  Will go right on majority/minority lines

What is cumulative voting?


A method of voting designed to allow minority shareholders representation on the board of directors. When cumulative voting is permitted or required, the number of members of the board of directors to be elected is multiplied by the total number of voting shares.  The result equals the number of votes a shareholder has and this total can be cast for one or more nominees for directors. All nominees stand for election at the same time. When cumulative voting is not required either by statute or under the articles of incorporation, the entire board can be elected by a majority of shares at a shareholders meeting.  Example A corporation has 10,000 shares issued and outstanding. The minority shareholders hold only 3,000 shares, and the majority shareholders hold the other 7,000 shares. Three members of the board are to be elected. The majority shareholders nominees are Allan, Burns and Caleb. The minority shareholders nominee is Davis. Can Davis be elected to the board by the minority shareholders? j If cumulative voting is not allowed, the answer is no. Means we are using straight voting j If cumulative voting is allowed, the answer is yes. The minority shareholders have 9,000 votes among them [the number of directors to be elected times the number of shares equals 3 times 3,000 which totals 9,000 votes]. j All of these votes can be cast to elect Davis. The majority shareholders have 21,000 votes [3 times 7,000 = 21,000 votes], but these votes have to be distributed among their three nominees. j The principle of cumulative voting is that no matter how the majority shareholders cast their votes, they will not be able to elect all three directors if the minority shareholders cast all of their 9,000 votes for Davis.  Concept is you can take all of your voting power for all these different seats and stick it behind one or two  Key is the majority shareholder wont own enough votes to block the minority shareholder on everything Proxies:a shareholder (as principal) can appoint someone else (as agent) to vote on his or her behalf at the shareholders meeting. Because it is not usually practical for owners of only a few shares of stock to attend shareholders meetings, such shareholders normally give third parties written authorization to vote their shares at the meeting. As a practical matter, they are necessary to insure a quorum. Proxy voting: Shareholder appoints a proxy (a.k.a. proxy agent) to vote his/her shares at the meeting 146 

Appointment effected by means of a proxy (a.k.a. proxy card)  Can specify how shares to be voted or give agent discretion  Revocable Proxy contests A shareholder (a.k.a. the insurgent) solicits votes in opposition to the incumbent board of directors  Electoral contests: Insurgent runs a slate of directors in opposition to slate nominated by incumbent board  Issue contests: Shareholder solicits votes against some proposal Proxy contests are relatively rare- why?  Shareholders are apathetic  Proxy contests cost a lot of money Proxy fights are costly: You own 10% of a firm worth $8 million that you think is badly managed. You think you can increase the value of the firm by 20% [$1.6 million] by replacing the BOD. You can make a tender offer for all the shares or wage a proxy fight:  Tender: you need $7.2 million to buy the remaining 90% of the stock of an $8 million firm; you will then capture all the gains from the increase in value. You could also just tender for another 41% and become a majority shareholder, and get 51% of the gains from the increase in value.  Proxy fight: if you can replace the BOD, and the value does in fact increase by 20%, your holdings too will increase by 20%. And when you are successful, the firm may reimburse you for your reasonable expenses. If you are NOT successful, you pay the whole ticket. Securities Exchange Act 14(a) It shall be unlawful for any person, by use of the mails or by any means or instrumentality of interstate commerce or of any facility of a national securities exchange or otherwise, in contravention of such rules and regulations as the Commission may prescribe as necessary or appropriate in the public interest or for the protection of investors, to solicit any proxy in respect of any security registered pursuant to Section 12 of this title SEC Proxy Rules 14a-3: Incumbent directors must provide annual report before soliciting proxies for annual meeting  This is one of the SEC rules that interprets 14(a) Anyone who solicits a proxy must provide a written proxy statement BEFORE soliciting the proxy. What is a Solicitation? Solicit includes not only direct requests to furnish, revoke or withhold proxies, but also ... communications which may indirectly accomplish such a result or constitute a step in a chain of communications designed ultimately to accomplish such a result.Long Island Lighting Co. v. Barbash, 779 F.2d 793, 796 (2d Cir.1985). There are some carve-outs to this broad meaning of solicitation:  Rule 14a-1(l)(2)(iv) exempts public statements of how the shareholder intends to vote and its reasons for doing so  Rule 14a-2(b)(1), subject to numerous exceptions, exempts persons who do not seek "the power to act as proxy for a security holder" and do not furnish or solicit "a form of revocation, abstention, consent or authorization Consequently, for example, a newspaper editorial advising a vote against incumbent managers is now definitively exempted  Rule 14a-2(b)(2) exempts solicitations of 10 or fewer persons  Rule 14a-2(b)(3)exempts the furnishing of proxy voting advice by someone with whom the shareholder has a business relationship Proxy Statement:a document in which the solicitor discloses information that may be relevant to the decision the shareholder must make: annual report; disclosure of conflict of interests; issues that solicitor plans to raise at the meeting 147

What goes into a proxy statement?  Information about the meeting  Background information directly related to issues to be voted on Biographical information about candidates for director vacancies Financial data about merger partners (if any) Other proxy statement rules  Rule 14a-6(b) re filing with SEC: solicitors must file proxy materials with the SEC  Rule 14a-7 re mailing: management gets a choice: mail the materials for the solicitor, at solicitors cost, or provide a list of shareholders.  Rule 14a-9 re fraudprevents fraud in the proxy solicitation contest Cost of soliciting proxies: Uncontested meeting:Because corporate statutes require firms to hold meetings of shareholders, the law allows managers to charge the firm (and thus the shareholders) for the cost of acquiring the proxies necessary to ensure a quorum at an uncontested meeting. Contested meeting:Under current law, incumbent managers also may charge the costs to the firm when an insurgent group contests their control. A. STRATEGIC USE OF PROXIES Levin v. Metro-Goldwyn-Mayer, Inc. (S.D.N.Y. 1967)p. 523 y What is plaintiffs problem with the corporation hiring a proxy-soliciting firm? Ps complain of the manner, method and means employed by Ds in the solicitation of proxies for the coming annual meetings of MGM stockholders. The fact that in retaining the proxy-soliciting firm the bill was for their services was passed to the corporation rather than to the individuals Hiring of 4 proxy soliciting firms seems like overkill; seems like they hired 4 to block the possibility of any of them working for Levin y And with hiring the special counsel? [T]he employment at corporate expense of special counsel for the sole and exclusive and no other purpose than the waging of a proxy contest on behalf of the individual defendants who have every right to pay for his valuable services with their own private funds, particularly in view of the fact that regularly employed attorneys are available to represent the corporate interests of MGM y Issue: Whether illegal or unfair means of communication such as demand judicial intervention, are being employed by the present management? y Held: They are not and concluded that injunctive relief sought should be denied. Ct. does not find the amounts recited to be paid excessive, or the method of operation disclosed by MGM management to be unfair or illegal. It contravenes no federal statute or S.E.C. rule or regulation . . . . y In the scheme of things this just wasnt that much y A ct. is not going to step in if in the incumbent fees are not excessive and the methods used are not illegal or unfair. y Again see the reluctance of courts to get into internal corporate affairs. y Is it necessary or even appropriate to get into the defendants arguments that they have done a swell job in managing the firm? Not appropriate b/c that is w/in the business judgment of the BOD. They have to make a supportable case that hiring addtl firm didnt violate their fiduciary duties. Just a way of getting appropriate disclosure to the shareholdersright of an independent stockholder to be fully informed is of supreme importance. Ct. cannot get into the merits of the underlying proxy. B. REIMBURSEMENT OF COSTS Rosenfeld v. Fairchild Engine & Airplane Corp. (N.Y. 1955)p. 525 148

o Insurgents spent $127K to get into company. They were outsiders, and had no fiduciary duties to the shareholders o Soliciting votes to get their slate of directors in o This $127K will have to be approved by shareholders o Incumbent Board: how much did it cost them to lose their fight?  $28K o The insurgents take over, become the sitting board, and give money to the outgoing board  Each of these three payments need to be focused on Does the plaintiff allege fraud in Rosenfeld? The P is a shareholder No, the P does not. His counsel conceded that the charges were fair and reasonable, but denied that they were legal charges which may be reimbursed. What was the real fight about here and who is Mr. Ward? Real fight here was a state law corporate governance one about the K of Mr. Ward. Mr. Ward had a K like Donald in Grimes Were the payments to the old BOD ratified by the shareholders? Payment of $127k which represented reimbursement of expenses to members of the prevailing group, was expressly ratified by a 16 to 1 majority vote of the stockholders. The payments to the old BOD were not ratified nor do they need to be. Only the payments to the incumbents need to be ratified. A successful insurgent seeking reimbursement needs shareholder approval.  Key is that since they are insurgents the shareholders had no ability to control those expenses; since they did not yet have a fiduciary duty, the shareholders had no say in these expenses. Did the old BOD need shareholder approval to reimburse itself for these expenses? No, they did not b/c their decision to pay themselves back is subject to scrutiny under the BJR b/c they owe fiduciary duties to their shareholders up until the moment they get kicked out. Only the incumbents need shareholder approval. Does the court differentiate between a contested and uncontested proxy solicitation? Uncontested meetings: the court says that due to stockholder indifference, and the difficulty of procuring a quorum, the corporation has to incur reasonable and proper expenses in soliciting proxies, or there would be no valid UNCONTESTED meetings.  One slate of directors nothing to fight about Contested meetings: in event of a fight, if the BOD cant use corporate funds to defend their actions, they would be at the mercy of the insurgents who might be loaded with cash.  More than one slate of directors competing What test does the court set out? When the BOD act in good faith in a contest over policy [as opposed to a purely personal power contest], they can incur reasonable and proper expenses to solicit proxies and defend their corporate policies and dont have to sit idly by. Proxy Contests: Reimbursement of Expenses Can management use corporate funds to pay for expenses they incur in conducting their proxy solicitation?  Yes, as long as the amounts are reasonable and the contest involves policy questions rather than just a purely personal power struggleRosenfeld How do we show policy contest as opposed to personal power struggles?  It is a matter of characterization  It is very unclear Proxy Contests  What would be a reasonable expense? 149

Disclosure statements to shareholders Telephone solicitations In person visits to major shareholders j Wining and dining said shareholders j Private jet to bring major shareholders to company HQ What about reimbursement of expenses? Can the insurgent use corporate funds to pay for expenses it incurs in conducting their proxy solicitation?  Only if approved by the shareholdersRosenfeld  Key is there is no checks and balances on what the insurgents spent b/c when they spent it they had no fiduciary duties What rules do we take away from Levin and Rosenfeld? The corporation may not reimburse either party unless the dispute concerns questions of policy [NOT PERSONNEL no pure power struggles] The firm may reimburse only reasonable and proper expenses. The firm may reimburse incumbents whether they win or lose without shareholder approval, and presumably their action is protected by the business judgment ruleas long as there is a dispute over policy. The firm may reimburse insurgents for their reasonable expenses only if they win, and only if shareholders ratify the payment.  Why do the insurgents warrant reimbursement? The theory is that they benefited the corporation and the other shareholders; but to make the reimbursement invulnerable to attack they must obtain shareholder approval after full disclosure. What problems come up from these rules? The distinction between policy and personnel is largely a charade. All proxy disputes involve personnel questions, and almost all involve questions of policy. More to the point, if the allocation of several hundred thousand dollars depends on a policy disagreement, the parties will generally find one. The rule against reimbursing either party in a personnel dispute could (hypothetically) have onerous consequences for the incumbents.  Suppose existing managers are doing a fine job. Suppose further that (for whatever reason) a group of insurgents decides to take their place-but raises no policy quarrels.  If so, then the incumbents have no reimbursement. In theory, the law seems to grant the incumbents reimbursement if the parties raise policy issues, but to deny incumbents the reimbursement if the insurgents admit that the incumbents are doing a fine job but fight to take over the company anyway. Proxy Litigation: Fraud Rule 14a-9 under 1934 Act 14(a) prohibits misrepresentations or omissions of a material fact in proxy materials  No solicitation subject to this regulation shall be made by means of any proxy statement or other communication, written or oral, containing any statement which, at the time and in the light of the circumstances under which it is made, is false or misleading with respect to any material fact, or which omits to state any material fact necessary in order to make the statements therein not false or misleading or necessary to correct any statement in any earlier communication with respect to the solicitation of a proxy for the same meeting or subject matter which has become false or misleading Proxy Litigation: Other Violations Soliciting without providing proxy statement Failing to file proxy materials w/ SEC Company soliciting proxies without first providing annual report Proxy Litigation 150

Who can sue?  SEC  U.S. Justice Department  Private cause of action for shareholders? [yes after Borak and Mills]  The company itself (typically against an insurgent) Recall when there is a proxy contest there are at least two proposals out there; each proponent is telling the shareholders to let me vote your shares; the co. will not sue itself if there is fraud in the co. proxy statement; they are typically suing b/c the insurgents proxy statement is missing something C. PRIVATE ACTIONS FOR PROXY RULE VIOLATIONS J.I. Case Co. v. Borak (1964)p. 532 SHAREHOLDERS HAVE A RT. TO BRING A COA FOR A MISLEADING PROXY STATEMENT y What is the shareholder/respondents complaint? P is shareholder of JI Case It charges that a merger btwn. Case and the American Tractor Co. was effected through circulation of a false and misleading proxy statement by those proposing the merger. It alleges a violation of Section 14(a) of the SEA of 1934 w/ reference to the proxy solicitation material. It purportedly failed to disclose unlawful market manipulation Argued to be material b/c if disclosed, merger would not have been approved; would have mattered to the shareholders when voting y What does JI Case argue? They argued that Congress made no specific reference to a private right of action in Section 14(a); that, in any event, the right would not extend to derivative suits and should be limited to prospective relief only. Some of the petitioners argue the merger can be dissolved only if it was fraudulent or nonbeneficial, issues upon which the proxy material would not bear. y What is the purpose of 14(a)? The section has a broad remedial purpose to prevent management [or anyone else] from obtaining authorization for any corporate action via deceptive or inadequate disclosure in a proxy solicitation. y What does the SC have to say about that? Even though the statute itself does not specifically reference a private right of action, since a chief purpose of the statute is to protect investors, the court finds an implication that it can provide judicial relief where necessary to achieve the protection of investors. It appears clear that private parties have a right under 27 to bring suit for violation of 14(a)of the Act. Indeed, this section specifically grants the appropriate District Courts jurisdiction over all suits in equity and actions at law brought to enforce any liability or duty created under the Act. y What about Cases claim that the merger can only be judicially dissolved if it was fraudulent or non-beneficial, issues that the proxy statement would not have dealt with? The SC says the causal relationship of the proxy materials to the merger is a question of fact for a trial, and they arent going to get into it here. They do get into it in our next case, Mills v. Electric Auto-Lite Co. y What about Cases claim that the remedies available for violations of 14(a) via 27 are limited to prospective relief? No sir. If this were the case, victims of deceptive proxy statements would be stuck in state court and the SC says thats a problem- the whole purpose of 14(a) might be frustrated if the laws of a particular state didnt appropriately legislate in this area. 151

Is the shareholders claim direct or derivative, and does it matter in this case? The SC says his claim is derivative it flows from the damage done to the corporation via the merger, and NOT to any damage directly inflicted upon this shareholder. But these cases are typically treated as BOTH direct [via class actions] and derivative [on behalf of the corporation] so it really doesnt matter since the court finds a private right of action under 14(a)for direct AND derivative claims.

Mills v. Electric Auto-Lite Co. (1970)p. 547-54

AMC 1/3 Merge t aler




50%

Auto-Lite

Plaintiff alleged that the proxy materials used in connection with the shareholder vote on the merger were false and misleading. How? Alleged misleading b/c it told A-L shareholders that their BOD recommended approval of the merger w/o also informing them all 11 of A-Ls directors were nominees of Mergenthaler & were under the control and domination of Mergenthaler. Plaintiffs sought to enjoin the shareholder vote. Later sought to have the merger set aside and to obtain other appropriate relief.

Elements of 14(a) Action (1) Materiality: material misstatement or omission of material fact  How defined in Mills? Information might have been considered important by a reasonable shareholder who was in the process of deciding how to vote; in other words, the statement or omission must have a significant propensity to affect the voting process.  Modern definition? [O]mitted fact is material if there is a substantial likelihood that a reasonable shareholder would consider it important in deciding how to vote.TSC Industries v. Northway (U.S. 1976) Elements of Action Causation: how is it proven?  Defect material? If so, was proxy solicitation itself, rather than the particular defect in the solicitation materials, an essential link in the accomplishment of the transaction?  In Mills, the proxy solicitation was necessary to get enough votes to approve the merger Footnote 7 left open issue of what happens if majority shareholder could approve deal by its votes alone  Causation 152

Mill

El

i A

-Li

Mergenthaler ned about 50% of Auto-Lites stock Mergenthaler as about / owned by American Manufacturing o American had voting control of Mergenthaler and through it Auto-Lite

54

y y

In Virginia Bankshares, Inc. v. Sandberg, VBI owned 85% of a subsidiary bank. The two merged. Under Virginia law, the merger required a supermajority vote (2/3 of outstanding shares). Because VBI owned 85%, it could approve deal even if all other shareholders voted no. Solicited proxies anyway. Supreme Court: Even if there was a misleading statement in the proxy material, plaintiff cant show causation between the misleading statement and the approval of the merger, because deal would have been approved without soliciting proxies Reliance?  As with Rule 10b-5, presumed where dealing with omission  As for misrepresentations, circuit split on whether affirmative proof of reliance is required Scienter?  Most courts say negligence suffices What is the result of the 7th Circuit approach? If the company could show by a preponderance of probabilities that the merger would have passed even if the statement were not misleading, petitioners get no relief. What does this sound like? It sounds like a common law fraud test of whether the injured party relied on the misrep. How on earth can you determine reliance by this many shareholders? The 7th Circuit declined to inquire into the reliance or lack thereof by thousands of stockholders, and instead decided the issue based on a proof of fairness of the terms of the merger. Because the merger was fair, the court reasoned, the shareholders would have approved it anyway So what is the 7th Circuit test? All liability is foreclosed IF the court finds the merger was fair. Plaintiffs appeal, claiming that the 7th Circuit has interpreted Borak in a way that frustrates the statutes policy of enforcement thru private litigation. Where does the requirement that the defect needs to have a significant propensity to affect the voting come from? The express terms of rule 14a-9 require that the defect have a significant propensity to affect the voting process which is designed to make sure that a cause of action cant be established by proof of a trivial defect, or an unrelated defect. Why wasnt this enough for the 7th circuit a showing that the defect was material? The 7th Cir. added the requirement that plaintiff prove that the defect had a decisive effect on voting. Good luck with that. What rule can we take from Mills? The objective rule that: Where there has been a finding of materiality, a shareholder has made a sufficient showing of causal relationship between the violation and the injury for which he seeks redress if, as here, he proves that the proxy solicitation itself, rather than the particular defect in the solicitation materials, was an essential link in the accomplishment of the transaction.  YOU SHOW CAUSATION IS SOLICITATION WAS NECESSARY TO APPROVE THE MATTER IN OTHER WORDS, YOU MATTER! DID THEY NEED YOUR VOTES?? WAS IT NECESSARY TO APPROVE THE TRANSACTION THE DEFECT MIGHT HAVE CAUSED ME TO VOTE THE OTHER WAY, AND IF I DID NOT VOTE YOU MIGHT HAVE WON Rationale: Plaintiffs show causation if they show that the defect has a significant propensity to affect the voting process. The issue thus becomes one of materiality, as long as the proxy 153

solicitation itself, rather than the particular defect in the solicitation materials, was an essential link in the accomplishment of the transaction.  That is, if the defect is material, and if the proxy solicitation was essential, plaintiff has satisfied the requirement of proof of causation.  Here, Merganthaler had 54 percent of the shares but needed 2/3. Hence, the solicitation was necessary. Remedies Prospective  Injunction corrective disclosure Will issue amended proxy statement Retrospective  Damages Must be a monetary injury  Rescission Where deal is long-since done? Monetary equivalent ON REMAND: The plaintiffs victory turned out to be hollow: The Supreme Court explicitly noted that the mergers fairness bore on the issue of damages, and on remand the Seventh Circuit dutifully held that the plaintiffs had shown causation but not damages. See Mills v. Electric Auto Like Co., 552 F.2d 1239 (7th Cir.), cert. denied, 434 U.S. 922 (1977). Issue of fairness does come in b/c if you show there was a material misstatement in the proxy statement and the votes were necessary to approve the transaction, you have made out your prima facie case, but even though you technically make it out, you still have to show damages (that is where fairness comes in)

D. SHAREHOLDERS PROPOSALS y Rule 14a-8: Shareholder Proposals Allows qualifying shareholders to put a proposal before their fellow shareholders  And have proxies solicited in favor of them in the companys proxy statement  Expense thus borne by the company y Popular shareholder proposals: in recent years, with the increasing activism of institutional shareholders, the following have become popular shareholder proposals:  to remove of poison pills;  to require annual election of directors;  to require secret balloting;  to require that directors hold a specified minimum amount of corporate shares;  to adopt cumulative voting;  to prevent the same person from being both CEO and chair of the board; and  to prohibit greenmail.
y

Other proposals worth mentioning are: to require that a majority of the board and of all key committees be independent directors; to provide a ballot for the election of directors with more nominees than there are seats, so shareholders have a choice; to link director pay to corporate performance; to require that the compensation committee be composed entirely of independent directors, with its own compensation consultant; and to create a committee of shareholders to advise the directors. Process for Excluding a Proposal First you need grounds to exclude the proposal: 154

PERSON: Proponent fails to satisfy the eligibility standards of 14a-8(b) SUBJECT MATTER: The proposal violates one of the substantive provisions of rule 14a-8(i) Management files a notice of intent to exclude with the SEC.  Accompanied by an opinion of counsel  Under rule 14a-8(f), management must notify the shareholder-proponent of remediable deficiencies in the proposal and provide an opportunity for them to be cured. SEC Response  Staff level action: Excludable? j Issue a no-action lettera non-precedentially binding letter from the SEC; conditional response from the SEC Must be included? j Notify the issuer of possible enforcement action if the proposal is excluded Not sure? j Proposal not includible in present form, but can be cured  Review by SEC commissioner: If SEC recommends no action, proponent can file for preliminary injunction (e.g., Lovenheim) Eligibility: Timing The proposal must be submitted to the corporation at least 120 days before the date on which proxy materials were mailed for the previous year's annual shareholder's meeting. E.g., if the firm mailed its proxy materials on May 1, 2005, you count back 120 days from May 1 to determine when a proposal must be submitted to be included in the 2006 proxy statement, which works out to about January 2, 2006. SEC is surprisingly strict in enforcing this requirement. Eligibility: Holdings 14a-8(b)(1): Proponent must have owned at least 1% or $2,000 (whichever is less) of the issuer's securities for at least one year prior to the date on which the proposal is submitted. Aggregation allowed for $ amount but not for holding period.  A, owned $700 in stock for 2 years; B, owned $400 in stock for 18 months; and C, owned $1000 in stock for 13 months. Okay.  D has owned $1200 in stock for two years; E has owned $1200 in stock for two months. Not okay. Eligibility: Length 14a-8(d): Proposal plus supporting statement cannot exceed 500 words  References to websites with more information okay, but website is subject to proxy rules  Why not an improper solicitation of proxies? Rule 14a-2(l)(2)(iv) permits a proponent to make public announcements about how it intends to vote and the reasons behind the voting decision, so long as proponent does not explicitly urge others to vote the same way Eligibility: Submitting Proposals 14a-8(c): Only one proposal per corporation per year  No limit on how many companies a proponent can submit proposals to 14a-8(h):Must show up at the meeting to present the proposal in person OR Ineligible to use the rule for the following two years Substantive grounds for exclusion The proposal does not concern a proper subject for action by shareholders ((i)(1)). 155

 

A common example of this would be a pro-posal concerning the ordinary business operations of the companya point that illustrates the overlap among these rules. These kinds of decisions are, as a matter of state law, left to the BOD and not open for shareholder approval It must be an action which it is proper for shareholders to initiate  Look to state law to decide that question E.g., DGCL 141(a): The business and affairs of every corporation . . . shall be managed by or under the direction of a board of directors . . . . So we cant tell the BOD what to do If shareholders not allowed to initiate, still ok if phrased as proposal  Why do we need precatory phrasing? Rule 14a-8(i)(1): If the proposal is not a proper subject of action for shareholders under the laws of the jurisdiction of the companys organization it can be excluded  What happens if precatory [referring to a wish or advisory suggestion which does not have the force of a demand or a request which under the law must be obeyed] proposal passes and board refuses to act? BJR (aka nothing) The proposal is illegal ((i)(2)).  Crack cocaine etc. The proposal violates the proxy rules ((i)(3)).  The classic example of this is a misleading proposal, since Rule 14a-9 prohibits misleading proxy material. The proposal concerns a personal grievance or benefit ((i)(4)). The proposal relates to less than 5% of the assets, net earnings or gross sales and is not otherwise significantly related to the companys business ((i)(5)  Rule 14a-8(i)(5): the proposal can be excluded if it relates to operations which account for: less than 5 percent of the companys total assets at the end of its most recent fiscal year and for less than 5 percent of its net earnings and gross sales for its most recent fiscal year and is not otherwise significantly related to the companys business The proposal concerns a matter beyond the power of the firm to effectuate ((i)(6)).  We will see in Dole that if the proponents argued that Dole should work for a national health insurance system (probably the proposal they really wanted to include), Dole could have excluded their proposal under this exception. The proposal relates to a companys ordinary business operations ((i)(7).  This exception is taken up in both Dole and Austin The proposal relates to BOD elections ((i)(8). The proposal conflicts with a company proposal ((i)(9)  the company then is required to specify the points of conflict, so no need for shareholder proposal. The proposal has already been substantially implemented ((i)(10). The proposal is duplicative of another proposal going into the same proxy statement ((i)(11). The proposal has been submitted in the past and has not obtained much support ((i)(12)).  The issue in these shareholder proposal fights is never whether the proponents will win. They never do. The issue is instead whether they will obtain enough support to resubmit the proposal the next year. Eligibility: Repeat Proposals Per Rule 14a-8(i)(12) a proposal can be excluded if it (or a substantially similar one) was submitted  once during the preceding 5 years and got less than 3% of the vote  twice in the preceding 5 years and got less than 6% of the vote the last time it was submitted 156

3 times in the preceding 5 years and got less than 10% of the vote the last time it was submitted The proposal relates to the specific amount of dividends ((i)(13). Lovenheim v. Iroquois Brands, Ltd. (D.D.C. 1985)p. 547 y Pte operations economically significant? $79,000 in revenues out of firm-wide revenues of $141 million y Why included? Otherwise significantly related includes ethical and/or social significance y Force-Feeding Not very pleasant for geese Do shareholders care? Should it be excludable? y Plaintiff argues: the last phrase in the exception in rule 14a-8(i)(5): and is not otherwise significantly related to the issuers business saves him- he does not dispute that his proposed resolution relates to a matter of little economic significance to the corporation. But he is parsing the rule to say that the and otherwise language saves him and prohibits the company from excluding his proposal. His view is that or otherwise indicates that the drafters of the rule intended that other noneconomic tests of significance be used. y Defendant argues: Plaintiff will lose since he cant show a likelihood of prevailing on the merits with regard to the issue of whether his proposal is otherwise significantly related to the issuers business. Plaintiff will not suffer irreparable injury without a preliminary injunction, since his proposal would likely not pass anyway  Granting this injunction would have a distinctly adverse effect on the company since investors react negatively to litigation and injunctions  Granting the injunction may lead investors to conclude that the firm is mistreating animals.  Granting the injunction would be contrary to public interest in allowing business to function free from harassment, and in preventing proxy statements from being cluttered y Holding: Seems clear that the meaning of significantly related is not limited to economic significance. In light of the ethical and social significance of Ps proposal & the fact that it implicates significant levels of sales, P has shown a likelihood of prevailing on the merits w/ regard to the issue of whether his proposal is otherwise significantly related to Iroquois/Delaware business. E. SHAREHOLDER INSPECTION RIGHTS y Shareholder inspection rights Policy concerns:  Shareholders have a legitimate interest in using the proxy system to hold the board accountable  Nobody wants a junk mail distributor to get access to the shareholder list or a competitor to get access to the corporations trade secrets and other proprietary information Why would a shareholder want to inspect?  Proxy contests; SEC Rule 14a-7: in a proxy contest, incumbent management must either: Mail the insurgents proxy materials for it Provide insurgent with a copy of the stockholder list  Which would management prefer? Mail itdont want to have to give them the list  Which would the insurgent prefer? The list 157

Why would a shareholder want to inspect? Shareholder litigation remember Grimes? DGCL 220(b):  Right to inspect for any proper purpose, and to make copies and extracts from: the corporation's stock ledger, a list of its stockholders, and its other books and records  Plus a limited right for subsidiary records  What other books and records are we talking about here? Bare minimum: j Articles of incorporation j Bylaws j Minutes of board and shareholder meetings j Board or shareholder actions by written consent j SEC filings and other public records  What about contracts, correspondence, and the like? The Delaware supreme court has held that a request to access such records must be very narrowly tailored: A Section 220 proceeding should result in an order circumscribed with rifled precision. Security First Corp. v. U.S. Die Casting and Development Co, 687 A.2d 563 (Del.1997).  Section 220(b): Shareholder must make a written demand setting forth a proper purpose A proper purpose is one reasonably related to such persons interest as a stockholder Section 220(c)  If shareholder only seeks access to the shareholder list, BoP on the corporation to show that shareholder doing so for an improper reason  If shareholder seeks access to other corporate records, BoP on shareholder to prove requisite proper purpose.

Purposes
Investigate alleged corporate is nage ent Collecting infor ation relev nt to v luing shares Co unic ting with fellow shareholders in connection with a planned proxy contest

Crane Co. v. Anaconda Co. (N.Y. 1976)p. 572-75 y Facts: Crane announced a tender offer for Anaconda stock Crane asked for the shareholder list Anaconda refused y Why did Crane claim it was entitled to the shareholder list? B/c it held in excess of 11% of Anacondas common stock & its request conformed to the reqts of the Bus. Corp. Law in that inspection was not reqd for a purpose other than the business of Anaconda & Crane had not participated in the sale of any stockholder list w/in the last 5 yrs.; it desired to communicate directly w/ the stockholders to inform them of its tender offer to reply to 158

Proper

I proper tte pting to discover proprietary business infor ation for the benefit of a co petitor ecure prospects for personal business Institute strike suits

39

y y

y y

the misleading statements issued & distributed by Anaconda to its stockholders & dispel any misconceptions & facilitate the further tender of Crane debentures What does the statute require in order to get the stockholder list? If you are a resident of NY, and a shareholder of record for over 6 months, or a resident holding at least 5% of a class of stock of a foreign corporation doing business in NY, you must furnish an affidavit that your inspection is not desired for a purpose which is in the interest of a business or object other than the business of the foreign corporation and that you have not sold or helped anyone sell any such shareholder list within the past 5 years. So was Anacondas refusal of this first request proper? Yes, Crane was not a shareholder at the time Where are these corporations incorporated? Crane was incorporated in Illinois and this suit is brought in New York, where it is a foreign corporation doing business there and Anaconda was a Montana corporation. So how is this being heard in a NY court, under NY law? What does defendant argue? Anaconda argued that corporate control seeking a shareholder list to convince the shareholders to sell their stock did NOT involve the business of the corporation. What does the court say about that? According to the court, not everything affecting the shareholders will affect the corporation, but everything that affects the corporation will affect the shareholders. The court liberally construes the statute Bottom line: An insurgent who hopes to organize a proxy fight must be able to communicate with other shareholders. The proxy rules under 14 of the 1934 Act provide for and control that communication. The incumbents must either mail the insurgents material for them, or give them a copy of the share-holder list. If the incumbents choose to do the mailing, then they may charge the insurgents for their expenses. Because incumbents seldom want to part with the shareholder list, they generally choose to mail the material themselves. Is this protection sufficient? For the insurgents, 14 is often unsatisfactory.  First, not all firms are subject to 14. Rather, the provision applies only to firms registered under the 1934 Actthe same limited group subject to the 16(b) rules.  Second, the insurgents may want to communicate directly with some of the major sharehold-ers. For that, they will need the shareholder list itself. And third, the insurgents may need access to corporate re-cords other than shareholder lists. Section 14 does not provide that access. So if the federal law is insufficient, what choice do these poor insurgents have? State law options  DGCL 220(b) allows inspection for any purpose reasonably related to such person's interest as a stockholder  NY statute limited to purposes relevant to the business of the corporation  Is a tender offer related to the business of the company?

Noneconomic purposes: Pillsbury v. Honeywell, Inc. (Minn. 1971)p. 575-78 y Facts: Plaintiff belonged to an antiwar group trying to stop Honeywell from producing anti personnel fragmentation bombs for the military After buying some Honeywell stock, plaintiff requested access to Honeywells shareholder list and to corporate records relating to production of such bombs y What was plaintiff seeking to do with the list? 159

y y

y y y y

He wanted to communicate w/ other shareholders in the hope of altering Honeywells BOD and thereby changing its policy. He wanted the business records to ensure accuracy. What is plaintiffs argument? A stockholder who disagrees with management has an absolute right to inspect corporate records for purposes of soliciting proxies this is a per se proper purpose. What does Honeywell say about that? They argue that a proper purpose contemplates concern with investment return. Holding? Pillsbury lacks a proper purpose for requesting the shareholder list or corporate records Purpose based solely on Pillsburys pre-existing social and political views rather than any economic interest What would be the effect of a ruling in his favor? Does the courts decision mean that no shareholder can ever bring a suit like this, aimed at reordering a corporations priorities? How could Pillsbury have had a better chance at success? If he had tied it to an economic issue or proper business cause Look at 220(c) is there a different standard for examining the stock ledger and for examining other books and records? Yes. The Delaware statute distinguishes between demands for a shareholder list and demands for other corporate records. With the former, the firm must prove that the shareholder does not have a proper purpose. With the latter, the shareholder must prove that he or she has a proper purpose. Which is it in our case?  Here he was seeking to try to get the shareholder list  The corp. has to show it was not permitted

Exam: 160

y y

Last day to ask questions is next Friday at 5pm Format: 3 Hour Exam 25 Multiple Choice (50 Points) 2 Short Essay Questions (50 Points)How much each sub-part is worth will be stated on the exam  One w/ 3 Sub-Parts  One w/ 2 Sub-Parts Mandatory Curve Imposed Review: Use IRAC!!! One of the essays is on agency or partnership and one of the essays is on corporations Note: 14(O) is not in the statute book Agency: Understand Restatement  Understand how to create one  Different kind of authority  Agency by estoppels NO CASES, EXCEPT GRIMES AND MARX  Auerbach, Zapata might also be important Partnership:  All partners are agents but not all agents are partners! Review model question on TWEN Dont overwriteyou might go off-track and give irrelevant information and wrong information

Review Session: y Regulation D: A safe-harbor from the public offering process For private placements When issuers want to raise money by selling securities as defined but they are not selling it in a public offering which is tragically undefined in the statute Allows you to avoid the registration process as long as you keep the guest list limited You get to include people who are very smart or have a lot of moneygovt has the idea that these folks need the protection of the securities law less Would be a multiple-choice question not an essay If picking btwn. the 3 choices in Reg. D it depends on how much money you want to raise and the number of accredited investors you want to have  Accredited investordefinition in securities law that gives the person status to come out of the list for Reg. D. If you were to put something on the radio, everyone is an offeree and you have created a public offering and you need to either register or come up w/ an exception y On the exam with partnerships, everyone will put in money which starts their capital account; wont deal w/ services and such y Opera Singer Case: Helps us see the idea of an interested director transaction b/c the CEO of an co. had a wife who was an opera singer and the co. hired her BJR ended up not applying here Thinking about 141 under Delawarethe operations are managed by the BOD and they have a lot freedom to do stuff Rebuttable presumption that the directors did a good job Can be rebutted by breach of duty of loyalty or care If no breach, they are protected from decisions that ended up to be bad 161

We can give them more protection via the articles of incorp. Contract vs. Tort Liability Agency set up by P and gives someone else the power to bind me, typically in K Agency thus usually stems around K issues Additional powers the agent has that comes from the concept of respondeat superior for when agents do what you told them to do, the way you told them to do it, when you told them to do it, and so on; can possibly commit torts while doing thus Just like they can bind you in K and not themselves, they can also subject you to liability for a tort and in addition to the agent assuming there is a master-servant relationship Agency by Estoppel:  No agency but the P negligently or intentionally allowed the someone to think there was one  Apparent authority thus exists and the P is bound in K  Only exists for K  For tort liability to a P for what an agent did is imputed liability; had nothing to do with what happened; person is agent and you are master and you will both share liability for what happened  This is a K remedy that creates an agency to protect the TP Regarding piercing the corporate veil and the tort creditors vs. K creditors: Actual Implied Authority Authority that is intentionally conferred by the P on the agent Can give direct actual express authority and built in is a lot of other stuffyou have to do other things incidental to what you are expressly told to; authority to do the things not specifically stated to do the things necessary to do what was specifically stated; P can be so detailed as to create no AIA so it is not unfair to the P Bond-Posting Statute: Designed to try to stop strike suits that are meritless Will say that certain populations will have to post a bond to cover expenses in case the lawsuit that is a load of crap Will carve out people who have a large interest in the co. (those with a specific dollar amt. or # of shares) Can tackhave a group of people as the P Some kind of critical mass in terms of ownership of stock that those people wont waste time b/c they have a big enough interest in the corp. Flying TigersCase Didnt want to post a bond and make a demandclaimed it was a direct lawsuit (in those you dont have to post a bond or make a demand) If he framed the arg. differently and instead said that the BOD was disloyal in doing this b/c they benefited financially or they didnt carefully consider the options violating the duty of care then that would be a derivative lawsuit which would trigger the bond post statute and demand or a showing the demand was futile Grimes If P had made the demand before the suit what would have happened?  Once made demand you forgo having a judicial conversation about whether the demand is necessary  If you make the demand you have conceded that the Board is not impartial  You thus waive your right to demonstrate that the Board is so conflicted that they cant determine a litigation  That is the reason to not make a demand  You then go to court and argue demand futility and if you lose, then you make the demand Aronson  Precursor to Grimes 162

y y y y

 It was superseded by Grimes so we dont need it anymore Dividends: Can only issue a dividend that where the co. is financially able to do so A dividend can only be declared when lawful A BOD can only declare a lawful dividend if they chose to Eisner and Van Gorkum: Difference? Both Delaware cases Different outcomes Couple of differences in decisions faced:  Nature of underlying transaction is so much more significant in Van Gorkum than Eisner b/c it is either net worth of the co. vs. one employment K  How the Boards discharged their decisions is different At least in the Eisner case they hired an appropriate expert  The guys in Van Gorkum didnt even have the beginnings of a 141(e)defense DOL BJR rule presupposes that the directors are putting the interests of the co. first and where there is a breach of the DOL there needs be an addtl level of scrutiny b/c he doesnt get the benefit of the assumption that he was loyal BJR is a first line of defense that will save the co. if there is no breach; an addtl line of defense is entire fairness where there is breach P will argue violation of DOC/DOL. D will say no I didnt. Ct. will say were not so sure about that and then D will say ok, maybe I did, but if look to entire fairness it is ok. Section 11 is only for the registration statement and 12 is for the prospectus which is part of the registration statement Marx Sets out a 3-prong approach where directors are purported to be too conflicted Looking for a circumstance where a majority of the Board has a stake in the transaction b/c then the presumption is that they are not able to impartially evaluate whether to proceed on the litigation or not; same concern if they are too controlled; etc. In order to show impartial you sometimes have to get into a little bit of the underlying acts even though that is not what you are really seeking to showyou are trying to show that the Board is so conflicted they could not decide whether to go forward with litigation or not Here, they showed the demand was futile on one of the counts; but in the long run couldnt prove their case Partnership Profits: 2 different UPAs If the partnership does not specifically take you out of the rules, they apply If it only takes you out of some, then the rest are covered by the UPA They are default rules Default regarding profits is that they are split equally Default regarding losses is that losses get split like profits If there was a profit provision but no provision with losses you go by the agreement about profits so it would be split the same, e.g., 90:10 on both profits and losses The unrealistic example is where there is only a provision on lossesyou split losses as specified; you split the profits equally then according to the default so profits 50:50 and losses 90:10 Securities: Insiders are not allowed to trade Nor are temporary insiders. Finally, tippees cannot trade. The outsiders are not restricted 163

y y

y y y y

Two ways to be liable for insider trading:  Can trade on stock on which you are a temporary insider or insider in  Can trade on stock where you are an outsider but got information b/c of an insider in the company When you get asked to trade for someone, you are a tippee and that is classical insider trading Where in your capacity at a certain co. you learn about another co., by trading on that other co.s stock, you breach a duty of confidentiality to your co. where you are an insider then there is misappropriation theory You need a breach and a trade for either form of insider trading Proxy Litigation: Ratification: You need to affirm This is an after the fact approval of an unauthorized act Works w/ shareholders How do you ratify?  Restatement guides: Need to affirm the transaction knowing all the material facts Must be intentional IRAC these questions!!! Answer the specific question asked ONLY cite relevant lawnot so much distinguishable ones You dont have to use cases on the examcan just use the rules that come from the cases Under Del. Law to see corp. records you need to have a valid business purpose and give prior notice and then you dont get to see too much Cant get a shareholders list b/c you want your daughter to sell the most girl scout cookies and then sell order forms to the list of shareholders

164

S-ar putea să vă placă și